NBDE Part 2 100 %

¡Supera tus tareas y exámenes ahora con Quizwiz!

Which of the following is the best course of action for reducing a warfarin patient's prothrombin time prior to oral surgery? Consult a physician and consider asking the patient not to take warfarin for two days before the surgery. Consult a physician and consider asking the patient to take half the normal dose of warfarin for two days prior to surgery. Administer a platelet transfusion. Administer a warfarin antagonist like heparin. Administer vitamin K.

Consult a physician and consider asking the patient not to take warfarin for two days before the surgery. Prothrombin time is the time required by the blood to form a clot and is measured in seconds. If a patient on warfarin wants a tooth extracted and has an international normalized ratio (INR) over 3.5, the patient's physician should be consulted about how to proceed. A common practice is for the physician to tell the patient to stop taking warfarin two days prior to the day of extraction. If the tooth is extracted while the patient's INR is too high, hemostasis may become an issue due to thinning of the blood by warfarin.

Vancomycin

Mechanism of Action: Inhibits cell wall synthesis by binding to the D-Ala-D-Ala terminal of the growing peptide chain during cell wall synthesis, resulting in inhibition of the transpeptidase, which prevents further elongation and cross-linking of the peptidoglycan matrix (see glycopeptide pharm

Tooth 20 (second mandibular left premolar) represents with a periodical radiolucency and symptomatic plural necrosis. Which of the following injection types would best obtain anesthetic prior to endodontic treatment?

IAN block Nerve blocks administer local anesthetic to nerve fibers upstream of the affected tooth. Active infections at the apex can interfere with local anesthetic delivered via local infiltration techniques such as PDL, intraosseus, and intrapulpal injections

Which of the following characteristics is not commonly displayed by carcinoma in situ? Pleomorphism Loss of cellular polarity Invasion Abnormal mitosis

Invasion Invasion is not a feature shown by carcinoma in situ. The basement membrane is not breached in the case of carcinoma in situ. Carcinoma in situ, which is also known as an "in situ neoplasm," is an abnormal mass of cells. These abnormal cells grow in their initial location and do not invade the surrounding tissues. Features include abnormal mitosis, loss of cellular polarity, and pleomorphism.

Which of the following describes the most common reason for a fractured occlusal rest on a removable partial denture (RPD)? Wrongly positioned rest Defective metal Insufficient rest seat Abnormal occlusion

Insufficient rest seat Though occlusal rest failure can occur due to defective metals and abnormal occlusion, occlusal rest failure most commonly results from insufficient space for the rest.

When preparing a tooth for the fabrication of a porcelain-fused-to-metal crown, which is the most common operator error that makes it difficult for laboratory technicians to fabricate the crown? Ill-defined margins Insufficient tooth reduction Inaccurate impressions or casts Inadequate description of shades and texture of the surrounding teeth

Insufficient tooth reduction Sufficient tooth reduction is necessary to fabricate a porcelain-fused-to-metal crown with sufficient bulk and dimension to withstand masticatory forces. Dental laboratories tend to fabricate bulky and overcontoured crowns to compensate for the insufficient tooth preparations. Insufficient tooth preparations increase the likelihood of overhanging and overcontoured crowns that invite plaque deposition and induce periodontal disease.

A deviation of the mandible towards the left on opening may be a result of:

Intracapsular analysis of the left temporomandibular joint One degenerative condition of the TMJ can be ankylosis of the joint. The most common cause is trauma or fracture to the joint or condyle. Intracapsular ankylosis presents with severe restriction of opening, deviation to the affected side, and decreased lateral excursions to the contralateral side Ankylosis of the temporomandibular joint (TMJ) is an intracapsular union of the disc-condyle complex to the temporal articular surface that restricts mandibular movements, including the fibrous adhesions or bony fusion between condyle, disc, glenoid fossa, and eminence. [Anterior disc displacement of the right TMJ would cause deviation of the mandible towards the affected side-the right side]

A central giant cell granuloma would most likely be found in

Patients younger than 30 The anterior mandible Female patients

Black tea bags make good hemostatic agents due to the _____ within the tea leaves

Tannic acid Regular black tea contains tannic acid, which is a local vasoconstrictor, which assists in the management of post-operative bleeding

The fusion of teeth is an anomaly which normally involves a bacterial etiology at the morphodifferentiation stage of tooth development. anterior teeth. the splitting of a tooth bud. one more dental unit in the arch than is normal. the union of two teeth through cementum only.

anterior teeth. Fusion results from the union of two adjacent tooth buds. It is usually found in anterior teeth.

Heparin functions by preventing the conversion of factor IX to factor IXa. factor IX to factor X. prothrombin to thrombin. PTA to PTC.

prothrombin to thrombin. Heparin activates and potentiates the activity of antithrombin, which inactivates thrombin and other proteases involved in blood clotting (most notably factor Xa). Heparin plays an important role in hemostasis, preventing coagulation by directly preventing soluble fibriniogen conversion into insoluble fibrin. Heparin is secreted by basophils and mast cells.

Monitoring aminoglycosides

random draw 6-14 hours after dose Blood drawing essential to prevent toxicity and ensure optimal treatment

A phoenix abscess is also known as a:

recrudescent abscess Recrudescence is the revival of material or behavior that had previously been stabilized, settled, or diminished. In medicine, it is usually defined as the recurrence of symptoms after a period of remission or quiescence, in which sense it can sometimes be synonymous with relapse A phoenix abscess develops as the granulomatous zone becomes contaminated or infected by elements from the root canal. Diagnosis is based on the acute symptoms (pain to percussion) plus radiographic examination, which reveals a large periapical radiolucency. Note: A phoenix abscess is always preceded by asymptomatic apical periodontitis. Signs and symptoms are identical to those of an acute apical abscess, but a radiograph will show a periapical radiolucency that indicates the presence of a chronic disease. Note: The term "phoenix Abscess" is becoming obsolete. The term replacing it seems to be "an acute exacerbation of asymptomatic apical periodontitis" (yes, the definition is now the term). A granuloma is defined as a growth of granulomatous tissue continuous with the periodontal ligament resulting from pulpal death with diffusion of toxic products into the periapical area. In most cases, a granuloma is symptomless. Radiographically, one sees a well-defined area of rarefaction with some irregularities, while clinically the tooth is not sensitive. A massive invasion of pulpal contaminants will result in the formation of an acute abscess (phoenix abscess). A cyst is an inflammatory response of the periapex, that develops from preexisting granulomatous tissue (granuloma). It is characterized by a central, fluid-filled, epithelium-lined cavity surrounded by granulomatous tissue and peripheral fibrous encapsulation. It is often associated with a chronically infected tooth. The tooth may be mobile. On radiographs, one will see a well-defined area of rarefaction (radiolucency) that is limited by a continuous radiopaque, sclerotic border of bone. It is usually asymptomatic. Important: A granuloma or a cyst can only be differentially diagnosed by histological examination. A phoenix abscess is an acute exacerbation of a chronic periapical lesion. It is a dental abscess that can occur immediately following root canal treatment. Another cause is due to untreated necrotic pulp (chronic apical periodontitis).[1] It is also the result of inadequate debridement during the endodontic procedure. Risk of occurrence of a phoenix abscess is minimised by correct identification and instrumentation of the entire root canal, ensuring no missed anatomy. Treatment involves repeating the endodontic treatment with improved debridement, or tooth extraction. Antibiotics might be indicated to control a spreading or systemic infection. Phoenix abscesses are believed to be due to a changing internal environment of the root canal system during the instrumentation stage of root canal treatment, causing a sudden worsening of the symptoms of chronic periradicular periodontitis.[1] This instrumentation is thought to stimulate the residual microbes in the root canal space to cause an inflammatory reaction. These microbes are predominantly facultative anaerobic gram-positive bacteria, such as Streptococcus, Enterococcus and Actinomyces species.[2] Another cause of a phoenix abscess is a decrease in a patient's resistance to these bacteria and their products.[1]

The end plate potential of neuromuscular junctions is established through the

reduction of the resting potential at the postsynaptic area by acetylcholine. End plate potentials (EPPs) are depolarizations of skeletal muscle fibers caused by neurotransmitters binding to the postsynaptic membrane in the neuromuscular junction. When an action potential reaches the axon terminal of a motor neuron, vesicles carrying neurotransmitters (mostly acetylcholine) are exocytosed, and the contents are released into the neuromuscular junction. These neurotransmitters bind to receptors on the postsynaptic membrane and lead to its depolarization.

class 3 malocclusion

relatively protruded mandible In Class III malocclusion, the mesiobuccal cusp tip of the maxillary first permanent molar is posterior to the buccal groove of the mandibular first permanent molar

Atropine is capable of relieving ptosis in myasthenia gravis patients. stimulating μ receptors. decreasing heart rate. stimulating sweat and saliva secretion.

relieving ptosis in myasthenia gravis patients. Atropine is a competitive inhibitor of muscarinic receptors M1-M5. It causes pupil dilation by paralysis of the cilliary muscle, preventing lens accommodation. It also prevents action of acetylcholine on muscarinic receptors M1-M5, causing an increase in sinoatrial (SA) node output and an increase in heart rate. Myasthenia gravis is an autoimmune neuromuscular disease that leads to fluctuating muscle weakness and fatigability. The symptoms are caused by circulating antibodies that block acetylcholine receptors at the postsynaptic neuromuscular junction, which inhibits the excitatory effects of the neurotransmitter acetylcholine on nicotinic receptors throughout neuromuscular junction.

Ototoxicity is a serious side effect seen with chloramphenicol. tramodol. amphotericin B. lidocaine. gentamycin.

gentamycin. Gentamycin is a powerful antibiotic but has to be administered with extreme caution due to its adverse effects on the auditory nerve. Gentamycin is an aminoglycoside used in treatment of severe infections. It may cause ototoxicity and renal dysfunction. Symptoms of eighth cranial nerve damage include tinnitus, ataxia, vertigo, and dizziness.

Valproic Acid

Anticonvulsant

Which of the following describes the best management strategy for a 13-year-old presenting with a permanent second molar with a 2.5 mm carious exposure that the patient says causes her pain spontaneously? Extraction Indirect pulp capping Direct pulp capping Apexogenesis Apexification

Apexogenesis Apexogenesis allows for the root to develop to completion in a tooth where the pulp was removed from the pulp chamber but not the root canals. Apexogensis describes the procedure in which a pulpotomy is performed and the chamber is filled with a layer of mineral trioxide aggregate (MTA), leaving the root intact and still able to finish forming. A 2.5 mm exposure is too large to consider pulp capping. Pulpotomy is contraindicated in permanent teeth whose roots have finished forming, because it may induce calcifcation of the pulp cavity. Apexification involves the addition of calcium hydroxide to the canal after the pulpal tissue is completely removed. Application of calcium hydroxide induces calcification across the apex. Root canal therapy is completed following calcification of the apex.

Roots can be divided into cervical, middle, and apical thirds. Accessory canals are most commonly found in what order?

Apical, Cervical, Middle The majority of accessory canals are found in the apical third third of the root at 74%. Following this, 15% of accessory canals are found in the cervical third, while 11% are found in the middle third. This information aids dental practitioners by reinforcing the importance of fully cleaning and shaping the entire root canal system. Since the apical third of the system is the furthest away from the access opening, it is often the most difficult to clean and shape thoroughly.

Seldin retractors

Are designed to retract oral soft tissues Seldin retractors do not have a sharp edge and should not be used to retract the mucoperiosteum. They are designed similarly to Henahan retractors.

Which sound is created when there is a very slight space in between the upper and lower teeth where the air can escape and create a sibilant sound? "S" sounds "F" and "V" sounds "T" and "Th" sounds Vowel sounds

"S" sounds The "S" sound is created when there is a very slight space in between the upper and lower teeth where the air can escape and create a sibilant sound or a hissing sound. "F" and "V" sounds can only be made when the air escapes out of the mouth while the upper anterior teeth slowly close down and touch the lower lip. "T" and "Th" sounds are formed when the mouth is slightly open and the tongue contacts with the tip or lingual surface of the upper anterior teeth.

What is the amoxicillin dosage for a 20 kg child for subacute bacterial endocarditis (SBE) prophylaxis?

1,000 mg The pediatric dose of amoxicillin is 50 mg/kg of body weight up to a 2,000 mg dosage of amoxicillin. 50 mg/kg × 20 kg = 1,000 mg amoxicillin.

Which of the following may be consequences of failure to remove the smear layer before obturation? Bacterial growth within the organic substrate Restriction of disinfectant action of irritants Interference of adhesion combined with less penetration of sealer into dentinal tubules Increased permeation of gutta percha obturation material into the dentinal tubules

1,2,3 The smear layer is essentially organic material left by the process of shaping the root canal system. Tis material is comprised of dentin debris and may be contaminated by bacteria or bacteria by products. There is much debate about whether it should be removed but evidence reports that removing the smear layer is the best mode of action

The settings for the nitrous oxide and oxygen sedation device are 4 L/min. oxygen and 3 L/min. nitrous oxide. At which of the following concentrations is the oxygen being administered? 98% 75% 57% 43% 33%

57% The nitrous oxide and oxygen ratio of 4 L/min. oxygen to 3 L/min. nitrous oxide shows a 57% oxygen concentration. In order to determine the percentage of oxygen use in the nitrous oxide administration, you must divide the liters per minute of the gas desired by the total amount of gas being delivered per minute. (4 L/min.) ÷ (4 L + 3 L/min.) = (4 L/min.) ÷ (7 L/min.) = 0.57 × 100 = 57%

What is the clindamycin dosage for a 40 kg child needing subacute bacterial endocarditis (SBE) prophylaxis? 1,200 mg 1,000 mg 800 mg 600 mg 500 mg

600 mg The pediatric dose of clindamycin is 20 mg/kg of body weight, up to 600 mg. 20 mg/kg × 40 kg = 800 mg clindamycin. The clindamycin dosage should not exceed the adult dosage, which is 600 mg.

A facial asymmetry developed in a boy after fracturing the left condylar neck as a result of a bicycle accident at age 5. This type of asymmetry is usually associated with:

A left-up occlusal cant Fracture of the condylar neck will result in the creation of scar tissue and limitation of the mandibular growth both in the sagittal and vertical directions at the affected side What is an occlusal cant? One condition that not too many people are aware of is the occlusal cant. To put it simply, an occlusal cant occurs when the teeth appear to be slanted relative to the nose and lips.

Persistent pain in the maxillary premolar and molar areas, without evidence of a dental cause, would most likely be caused by:

A maxillary sinus infection Because the pain is persistent and in the region near the maxillary sinus, this non-dental concern is most likely due to a cold or sinus infection. The dentist should eliminate any possible dental causes for these symptoms and should be aware of adjacent structures and non-dental infections.

Which of the following constitues an endodontic emergency?

A patient has had tooth pain for 8 hours and cannot sleep or work True endodontic emergencies involve severe pain and impairment to ability to function. A patient who has had pain for 8 hours and cannot sleep or work constitutes a true emergency. Such patients should be seen for unscheduled visits, and not sent away for another time

An infrabony periodontal pocket consist of:

A pocket in which the bottom lies apical tot the level of the adjacent alveolar bone An infra bony periodontal pocket consists of a pocket in which the bottom lies apical to the level of the adjacent alveolar bone. These pockets may also be known as intrabony, subcostal or intra-alveolar. Pockets may involve one, two or more tooth surfaces and are most common in furcation areas [a supra bony pocket is a pocket in which the bottom is coronal to the underlying alveolar bone]

Which of the following methods for gutta-percha sterilization is most effective and useful for dental purposes?

5.25% NaOCl The most effective sterilization method for gutta-percha points is 5.25% sodium hypochlorite solution for one minute. This type of solution was found to kill bacterial spores. Do not autoclave gutta percha

According to the National Council of Radiation Protection what is the recommendation on annual limits for human exposure to whole body ionizing radiation for occupational doses relative to stochastic effects?

50mSv, (The sievert is a derived unit of ionizing radiation dose in the International System of Units and is a measure of the health effect of low levels of ionizing radiation on the human body.) Cancers and genetic effects are examples of stochastic effects. Although receiving 50 mSv of whole body radiation in a year as a result of performing one's occupation may be considered to present minimal risk, every effort should be made to keep the dose as low as possible.

When placing an amalcore restoration on an endodontically treated tooth, the minimum depth the amalgam should penetrate the canal is 1 mm. 2 mm. 3 mm. 4 mm. 5 mm.

3 mm. A depth of at least 3 mm into the root canals is essential for the retention of an amalgam restoration in an endodontically treated tooth.Amalgam depends upon mechanical retention because it does not demonstrate chemical bonding with the tooth structure.After completion of the root canal treatment, roughly 3 mm of gutta percha should be removed from the root canals using a heated instrument.The space created in the root canal should be filled and condensed with amalgam before carrying out coronal filling; this is known as the Nayyar core technique.

Tylenol® 3 contains which amount of codeine? 7.5 mg 15 mg 20 mg 30 mg 60 mg

30 mg Tylenol is a proprietary name for the drug paracetamol (acetaminophen). Tylenol has antipyretic and analgesic effects and is sold in several formulations containing codeine. The formulation of acetaminophen and codeine in Tylenol is as follows: Tylenol 1 contains 325 mg acetaminophen, 8 mg codeine. Tylenol 2 contains 300 mg acetaminophen, 15 mg codeine. Tylenol 3 contains 300 mg acetaminophen, 30 mg codeine. Tylenol 4 contains 300 mg acetaminophen, 60 mg codeine.

Which of the following describes the elimination half-life for penicillin VK?

30 minutes Penicillin VK has a range of antimicrobial activity against Gram-positive bacteria. Penicillin VK demonstrates a short, 30-minute half-life because it is rapidly excreted in the urine. Roughly 90% of the excretion of penicillin VK occurs through active tubular transport.

Which of the following bleaching solutions is known to cause resorption?

30% hydrogen peroxide Clinical reports and histological studies have shown that internal bleaching may induce external root resorption. The oxidizing agent, particularly 30% hydrogen peroxide, may be the culprit. However, the exact mechanism by which the perodontium or cementum is damaged has not been elucidated.

Which of the following condylar positions is the most stable and should be used when fabricating an occlusal splint for a patient with temporomandibular joint dysfunction (TMD)? Vertical dimension of rest Maximum intercuspation Centric occlusion Centric relation

Centric relation The occlusal splint should be fabricated in centric relation (CR) because it is the most stable, reproducible position. CR describes the maxillomandibular relationship when the condyles articulate with the thinnest avascular portion of the articular disc so that the condyle is in its most anterior-superior position against the condylar eminence.

Which of the following tetracycline class drugs should only be taken once daily due to its long half-life?

Doxycycline Doxycycline has the longest half-life of the tetracycline group and is used both prophylactically and therapeutically. Doxycyline is used to prevent malaria and to treat Lyme disease, Rocky Mountain spotted fever, and acne. Taking doxycycline with dairy, antacids, calcium supplements, iron products, or laxatives containing magnesium may decrease the effectiveness of the drug.

All of the following are characteristics of Amelogenesis Imperfecta EXCEPT one. Which is the exception?

No treatment is usually needed Treatments for people with this condition usually include full crowns. Amelogenesis Imperfecta is inherited as an autosomal dominant trait in which it affects the formation of the enamel portion of the tooth. Both dentitions, permanent and primary may be affected. There are three types of this condition, which include hypo plastic, hypo calcified, and hypo maturation types Due to the fact that the enamel portion of the tooth is affected, usually teeth have abnormal colors that range from yellow, to brown, and even grey. Some teeth appear to be pitted and somewhat smaller than normal. People with this condition usually have a higher risk of developing caries. Radiographic findings usually reveal normal pulp and dentin while the enamel is reduced

If a casting contained 60% noble metal by weight with no gold content, which of the following classifications of metal alloys would it fall under?

Noble metal Noble metals require only 25% noble metal by weight and have no gold requirement. So the 60% noble metal by weight in the alloy in question is enough to make it a noble alloy even though there is no gold content. [High noble metal alloys require 60% by weight of noble metal content with at least 40% of that being gold.] [Base metals are classified as such when they have less than 25% noble metal by weight with no gold content requirement]

A practitioner has stabilized a permanent anterior tooth that has a horizontal fracture in the apical one third of the root. The tooth will most likely:

Remain functional and vital Generally, the more cervival the fracture, the more mobility and displacement of the coronal segment and a greater likelihood of pulp necrosis of this segment if not promptly repositioned.

Which of the following is responsible for mild pain to hot, cold, and sweet stimuli for days following the placement of an amalgam? Reversible pulpitis Pulpal necrosis Irreversible pulpitis Symptomatic apical periodontitis Asymptomatic apical periodontitis

Reversible pulpitis Placement of a restoration may irritate the pulp because of the desiccation of the tooth structure and mechanical trauma from the preparation for the restoration. A restoration that is slightly too high may also make the tooth sensitive. Reversible pulpitis following restoration placement causes general mild sensitivity that dissipates within a few days. The pain associated with reversible pulpitis caused by restoration placement is not spontaneous and is not stimulated by percussion or palpation.

All of the following is true regarding calcification and eruption of dentition

The sequence of eruption of primary teeth is central incisor, lateral incisor, first molar, canine, second molar A 6 month variation in time of eruption is considered normal Mandibular pairs precede maxillary pairs For eruption of permanent teeth, eruption to root completion is approximately 3 years Teeth typically erupt through the bone with 2/3rds root formation. Teeth also typically erupt through the gingiva with 3/4s root formation

Which of the following describes the most common reason ledges are formed during endodontic procedures? Too large a file, too fast Unachieved straight line access Improper irrigation Fallen pulp stone in canal

Unachieved straight line access Unnecessary deflection of the endodontic file may cause a loss of instrument control and subsequent ledge formation. Attempts to clean and shape the canal without proper straight line access often induce ledging, transportation, and zipping perforations. The goal of straight line access is to remove coronal tooth structure, to allow a straight canal for maximum freedom for the instrument in the more confined, difficult apical sections. Most endodontic cases are ledged, because the clinician is attempting to guide the instrument through too many curves.

When ingested, isopropyl alcohol is metabolized mostly into acetone. methanol. ethanol. acetaldehyde. formaldehyde.

acetone. Isopropyl alcohol is oxidized in the liver by the alcohol dehydrogenase into acetone and then acetate, formate, and finally CO2. Formation of acetone contributes to the central nervous system (CNS) depression experienced in isopropyl alcohol poisoning. 20-50% of absorbed isopropyl alcohol is excreted unchanged.

Renal tubular secretion is the primary method of excretion for bacitracin. benzylpenicillin. polymyxin. streptomycin. tetracycline.

benzylpenicillin. Benzylpenicillin (pen G) is the gold standard form of penicillin and is given parenterally due to its being unstable in stomach acid. Pen G exits the body primarily through tubular secretion. Because pen G is given parenterally, higher tissue concentrations of pen G can be achieved than is possible with pen V. Penicillin G is effective mainly against Gram-positive organisms, and some Gram-negative organisms, such as Neisseria gonorrhoeae and Neisseria meningitidis, are also reported to be susceptible to pen G.

Alpha 1 receptors

vasoconstriction of heart

Addison's disease

occurs when the adrenal glands do not produce enough of the hormones cortisol or aldosterone

The drug administration method that is characterized by the slowest absorption is intramuscular. intravenous. oral. sublingual. subcutaneous.

oral Oral administration has the slowest absorption rate, because it follows a long digestive path prior to being absorbed into the body. Drugs are immediately distributed when they are administered intravenously. The effect is rapid because the drug is placed directly into the bloodstream. The intramuscular route is administered into the voluntary muscle, which provides faster absorption than oral and subcutaneous routes. Subcutaneous administration is performed by inserting the drug under the skin, where it is absorbed in a slow manner, prolonging its action in the body.

Which artery descends on the posterior surface of the maxilla and supplies the maxillary sinus and the maxillary molar and premolar teeth? sphenopalatine artery greater palatine artery posterior superior alveolar artery infraorbital artery

posterior superior alveolar artery The external carotid artery supplies most of the head and neck, except for the brain (the brain gets its blood supply from the internal carotid and the vertebral arteries). The external carotid passes through the parotid salivary gland and terminates as the maxillary and superficial temporal arteries. The superficial temporal artery supplies the scalp. The maxillary artery leaves the infratemporal fossa by passing through the pterygomaxillary fissure into the pterygopalatine fossa. Here it splits up into branches that accompany the branches of the maxillary nerve. It supplies the muscles of mastication, the maxillary and mandibular teeth, the palate, and almost all of the nasal cavity. The mandibular teeth receive blood from the inferior alveolar artery, which is a branch of the maxillary artery. The maxillary teeth also receive blood from branches of the maxillary artery as follows: Posterior teeth: from the posterior superior alveolar artery. Anterior teeth: from the anterior and middle superior alveolar arteries. Remember: The venous return of both dental arches is the pterygoid plexus of veins. Branches of the maxillary artery that accompany the branches of the maxillary nerve: The posterior superior alveolar artery descends on the posterior surface of the maxilla and supplies the maxillary sinus and the maxillary molar and premolar teeth. The infraorbital artery enters the orbital cavity through the inferior orbital fissure. It ends by emerging on the face with the infraorbital nerve. The greater palatine artery descends through the greater palatine canal with the greater palatine nerve. It is distributed to the mucous membrane covering the oral surface of the hard palate. The pharyngeal branch passes backward to supply the mucous membrane of the roof of the nasopharynx. The sphenopalatine artery passes through the sphenopalatine foramen into the nasal cavity. It supplies the mucous membrane of the nasal cavity

The labiolingual dimension of maxillary canine pulp chamber is _______its mesiodistal dimension.

wider than The root canal morphology of the maxillary incisor and the maxillary canine is similar in several parameters. However, one of the primary differences is that the canine root canal system is wider labiolingually than mesiodistally, while for the maxillary central incisor, the pulp chamber is wider mesiodistally than labiolingually. A maxillary canine has no pulp horns, whereas, three pulp horns are observed in a newly erupted central incisor. The absence of pulp horns differentiates a maxillary canine from maxillary central incisor. The pulp chamber in both the maxillary central and the maxillary lateral incisors is similar, however, bigger in central incisors. An individual root canal in maxillary canines is broader labiopalatally than mesiodistally, but a gradual taper is observed from the middle third to the apical constriction In maxillary incisors, the access outline is shaped like a rounded triangle with its base towards the incisal edge The shape of access opening of maxillary lateral incisors is dependent on the prominence of the pulp horns and is similar to that of maxillary central incisors. However, it is usually more ovoid and smaller in size than the opening of central incisors For maxillary canines, the access opening though similar to maxillary lateral incisors, is oval or slot shaped in accordance with the pulp chamber anatomy.

Sensory innervation to the palate is supplied by _____. Motor innervation to tensor veli palatini is supplied by _____. A greater palatine block can anesthetize the palate as anterior as the maxillary _____. CN V2, CN V3, first premolar CN V3, CN V2, canine CN V2, CN V3, canine CN V3, CN V2, first premolar

• CN V2, CN V3, first premolar The greater palatine foramen is generally located halfway between the gingival margin and midline of the palate, approximately 5 mm anterior to the junction of the hard and soft palate (vibrating line) distal to the apex of the maxillary second molar. The hard palate is perforated by the following foramina: The incisive foramen, posterior to the maxillary incisors, transmits the nasopalatine nerves and the terminal branches of the sphenopalatine artery The greater palatine foramen, is most frequently located distal to the maxillary second molar, transmits the greater palatine vessels and nerve The lesser palatine foramen, just posterior to the greater palatine foramen, transmits the lesser palatine vessels and nerve Nerves of the palate: Sensory Innervation to the palate: is supplied by the maxillary (CN V2) nerve. The anterior part of the hard palate is supplied by the nasopalatine nerve, which passes through the incisive foramen. The posterior part of the hard palate is supplied by the greater palatine nerve which passes through the greater palatine foramen. The soft palate is supplied by the lesser palatine nerve which passes through the lesser palatine foramen. Motor Innervation: the tensor veli palatini is innervated by a muscular branch from the mandibular division of the trigeminal nerve (CN V). All other muscles are innervated by the pharyngeal plexus (motor portion from the vagus nerve and cranial part of the accessory nerve). The greater palatine block or GP block is useful for dental procedures involving palatal soft tissues distal to the maxillary canine. This maxillary block anesthetizes the posterior portion of the hard palate, anteriorly as far as the maxillary first premolar and medially to the midline.Target area: the greater (anterior) palatine nerve as it passes anteriorly between the soft tissues and bone of the hard palate. The nasopalatine nerve block anesthetizes the anterior portion of the hard palate (soft and hard tissues) from the mesial aspect of the right first premolar to the mesial aspect of the left first premolar. Target area: incisive foramen, beneath the incisive papilla.

Hypercortisolemia can lead to ______, whereas hypocortisolemia can lead to______. Cushing syndrome, Addison disease Cushing syndrome, ectopic ACTH syndrome Addison disease, Cushing syndrome Addison disease, ectopic ACTH syndrome

• Cushing syndrome, Addison disease Cushing syndrome is a hormonal disorder caused by prolonged exposure of the body's tissues to high levels of the hormone cortisol. This results in characteristic changes in body hiatus, including moon facies, truncal obesity, muscular wasting, and hirsutism. Sometimes called "hypercortisolism," it is relatively rare and most commonly affects adults aged 20 to 50. The female-to-male incidence ratio is approximately 5:1. Patients with Cushing syndrome are often hypertensive because of fluid retention. Long-term glucocorticoid excess can result in decreased collagen production, a tendency to bruise easily, poor wound healing, and osteoporosis. They are often at an increased risk for infection. Laboratory studies may reveal increased blood glucose levels because of interference with carbohydrate metabolism, and examination of the peripheral blood smear may demonstrate slight decrease in eosinophil and lymphocyte counts. Important: The patient's cardiovascular status must be evaluated and treated if necessary prior to surgery. Note: The most common cause of Cushing syndrome is a tumor in the pituitary gland. Remember: Addison disease occurs when the adrenal glands do not produce enough of the hormone cortisol and in some cases, the hormone aldosterone. For this reason, the disease is sometimes called chronic adrenal insufficiency, or hypocortisolism.

All of the following muscle/nerve combinations are correct EXCEPT one. Which one is the EXCEPTION? Trapezius muscle / accessory nerve Stylopharyngeus muscle / glossopharyngeal nerve Sternocleidomastoid muscle / vagus nerve Cricothyroid muscle / superior laryngeal nerve

• Sternocleidomastoid muscle / vagus nerve

Which of the following are NOT appropriate care recommendations for an avulsed tooth?

-Place the tooth in water and come into the office immediately Place the tooth in alcohol and come immediately These are not recommended because the osmolarity and composition would damage the cells on the root surface It is important to use transport mediums such as milk, saliva, Hanks solution.

A 45 year old female patient presents to your office with malocclusion. What considerations are required for adult orthodontic treatment?

-Treatment goals may not include ideal collusion and facial appearance (because adults have already completed their physical growth and development) -Coordination of all dental care will be required due to the likelihood of existing restorations and periodontal concerns. -Particular sensitivity may be required to create esthetically pleasing appliances (Orthodontic appliances are typically found in adolescents, so there is a stigma in regards to treatment) -Adult patients are more interested in their treatment than their adolescent counterparts

Your patient suffers from occasional angina and has mild to moderate hypertension. What should be the maximum dose of epinephrine administered within a 30 minute period?

0.04mg Care should be taken to avoid excessive epinephrine administration by using proper injection techniques

What is the appropriate risk of infection after a percutaneous exposure from a patient infected with HIV?

0.3% The appropriate risk of infection following percutaneous exposure from an HIV infected patient is 0.3%. Though considered one of the worst infectious diseases to contract, the risk of transmission is low compared to Hepatitis B (30% percutaneous) and Hepatitis C (1.8%, needle stick)

Which blood alcohol content is closest to the median lethal dose (LD50) of ethanol? 0.5% 0.3% 0.2% 0.1% 0.08%

0.5% The median lethal dose (LD50) is the dose of a substance that would kill half of the experimental population.LD50 is reached when the alcohol concentration in the bloodstream reaches 0.4-0.5%.LD50 is often used to measure the level of toxicity.Variation in LD50 may be influenced by genetic factors, environmental factors, and the mode of administration.

The ideal apical termination point for appropriate cleaning and shaping of a pulpit tooth is considered to be:

0.5-1mm short of the radiographic index A vital should be instrumented to within 2mm of the radiographic apex. Cleaning and shaping should not be done past the apical foramen because extruding more affected material past the apex may create greater inflammatory products

The optimal concentration of fluoride in drinking water should be ____

0.7 ppm At this level the risk of skeletal fluorosis is minimal

Which is the optimal range of drinking water fluoridation as defined by the United States Centers for Disease Control and Prevention (CDC) guidelines? 1.9-2.4 ppm 0.3-1.8 ppm 0.7-1.2 ppm 0.1-0.6 ppm

0.7-1.2 ppm United States Centers for Disease Control and Prevention (CDC) guidelines indicate that the optimal range of fluoride in drinking water should be 0.7 to 1.2 ppm.Water fluoridation is the most economical and the most effective technique in delivering fluoride to the people of the community.The amount of fluoride allowed in water varies widely from place to place depending on the weather conditions and the amount of water intake of people in the community.

A patient presents to the office with recurrent herpes labialis. For most patients with this condition, their primary herpetic gingivostomatitis episode most likely occurred during which age range? 21-25 years 16-20 years 11-15 years 6-10 years 1-5 years

1-5 years Herpetic gingivostomatitis is an acute viral disease caused by the herpes simplex virus. Primary herpetic gingivostomatitis is an infection transmitted through direct contact or through aerosol droplets. It clinically appears as yellow fluid-filled vesicles that leave ulcers when the vesicles rupture. Vesicles commonly appear on the gingiva, oral mucosa, and vermilion border and usually affect children five years old or younger. Secondary herpetic gingivostomatitis is a reactivation of the latent virus due to trauma, stress, or immunosuppression. The vesicles may be found intraorally or extraorally on keratinized surfaces. Primary herpetic gingivostomatitis typically occurs in children between six months and five years of age, but it can occur in older children and adolescents [1]. (See "Epidemiology, clinical manifestations, and diagnosis of herpes simplex virus type 1 infection".)

Which of the following represents the median in the following data set? 1, 1, 2, 2, 3, 7, 9 (n = 7; sum = 25) 20 15 7 3 2 1

2 Median denotes the middle-most number in a number series; there are seven numbers, and 2 is the fourth number. Mean measures the average number or score among the numbers included in the data set. Mode denotes the most prominent number or the number that appeared the most in the number list.

Which of the following describes how thick the first increment should be for a class II restoration to ensure proper curing of the composite and the lowest strain on the tooth structure due to polymerization shrinkage? (Not counting snowplow or sandwich techniques) 1.0 mm 2.0 mm 2.5 mm 0.5 mm <0.5 mm

1.0 mm The first increment in a class II restoration must be 1-mm thick if the snowplow technique is not used. The first increment is placed on the gingival floor of the restoration.The snowplow technique employs the use of flowable composites in conjunction with the posterior composite restorations.An initial thin layer of flowable composite is placed on the gingival or pulpal floor and left uncured at this stage.An initial increment of heavily filled composite resin is placed or pushed over the unset flowable composite, displacing most of it into the areas where heavily filled composite cannot penetrate. Its advantages are: Decreased voids in the restoration Decreased gingival marginal leakage 0 CommentsSort by

What is the appropriate risk of infection after a needle stick from a patient infected with the hepatitis C virus?

1.8% Due to the fact that there is no vaccine for HCV, prevention is critical

Which of the following is the minimum number of flutes needed on a carbide bur for it to be considered a "finishing bur"? Finishing carbide burs do not have flutes. 24 6 18 12

12 The minimum number of flutes needed on a carbide bur to be considered a finishing bur is 12. Carbide burs are used most commonly for excavating and preparing cavities, finishing cavity walls, finishing restoration surfaces, drilling old fillings, finishing crown preparations, contouring bone, removing impacted teeth, and separating crowns and bridges. Carbide burs are made of tungsten carbide, a metal that is extremely hard and can withstand high temperatures. Because of their hardness, carbide burs can maintain a sharp cutting edge and be used many times without becoming dull. Carbide burs are brittle and have a tendency to fracture under pressure. Because of their brittleness, they are best operated at high speeds with light pressure. Trimming and finishing burs have more blades than operative burs, and the blades are closer together and shallower, making them ideal for fine finishing and polishing.

Please match the following cranial nerves with their appropriate preganglionic parasympathetic fibers. Remember, postganglionic fibers from these nerves are carried via the opthalmic nerve (V1), maxillary nerve (V2), and mandibular nerve (V3). 3. CN IX Glossopharyngeal 2. CN VII Facial 1. CN III Oculomotor A. Parotid Gland C. Lacrimal, Submandibular, Sublingual Gland B. Pupil Constriction, Accomodation

1:B, 2:C, 3:A The facial nerve leaves the cranial cavity by passing through the internal acoustic meatus, which leads to the facial canal inside the temporal bone. Finally, the nerve exits the skull by way of the stylomastoid foramen of the temporal bone. Note: If you cut the facial nerve just after its exit from the stylomastoid foramen, it would cause a loss of innervation to the muscles of facial expression. The facial nerve carries an efferent component for the muscles of facial expression and for the preganglionic parasympathetic innervation of the lacrimal gland (relaying in the pterygopalatine ganglion) and submandibular and sublingual glands (relaying in the submandibular ganglion). The afferent component serves a tiny patch of skin behind the ear, taste sensation, and the body of the tongue. Clinical information: Bell palsy: involves unilateral facial paralysis with no known cause, except that there is a loss of excitability of the involved facial nerve. The onset of this paralysis is abrupt, and most symptoms reach their peak in 2 days. One theory of its cause is that the facial nerve becomes inflamed within the temporal bone, possibly with a viral etiology. Trigeminal neuralgia (tic douloureux): also has no known cause but involves the afferent nerves of the trigeminal nerve. It usually involves the maxillary or mandibular nerve branches but not the ophthalmic branch. One theory is that this lesion is caused by pressure on the sensory root of the trigeminal ganglion by area blood vessels. Clinically, the patient feels excruciating short-term pain (tic) when facial trigger zones are touched or when speaking or masticating, setting off associated brief muscle spasms in the area. The right side of the face is affected more commonly than the left. It is more common in females. Carbamazepine (Tegretol®) is still the mainstay of treatment.

Which of the following represents the median of the data set? 1, 1, 1, 1, 2, 2, 2, 3, 3, 7, 9 (n = 11 and sum = 32) 1 2 30 7 3

2 The median of a number series denotes the middle-most number in the series. The mean of a number series measures the average number or score among the numbers included in the data set. The mode of a number series denotes the most prominent number, or the number that appears most frequently in the series.

Which of the following doses of epinephrine is considered the maximum safe dose that should be administered to a patient with a history of significant cardiovascular disease?

2 mL 1:50,000 While administering epinephrine to patients who had or currently have cardiovascular disorders like cardiac arrhythmias or coronary artery disease, extreme caution must be observed. Administration of epinephrine carries the risk of precipitation of angina pectoris in patients with ischemic heart disease or coronary insufficiency. Patients with a history of cardiovascular problems should not exceed 0.04 mg of epinephrine in a single visit. A 1:1,000 dosage is 1 mg/mL, so a 1:100,000 dosage is 0.01 mg/mL. When using a 1:50,000 concentration of epinephrine, the dosage should not exceed 2 mL for cardiac patients. A 1.8 mL carpule with 1:100,000 epinephrine yields 0.018 mg of epinephrine, so two carpules may be given safely, because 0.036 mg is lower than the 0.04 mg limit. Dosagemg/mL1:1,0001 mg/mL1:50,0000.02 mg/mL1:100,0000.01 mg/mL1:200,0000.005 mg/mL

Which of the following describes the maximum amount of resin composite that can be properly cured per layer of a restoration? 4 mm 0.5 mm 2 mm 1 mm Depth does not matter

2 mm The maximum depth of resin that can be properly cured per layer is approximately 2 mm.Light-cured composites are currently among the most popular restorative materials because of their ease of handling.The success of composite-resin restorations depends on the access of high-intensity curing light to the material.If the composite thickness exceeds 2 mm, inadequate curing occurs. This phenomenon is especially seen with darker shade composites. Filler particles and coloring agents tend to scatter or absorb the curing light within 1-2 mm of the material.

The patient is concerned about the post-infection rate. According to Powell, what is the expected rate of infection after periodontal surgical techniques are performed? 1% 2% 3% 4%

2% Following the completion of the surgical procedure, the patient is advised to avoid any trauma or tooth brushing in the treated area for 2 weeks. 0.12% chlorhexidine rinses are usually prescribed. The most common complication of the esthetic crown lengthening procedure is recession, if excessive bone resection has been done. After the ostectomy and osteoplasty procedures, the patient can present various complications, similar with a traditional osseous resective procedure. Poorly controlled diabetes mellitus and smoking have been reported to predispose patients to periodontal disease and increased risk for an adverse response to treatment. According to Powell the prevalence of postoperative infections after various periodontal procedures was 2% (22 cases out of 1052 surgical procedures). The highest rate was not for the osseous resective procedures, but for the free gingival graft procedure, followed by subepithelial connective tissue graft and implant placement.

Often alcohol hand hygiene is used in dental facilities. Since alcohol is also flammable, what is the maximum volume that one vessel of such a liquid may contain a dental operatory?

2.0 L The American Society for Healthcare Engineering has made recommendations regarding the use of alcohol based hand rub in dental practice. The maximum volume of a container of such liquid in an operator is 2.0L. [If the liquid is in the hallway, it should not exceed 1.2 L.

What INR level is safest to proceed with treatment?

2.5 The therapeutic range for a patient taking warfarin is an INR of 2.0-3.0. Generally, the INR should be below 3.0 for any extensive surgical procedure where bleeding is expected. A full mouth extraction would indicate significant bleeding.

What percentage is the cut off to differentiate between localized and generalized categories in periodontal diagnosis?

30% During the diagnosis of periodontal disease, a clinician will need to diagnose the extent of disease. Localized periodontal disease is when there is less than 30% involvement whilst generalized periodontal disease is when there is more than 30% involvement with periodontal disease

Which human herpes virus number is associated with Kaposi sarcoma?

8 Kaposi sarcoma is a malignant proliferation of endothelial cells. It is most commonly seen as a complication of AIDS, and its incidence has been markedly reduced by new antiretroviral therapies. It may be seen as an endemic African type or classic Mediterranean type Kaposi's sarcoma (KS) is a type of cancer that can form masses in the skin, in lymph nodes, in the mouth, or in other organs.[4][6] The skin lesions are usually painless, purple and may be flat or raised.[6][8] Lesions can occur singly, multiply in a limited area, or may be widespread.[6] Depending on the sub-type of disease and level of immune suppression, KS may worsen either gradually or quickly.[6] KS is caused by a combination of immune suppression (such as due to HIV/AIDS) and infection by Human herpesvirus 8 (HHV8 - also called KS-associated herpesvirus (KSHV)).[8]

The recommended vertical angulation when making bitewing radiographs of the posterior teeth may vary from 10 degrees to -5 degrees. -5 degrees to 10 degrees. 5 degrees to 10 degrees. 0 degrees to 5 degrees.

5 degrees to 10 degrees. The recommended vertical angulation for bitewing radiographs usually ranges from 5 to 10 degrees because the upper and lower portion of the film contacts the palate and the lingual of lower posterior teeth at a different angle.When taking bitewing radiographs, the upper part of the film is usually positioned at an angle of 20 degrees while it contacts the palate and the lower part of the film is upright.Because of this angulation, the x-ray tube must be angulated to approximately 5 to 10 degrees vertically to compensate for the film position and to achieve accurate bitewing records. 0 CommentsSort by

The recommended pharmacologic treatment for xerostomia is 10 mg phenytoin. 5 mg pilocarpine. 5 mg levonordefrin. 1 mg malathion.

5 mg pilocarpine. Pilocarpine is a parasympathomimetic alkaloid considered a nonselective muscarinic receptor agonist. It stimulates the secretion of large amounts of saliva and sweat through muscarinic receptor stimulation. Pilocarpine has been used in the treatment of chronic open-angle glaucoma.

Which is the maximum distance necessary between the crest of the bone and the two restoration contact points to achieve 100% complete interproximal papilla fill? 3 mm 4 mm 5 mm 6 mm

5 mm According to Tarnow et al. (1992), almost 100% of the cases present with a complete papilla fill when the contact point is located 5 mm and less away from the crest of bone. This study was designed to determine whether the distance from the base of the contact area to the crest of bone could be correlated with the presence or absence of the interproximal papilla in humans. If a space was visible apical to the contact point, then the papilla was deemed missing; if tissue filled the embrasure space, the papilla was considered to be present. The results showed that when the measurement from the contact point to the crest of bone was: 5 mm or less, the papilla was present almost 100% of the time. 6 mm, the papilla was present 56% of the time. 7 mm or more, the papilla was present 27% of the time or less.

Which of the following describes the dosage necessary for subacute bacterial endocarditis (SBE) prophylaxis using clindamycin in an adult patient? 2 g one hour prior to the procedure 600 mg one hour prior to the procedure 500 mg one hour prior to the procedure 300 mg one hour prior to the procedure 250 mg one hour prior to the procedure

600 mg one hour prior to the procedure Clindamycin 600 mg one hour before the procedure can be prescribed when the patient is allergic to penicillins. Subacute bacterial endocarditis (SBE) prophylaxis requires one of the following: Oral amoxicillin 2 g (50 mg/kg) If unable to take oral medication: Ampicillin 2 g intramuscularly (IM) or intravenously (IV) (50 mg/kg IM or IV) Cefazolin or ceftriaxone 1 g IM or IV (50 mg/kg IM or IV) If allergic to penicillins or ampicillin and able to take an oral regimen: Clindamycin 600 mg (20 mg/kg) Azithromycin 500 mg (15 mg/kg) Clarithromycin 500 mg (15 mg/kg) If allergic to penicillins or ampicillin and unable to take oral medication: Cefazolin or ceftriaxone 1 g IM or IV (50 mg/kg IM or IV) Clindamycin 600 mg IM or IV (20 mg/kg IM or IV)

Which of the following is the percentage of liquid Nitrous Oxide that is in an "E" cylinder when showing that it is 50% empty? <1% 50% 100% 95% 25%

<1% Nitrous oxide is stored as a compressed gas and liquid in commercial cylinders. The pressure regulator measures the gas pressure of the cylinder. Nitroux oxide gas that is released from the cylinder is replenished by the evaporation of liquid nitrous oxide, maintaining a nearly constant internal pressure. Once the liquid nitrous oxide is exhausted, the reading on the pressure regulator will begin to fall as the gas is no longer being replenished by evaporating liquid nitrous oxide. Compressed gas cylinders come in a variety of sizes, and each use and size is designated by a series of numbers and letters. The number on the side of the tank denotes the number of liters of gas the cylinder holds. The letters A, B, C, D, and E designate the size of the cylinder. The "A" cylinder is the smallest cylinder, with 34 L of compressed gas; an "E" cylinder is the largest (680 L). When the regulator mark 50% of gas , that means that the liquid is almost done

Features of the compound odontoma include:

A collection of tooth like structures Compound odontomas are collections of multiple, tooth like structures. Odontomas are developmental anomalies. They are the most common. Odontogenic tumors classified into two types: compound and complex. They often have a loose connective tissue matrix surrounding them and may have mature enamel caps as well. They are often named toothless. An odontoma, also known as an odontome, is a benign tumour[4] linked to tooth development.[5] Specifically, it is a dental hamartoma, meaning that it is composed of normal dental tissue that has grown in an irregular way. It includes both odontogenic hard and soft tissues.[1] As with normal tooth development, odontomas stop growing once mature which makes them benign.[6] The average age of people found with an odontoma is 14.[7] The condition is frequently associated with one or more unerupted teeth and is often detected through failure of teeth to erupt at the expected time. Though most cases are found impacted within the jaw there are instances where odontomas have erupted into the oral cavity

In the operating room, the dentist proceeds to place a block using a local anesthetic. The patient reports a metallic taste and dizziness. The patient becomes unresponsive, and rhythmic tonic-clonic movements are seen. She experiences bladder incontinenece. Which is correct?

A decrease in protein binding would increase the likelihood of CNS toxicity. Adminstering a benzodiazepine decreases the likelihood of seizures Using lidocaine instead of ropivacaine (epidural shots) would have decreased her likelihood of CNS toxicity from local anesthetics She may progress to generalized CNS depression and coma CNS toxicity from local anesthetics is exceedingly rare

Which of the following statements is true regarding class IV composite restorations? No answers apply. The use of retentive pins for support on class IV restorations is recommended. A final veneer layer of micro-filled resin will help create a smooth, glossy surface. Tooth fragments should never be reattached to the remaining tooth structure.

A final veneer layer of micro-filled resin will help create a smooth, glossy surface A final veneer layer of micro-filled resin will help create a smooth and glossy surface due to its small particles. Polished surfaces are less receptive to plaque accumulation and extrinsic staining. Retentive pins are never recommended for support and retention in class IV restorations.

In order to remove the calculus present on this patient, the clinician needs to utilize a scaling stroke with the instrument of choice. Which of the following is the best description of a scaling stroke? A firm pull stroke with adequate lateral pressure applied in a coronal direction at the deposit A firm push stroke with adequate lateral pressure applied in a vertical direction at the deposit A light push stroke with light lateral pressure applied in a vertical direction at the deposit A light horizontal pull stroke with light lateral pressure applied in an oblique direction at the deposit

A firm pull stroke with adequate lateral pressure applied in a coronal direction at the deposit Basic concepts of instrumentation include the following: Grasp: utilizing a modified pen grasp Activation: using forearm and wrist in one motion Adaptation: rolling instrument to keep toe adapted to tooth surface Insertion: closed blade at 0 degrees angulation Exploratory stroke: light lateral pressure Scaling stroke: firm pull stroke with adequate lateral pressure applied in a coronal direction at the deposit

What is full coverage restoration? The main purpose of a dental crown is to reinstate the structural integrity of a tooth. Also known as a full coverage restoration, a crown can strengthen your tooth, improve your smile and give you the natural function you need to chew, bite and eat your favorite foods without any pain.

A full coverage restoration is also known as a dental crown and requires extensive removal of tooth structure A full coverage restoration has a greater retention and resistance form than a partial coverage restoration The partial coverage restoration is fabricated to replace the destructed tooth structure so that the natural tooth structure is preserved.

Which of the following are related to vital teeth and usually do not warrant endodontic therapy?

A radicular cyst usually occurs in a preexisting granuloma. Seldom is painful. Radiolucency at apex of nonvital tooth. - A cementoma occurs most frequently in the anterior region of the mandible. It starts as a radiolucent lesion and then calcifies. The cementoma does not affect pulp vitality. Also called periapical cemental dysplasia. - A traumatic bone cyst is not a true cyst since there is no epithelial lining. Found mostly in young people; asymptomatic. Radiolucency that appears to scallop around the roots of teeth. Teeth are usually vital. - A lateral periodontal cyst occurs on a lateral periodontal location and it is of developmental origin arising from cystic degeneration of clear cells of the dental lamina. Tooth is vital. - An ameloblastoma is a benign, locally aggressive tumor arising from the odontogenic ectoderm. Lesions occur as multilocular radiolucencies and frequently cause extensive root resorption. The mandible is affected four times more frequently than the maxilla. - A cementoblastoma is an odontogenic tumor characterized by the proliferation of functional cementoblasts that form a large mass of cementum or cementum-like tissue on the tooth root.

Hospitalization is the most recommended option to treat which of the following patients? A child with type 1 diabetes requiring extraction of a tooth with an associated chronic apical abscess A severely developmentally-disabled patient requiring a full mouth gingivectomy to treat medication-induced gingival hyperplasia A patient being treated with anticoagulants with a prothrombin time 3.5 times that of the control time, requiring a single tooth extraction A patient with a prosthetic heart valve requiring a three-tooth extraction and alveoloplasty

A severely developmentally-disabled patient requiring a full mouth gingivectomy to treat medication-induced gingival hyperplasia Hospitalization is mandatory for the treatment of a severely developmentally-disabled patient requiring a full mouth gingivectomy to treat medication-induced gingival hyperplasia. A full mouth gingivectomy is a serious periodontal surgical procedure requiring complete patient compliance and support. In a disabled patient, general anesthesia administration should be carried out for the procedure, which requires a hospital set up and emergency equipment on hand. In a patient with type 1 diabetes mellitus, single tooth extraction can be carried out easily under local anesthesia. A patient on anticoagulants can be managed with a discussion of the patient's international normalized ratio (INR) levels with their physician and treating them accordingly. A patient with a prosthetic heart valve can be managed through antibiotic prophylaxis before the procedure.

Oral manifestations of geriatric patients would include which of the following? Abrasion and attrition Increased pulp chamber size Decreased incidence of oral cancer Increased tooth sensitivity

Abrasion and attrition Oral manifestations of geriatric patients affect the soft tissues and hard tissues. Soft tissues feature the following: Dry, purse-string lips with angular cheilitis Capillary fragility Hyperkeratosis of oral mucosa Fissures Sublingual varicosities Reduced taste from the loss of taste buds Hard tissues feature the following: Abrasion Attrition Dark color of teeth Root caries Decreased tooth sensitivity Decreased pulp chamber size

Which of the following rates principally determines the action of onset of a drug administered orally? Temperature Excretion Distribution Biotransformation Absorption

Absorption Absorption is the major factor determining the onset of action of orally administered drugs. Intravenous drugs demonstrate rapid onset of action due to high absorption, and oral drugs have a delayed onset of action due to their first pass metabolism. Onset of action is the time period a drug takes to show its pharmacologic effect in the body. The onset of action of drugs varies depending upon the routes of administration in the body. Different routes of administration provide different absorption rates.

Which of the following medications inhibit α-glucosidase and reduce glucose absorption in the gastrointestinal (GI) tract? Pioglitazone Insulin Acarbose Metformin Glyburide

Acarbose Acarbose inhibits α-glucosidase, an intestinal enzyme found in the small intestine that breaks down starch (polysaccharides) into glucose (monosaccharides). This reduces the glucose absorbed by the GI tract. If a patient is taking acarbose and becomes hypoglycemic, the patient must eat something with monosaccharides, such as glucose tablets or gel. Fruit juice or starchy foods do not effectively reverse the hypoglycemic state. Hepatitis may develop from acarbose use; liver enzymes should be checked before and during use.

Which of the following conditions is most likely to develop into a malignant lesion? Actinic cheilitis Lichen planus White sponge nevus Hyperkeratosis

Actinic cheilitis Actinic cheilitis (AC) (actinic cheilosis, actinic keratosis of lip, solar cheilosis, sailor lip, or farmer lip) describes a burn or inflammatory condition of the lip caused by long-term sunlight exposure and is considered a variant of actinic keratosis of the lip. AC is documented to possess the greatest malignant potential among the lesions listed. AC is prone to progress into squamous cell carcinoma and is characterized by a thickening whitish discoloration of the lips at the border of the lip and skin and loss of vermillion border. AC most commonly affects the lower lip because the lower lip is typically more exposed to the sun. Because of this, the commissures (corners of the mouth) are not usually involved. Affected individuals may experience symptoms such as a dry sensation and cracking of the lips that is usually painless and persistent. The appearance of AC is variable. White lesions indicate hyperkeratosis. Red, erosive, or ulcerative lesions indicate atrophy, loss of epithelium, and inflammation. Hyperkeratosis is associated with the thickening of the outermost layer of the epidermis and presence of an abnormal quantity of keratin. Lichen planus is an autoimmune disease of skin and mucous membranes. White sponge nevus involves a defect in the normal process of keratinization of the mucosa.

Which of the following analgesics is indicated for mild dental pain for a pregnant patient?

Acetaminophen Acetaminophen is indicated for mild to moderate pain in pregnant women. Ibuprofen is contraindicated early in the pregnancy (1-13 weeks) because it may increase the likelihood of miscarriage. It is also contraindicated in the third trimester because it may increase the child's risk of heart problems. Aspirin is contraindicated for pregnant women because it may induce premature closure of the fetal ductus arteriosus. Ketamine is contraindicated because it may cause hallucinations in the mother. Hydrocodone is contraindicated because it is a pregnancy class C drug. 0 CommentsSort by Studies have found that the main risk factor for Reye's syndrome is taking aspirin or other related drugs, called salicylates.

Which of the following provides the LEAST anti-inflammatory effect?

Acetaminophen Acetaminophen, also known as Paracetamol®, is an analgesic drug that also has antipyretic property. It is popularly used in relieving headaches, fever, and mild pains. It also has an anti-inflammatory property but is weaker when compared with NSAIDs. The antipyretic property of acetaminophen makes it very useful in controlling fever. Most flu and cold medications incorporate acetaminophen in their formulation for this reason. Acetaminophen is also combined with opioid analgesics in managing postsurgical and severe types of pain. Aspirin and ibuprofen are among the NSAIDs, and cortisol is a steroid hormone that is released by the adrenal cortex. They all provide significant anti-inflammatory effects.

Each of the following is classified as a nonsteroidal anti-inflammatory drug (NSAID) EXCEPT one. Which one is the EXCEPTION? Acetaminophen Acetylsalicylic acid Diflunisal Ketorolac Ibuprofen

Acetaminophen All of the medications listed are analgesics, but acetaminophen (Tylenol) does not demonstrate anti-inflammatory effects. Non-steroidal anti-inflammatory drugs (NSAIDs) demonstrate anti-inflammatory properties. Acetylsalicylic acid, ibuprofen, ketorolac, and diflunisal are all considered NSAIDs. It is recommended that patients keep acetaminophen usage below 3,000 mg/day to prevent liver damage. Acetylsalicylic acid (aspirin) may cause gastrointestinal bleeding.

What substance alters the cell membrane permeability and nueromuscular junction once the threshold stimulus has been achieved? Acetylcholine Acetylsalicylic acid Cholinesterase Hydroxycholine

Acetylcholine The cell membrane becomes more permeable with the release of acetylcholine. Acetylcholine is a chemical neurotransmitter that works for both the central nervous system and the peripheral nervous system of the human body. Acetylcholine attaches to the acetylcholine receptors and initiates the opening of sodium (Na) channels within the cell membrane.

Which of the following analgesics is associated with Reye syndrome?

Acetylsalicylic acid Reye syndrome is a potentially fatal syndrome featuring the following: Rashes Vomiting Brain damage Liver damage The exact cause is unknown, and though it has been associated with acetylsalicylic acid (aspirin) consumption by children with viral illness, it also has occurred in the absence of aspirin use.

This patient's gingival condition stems from a lack of oral hygiene. Which of the following is the correct sequence for the stages of dental plaque development? Matrix forms, bacteria adhere to the tooth surface, and acquired pellicle forms. Bacteria multiply and colonize, bacteria adhere to the tooth surface, and acquired pellicle forms. Bacteria adhere to the tooth surface, acquired pellicle forms, and bacteria multiply and colonize. Acquired pellicle forms, bacteria adhere to the tooth surface, and bacteria multiply and colonize.

Acquired pellicle forms, bacteria adhere to the tooth surface, and bacteria multiply and colonize. The stages of plaque formation are as follows: Acquired pellicle forms. Bacterial plaque biofilm adheres to the tooth surface. Bacteria multiply. Complex microbial community forms.

A 17 year old with unprepared congenital cyanotic heart disease must visit the dentist to remove two impacted lower third molars in the near future. Which of the following should be done with regards to this patient prior to their dental procedure?

Adminster prophylactic amoxicillin Oral amoxicillin is given to patients who need prophylaxis for a dental procedure if they are able to take oral medications and are not allergic to penicillins Prophylaxis is recommended in this patient because they have a congenital heart defect that is considered cyanotic which hasn't been repaired

Which of the following species is Gram-positive, facultatively anaerobic, rod-shaped, filamentous, and most commonly associated with root surface caries? Streptococcus gordonii Streptococcus salivarius Streptococcus mutans Porphyromonas gingivalis Actinomyces viscosus

Actinomyces viscosus Actinomyces viscosus is commonly associated with acid production and root caries.Porphyromonas gingivalis is considered an etiologic pathogen of periodontitis.Streptococcus gordonii is one of the early colonizers involved in plaque formation.Streptococcus mutans is highly associated with coronal caries.Streptococcus salivarius is the most common oral bacterium and resides mostly on the tongue.

Each of the following is a mechanism of action for oral diabetes drugs EXCEPT one. Which is the EXCEPTION?

Acts as an insulin analogue and binds to insulin receptors Oral diabetic drugs do not act as insulin analogues and do not bind to insulin receptors. Sulfonylureas and meglitinides stimulate insulin release from B-cells in the pancreas. Exenatide, liraglutide, and pramlintide block glucagon release from the pancreas. Metformin acts to increase tissue glucose uptake.

A 14 year old female presented with a nontender gingival swelling located in the anterior maxilla. The denies any pain associated with the swelling. However, after examination and radiographic films taken, there was a radiolucent lesion with small opaque foci located around the maxillary canine that was impacted. A biopsy was performed and results showed rosettes, (Rosettes are round assemblage of cells found in tumors), like structures of columnar epithelial cells with a periodic acid Schiff positive material observed. Which of the following is the most likely diagnosis for this patient's lesion?

Adenamatoid odontogenic tumor that is derived from the dental lamina or the enamel organ. It is usually seen more commonly in younger people between the ages of five to thirty and has a predilection for females. These lesions are more commonly seen in the anterior maxilla and is associated with the crowns of impacted teeth; most commonly the maxillary canine. Radiographically, the lesion presents as a radiolucent lesion that have small opaque foci that reflects the calcification. These tumors are asymptomatic and have a nontender gingival swelling associated with it. Histologically, it has columanar epithelial cells that are rosettes and ductile structures, Treatment is usually enucleation due to its encapsulated lesion that does not have recurrence.

Marginal or partial resection (removal) should be considered the primary treatment modality for all of the following lesions EXCEPT one. Which is the EXCEPTION?

Adenomatoid odontogenic tumor Adenamatoid odontogenic tumors behave in a completely benign fashion. Conservative surgical excision or enucleation is usually curative. The recurrence rate after enucleation is rare [The appropriate treatment of ameloblastoma is marginal or an en bloc resection, with a 1.5cm border of normal tissue beyond the radiographic margin of the lesion due to the high recurrence rates of ameloblastoma]

A 22 year old patient with an unprepared cyanotic congenital heart defect is scheduled for a routine teeth cleaning. The patient is allergic to penicillins. Which of the following should be done with regards to this patient prior to their dental procedure?

Adminster oral prophylactic clindamycin clindamycin is not a penicillin antibiotic. Instead, it belongs to a class of medications known as lincomycin antibiotics, or lincosamides. Clindamycin is an alternative to the penicillins and cephalosporins for the treatment of skin and soft tissue infections. These types of patients are typically given amoxicillin if not allergic to penicillins. However, that is not the case in this situation

Which of the following factors is the most likely reason for a midline fracture of a denture that has been used for many years? Inadequate extension of the posterior palatal seal Advanced resorption of the alveolar ridge Over-relief of the incisive papilla area Porosity of the denture base material acquired during processing

Advanced resorption of the alveolar ridge Advanced resorption of the alveolar ridge causes the dentures to rock or move in a seesaw motion using the palatal area of the denture as a fulcrum.As the dentures move back and forth, the force of mastication creates stress along the midline of the denture.Through time, the stress created along the midline of the ill-fitting denture causes it to crack then fracture completely.

When determining the optimal concentration for community water fluoridation, which of the following factors is most important? Decayed/missing/filled teeth (DMFT) score of the community Adult-to-child ratio of the community Weight of the average community member Air temperature

Air temperature The optimal fluoride concentration for a community water source is typically based on the air temperature of the region.People living in areas that experience high temperatures should have lower fluoride levels in their community water because the majority would drink copious amounts of water.Communities with cold air temperatures should have higher levels of fluoride in community water because people drink less in this type of weather. Update: since 2015, the department of Health and Human Services along with the Americans with Disabilities Act have modified the guidelines to recommend a uniform 0.7 ppm for all communities, regardless of temperature.

Each of the following is a consequence (positive or negative) of dentinal smear layer removal EXCEPT one. Which is the EXCEPTION? All answer choices are consequences of dentinal smear layer removal Makes microleakage potentially more significant as a barrier to the pulp is removed. Increases dentin wetness Increases bond strength

All answer choices are consequences of dentinal smear layer removal The smear layer is defined as any debris, calcific in nature, which is produced by reduction or instrumentation of enamel, dentin, and cementum.Smear layers interfere with any adhesive bond formed between the tooth structure and the restorative material.There are two strategies used to overcome the low attachment strengths of smear layer: Removal of the smear layer prior to bonding by acid etching Use of bonding agents that can penetrate the smear layer and incorporate it into the bonding layer Removal of smear layer has these consequences: Increases wetness of dentin Increases bond strength Makes microleakage potentially more significant as a barrier to the pulp is removed Increases fluid flow onto the exposed dentin surface

Which of the following instruments is most useful when polishing an amalgam restoration? All answers apply Prophylaxis cup with tin oxide Abrasive-impregnated rubber points Sof-Lex disks

All answers apply Various polishing instruments, such as Sof-Lex disks, abrasive-impregnated rubber points, and prophylaxis cup with tin oxide are used when polishing amalgam restorations.It is advised to carry out finishing and polishing of amalgam restorations at least 24 hours AFTER the placement of the amalgam so that complete setting of the amalgam can occur.

Which of the following esthetic properties does dentin provide? All answers apply. Fluorescence No answers apply. Opaqueness Chroma

All answers apply. Dentin provides chroma, opaqueness, and fluorescence to above-lying dental enamel. Yellowish-white color of natural teeth is due to the underlying dentin.Chroma is the intensity of any particular hue or color. Fluorescence is the emission of light by a substance that has absorbed light or other electromagnetic radiation.Opaqueness describes the absorption and scattering of radiation in a medium. It is neither transparent, allowing all light to pass through, nor translucent, allowing some light to pass through.

The American Dental Association (ADA) statement on posterior resin-based composite restorations suggests their use for which of the following? All answers are correct. Areas where esthetics is important Conservative tooth preparations Small- and moderate-sized restorations

All answers are correct. The American Dental Association statement on posterior resin-based composites endorses the use of posterior composites in the following conditions: Small- and moderate-sized restorations Conservative tooth preparations Areas where esthetics is important Posterior composite resins are contraindicated in the following situations: Large-sized cavity preparations Areas where isolation is difficult to achieve Exposed root surfaces where it is difficult to achieve optimum bonding

Which of the following is the reason zinc is a constituent of amalgam? It enhances the mechanical properties. It reduces marginal fracture. It prolongs the service of the restoration. All answers are correct.

All answers are correct. Explanation Zinc is added to amalgam to prevent the oxidation of other metals in the alloy during the manufacturing process, which keeps the alloy from turning dark. Zinc accomplishes this by combining readily with oxygen to form zinc oxide. Various benefits of zinc in amalgam include the following: It enhances the mechanical properties of amalgam. It reduces marginal fracture rate. It prolongs the service of the restoration.

Which of the following clinical scenarios would warrant antibiotic prophylaxis prior to major dentistry? A patient with a prosthetic artificial heart valve A patient with a previous history of endocarditis A patient with repaired cyanotic congenital heart defect A patient with a valvulopathy after heart transplant

All of the above

Which of the following terms regarding periodontal instrumentation is correctly paired with its definition? Stroke: a single, unidirectional motion that caries out the instrument's function Adaptation: refers tot the relation between the working end and the tooth surface Angulation: describes the relation between the instrument tip and tooth Activation: is the motion of the hand pivoting on a fulcrum to produce a stroke

All of the above are correct

Which of the following statements regarding the creep of amalgam is correct? Alloys with a higher copper content demonstrate lower creep values. Alloys with low copper content demonstrate lower creep values. Creep value is NOT a property of amalgam restorations, only nobel ones. The lower the creep value the worse the marginal integrity of the restoration.

Alloys with a higher copper content demonstrate lower creep values. Alloys with higher copper content demonstrate lower creep values because they exhibit little or no gamma 2 phase.Creep is defined as the time-dependent, plastic, permanent deformation of amalgam under static or dynamic loading.Creep causes protrusion of amalgam out of the cavity. The protruded edges are unsupported, weak, and are further weakened by corrosion leading to fracture, ditching, and microleakage of the restoration.The gamma 2 phase is primarily responsible for higher creep values.

Which of the following agents is prescribed as an anti-Parkinson drug?

Amantadine Amantadine is a weak antagonist of the N-methyl-D-aspartate (NMDA) type of glutamate receptor, which increases dopamine release and blocks dopamine reuptake. Amantadine was used as an antiviral agent against influenza A until 2009.

Which of the following pharmaceuticals is the etiologic agent in the remission of acute leukemia in children? Mercaptopurine Mechlorethamine Aminopterin Carmustine Streptozocin

Aminopterin Aminopterin is a 4-amino analog of folic acid that acts as an antineoplastic agent with immunosuppressive properties. It was replaced by methotrexate due to its more favorable therapeutic index. Aminopterin competes for the folate-binding site of the enzyme dihydrofolate reductase, blocking tetrahydrofolate (THF) and resulting in the depletion of nucleotide precursors and inhibition of DNA, RNA, and protein synthesis.

Which of the following should a dentist primarily consider when restoring the incisal edge or enameloplasty for a patient with slight chipping of the incisal edge of teeth #8 and #9? Height of the smile line Shape of incisal embrasures Location of proximal contacts Amount of translucent enamel present

Amount of translucent enamel present Explanation It is very important to consider morphology and translucency when restoring damaged incisal edges of maxillary incisors. The thickness of enamel in the incisal surface is greater than that of the facial surface. translucency. the quality of allowing light to pass diffusely

Which of the following antibiotics acts upon the 50S ribosomal subunit?

Antibiotics that attack the 50S ribosomal subunit include the following: Azithromycin Clarithromycin Erythromycin Lincomycin Minocycline prevents the function of the 30S ribosomal subunit. Amoxicillin, like penicillin, prevents NAG-NAM cross-linkages in cell walls.

A 40-year-old patient diagnosed with an acute apical abscess presents to your office for the extraction of tooth #3. Penicillin prophylaxis is administered because the patient has a history of rheumatic fever and valve replacement. Periorbital and labial edema are discovered 15 minutes after administration of the penicillin. Which is the most likely diagnosis for what this patient is experiencing? Angioedema Hematoma formation Congestive heart failure An acute anaphylactoid reaction

Angioedema In this patient, an allergic reaction to penicillin has initiated angioedema. As mast cells react against an allergen, histamine release occurs, producing an allergic reaction and angioedema. Symptoms of angioedema include urticaria, shortness of breath (in severe cases), and swelling of the periorbital area, lips, and tongue.

The behavior theory model describes the ABCs of change. What are the ABCs?

Antecedent Behavior Consequence (of behavior) The behavioral theory, or ABC model, describes the occurrence of a particular behavior can be understood as a complex interaction between an antecedent, a behavior itself, and the consequences of a behavior.

Carbamazepine

Anticonvulsant

Phenobarbital

Anticonvulsant

A 53-year-old male presents for a full mouth tooth extraction and reports taking 10 mg oral prednisone daily to control his ulcerative colitis for the past five years. After consulting with the prescribing physician, what is the most likely course of action?

Ask the patient to continue taking prednisone and consider temporarily increasing the dose. Prednisone is an immunosuppressing steroid drug that can affect the health condition of the patient during and after surgical treatment. Intake of corticosteroid drugs like prednisone should not be immediately terminated. The physician should advise the patient to temporarily increase the dose to help cope with the stress of surgery. Sudden reduction or total absence of prednisone may induce a fatal crisis among patients who take it regularly. Prednisone is a glucocorticoid along with cortisol that when administered exogenously can be used to treat conditions like Addison's disease Prednisone acts by inhibiting Nuclear Factor Kappa B (NFKB) which normally mediates inflammation through the production of inflammatory mediators

Essential hypertension can be treated with administration of a β-adrenergic receptor-blocking drug. Which of the following agents is a beta blocker?

Atenolol Atenolol is a selective β1 receptor antagonist used to treat low-grade essential hypertension. Atenolol is currently recommended only in special circumstances as complementary medication in hypertension. Angiotensin-converting enzyme (ACE) inhibitors and diuretics are more commonly used as first-line therapy. Valsartan is an angiotensin II receptor antagonist (more commonly called an ARB, or angiotensin receptor blocker). Verapamil is a calcium channel blocker. Hydralazine is a direct-acting smooth muscle relaxant used to treat hypertension by relaxing vascular smooth muscle and dilating arteries and arterioles, decreasing peripheral resistance, lowering blood pressure, and decreasing afterload. Phenytoin is an antiepileptic drug.

Which of the following treatment options for temporomandibular joint derangement is the least invasive?

Athrocentesis Arthrocentesis (also called joint aspiration) is a procedure where a doctor uses a needle to take fluid out of a joint. Joints are where two bones meet. They allow our bodies to move. The hips, knees, ankles, elbows, shoulders, and knuckles are all joints. Athrocentesis is a procedure in which ports or cannulas are placed into the TMJ to provide a vehicle to deliver lavage (In medicine, therapeutic irrigation or lavage is cleaning or rinsing) to the joint. This also breaks up adhesion in the joint. This procedure is surgical and is the most minimally invasive Disc repositioning is an invasive surgical procedure in which the joint is exposed and the disc is repositioned. In some cases it can provide relief from symptoms but in 10-15% of cases there is no improvement or worsening of the condition.

Which of the following is an antimuscarinic agent used to treat bradycardia?

Atropine Atropine is considered the first-line drug to be used to treat bradycardia. Atropine increases the heart rate by blocking the action of the vagus nerve and increasing the firing of the sinoatrial (SA) node. Intravenous injection of atropine is only a temporary solution for bradycardia. Installation of a pacemaker is a better and more effective measure. Atropine increases the heart rate and improves the atrioventricular conduction by blocking the parasympathetic influences on the heart.

Which of the following are recognized methods to cure resin sealants? Auto-curing Chemical polymerization Photopolymerization

Auto-curing 28% Chemical polymerization 40% Photopolymerization Pits and fissure sealants are most efficient in sealing the caries-susceptible areas in a tooth.Pits and fissure sealants are placed after minimal or no preparation of the tooth surfaces and then cured to make a hermetic seal.Self-cure, or auto-cure, pits and fissure sealants work through chemical polymerization.Visible light-curing (VLC) sealants are cured through the mechanism of photopolymerization with the help of visible light-curing units. 0 Comments

Gardner syndrome is associated with: Multiple osteomas Intestinal polyps Multiple basal cell carcinomas of the skin Supernumerary teeth

B. 1,2, and 4 Gardner syndrome is an autosomal dominant condition associated with multiple osteomas and supernumerary teeth as well as intestinal polyps and fibromas of the skin, These patients are also at a high risk for colorectal cancer since the intestinal polyps tend to undergo malignant transformation Multiple basal cell carcinomas of the skin are associated with nevoid basal cell carcinoma syndrome or goblin syndrome

Antibiotics have been discovered and harvested from

Bacteria and fungi Antibiotics are substances derived from bacteria, fungi, or other organisms that have the ability to disrupt or destroy the development and growth of other microorganisms. Antibiotics are procured by means of biosynthesis, wherein a living organism creates the antibiotic in a controlled environment set up by the laboratory to be harvested. The substances derived from these microorganisms are then studied and modified to improve their qualities and make them last longer.

Which type of financial arrangement allows the dentist to charge the patient the difference between what the insurance plan agrees to pay and the dentist's usual customary rate (UCR)? Prospective reimbursement Payment differential Managed care Balance billing

Balance billing

The material used to protect the pulp and act as a dentin replacement is?

Base Base, usually cement, is used to prepare a cavity before the insertion of a permeant restoration. It is used to protect the pulp tissue and replace any dentin removed [a cavity sealer is a material which occludes the dentinal tubules and provides a protective barrier for a freshly cut tooth of the prepared cavity]

Which of the following systemic diseases is associated with aphthous-like ulcerations?

Behcet's disease Celiac disease: caused by a reaction to gliadin, a protein found in wheat IgA deficiency: Immunoglobulin A (IgA) is the first line of defence in the resistance against infection, via inhibiting bacterial and viral adhesion to epithelial cells and by neutralisation of bacterial toxins and virus, both extra- and intracellularly. {also associated with apthous ulcers] -Nutritional deficiencies, such as iron, B1, B2, B6, and B12 are associated with pathos-like ulcerations

Which of the following is the best antibiotic for the treatment of primary syphillis?

Benzathine penicillin The first line treatment for primary syphillis is intramuscular benzathine penicillin G, or a single dose of oral azithromycin

Which of the following describes the mechanism through which propranolol functions as an antianginal?

Blocking the β-adrenergic receptors of cardiac muscle Propranolol is the first non-selective beta blocker developed and is used to treat hypertension, anxiety, and panic attacks. Propranolol blocks the action of epinephrine and norepinephrine on both β1- and β2-adrenergic receptors and exhibits little intrinsic sympathomimetic activity. Due to its high penetration across the blood-brain barrier, lipophilic beta blockers such as propranolol and metoprolol are more likely to cause sleep disturbances such as insomnia, vivid dreams, and nightmares.

Which is NOT a characteristic of ectodermal dysplasia? Defective hair Hypoplastic sweat glands Blue sclera Atrophic skin Hypodontia (oligodontia)

Blue sclera Blue sclerae are NOT a characteristic of ectodermal dysplasia but are a clinical sign that usually manifests in diseases like osteogenesis imperfecta, Ehlers-Danlos syndrome, and Marfan syndrome. Ectodermal dysplasia is a group of genetic disorders that occur due to the poor development of organs and body structures that are ectodermal in origin. Ectodermal dysplasia may cause abnormal development or failure of formation of the structures within the body that originate from the ectoderm, which include the teeth, skin, hair, and sweat glands.

Which of the following side effects might patients experience if they are prescribed a strong anticholinergic to reduce their salivary flow?

Blurred vision Atropine is an anticholinergic agent used to reduce salivary flow. Common side effects of atropine include blurred vision, photophobia, tachycardia, and anhidrosis. Anhidrosis is the inability to sweat normally. When you don't sweat (perspire), your body can't cool itself, which can lead to overheating and sometimes to heatstroke — a potentially fatal condition. Anhidrosis — sometimes called hypohidrosis — can be difficult to diagnose

Periodontal packs usually contain zinc oxide and always contain eugenol Packs usually enhance healing rate of tissues

Both statements are false Most surgical sites are covered with a periodontal pack. Packs are placed to protect the surgical wound, minimize patient discomfort, maintain tissue placement, and help prevent postoperative bleeding. Packs usually do not enhance healing rate of tissues. Packs usually contain zinc oxide and may be either eugenol or non eugenol containing. Antibiotics have been incorporated into some packs. Packs are retained mechanically by interlocking into interdental spaces.

Alveoloplasty is best characterized by which of the following? Soft tissue is repositioned to increase denture-bearing area. Bone is added to the ridge to increase the overall ridge height. Bone is smoothed to remove ridge undercuts. Gingiva and bone are reduced apically around a tooth to restore biologic width. Excess gingival tissue is removed.

Bone is smoothed to remove ridge undercuts. Alveoloplasty is the smoothing and recontouring of alveolar bone to create a topography favorable for proper wound healing and denture support.The types of alveoloplasty are as follows: Post extraction alveoloplasty recontours the alveolar ridge to remove knife-edge residual ridges and spiny bony projections, which are not good denture stress-bearing areas. They are also used to remove undesirable ridge undercuts, which alter the path of denture placement and consequently affect retention. Cortical alveoloplasty recontours and smooths bone so that no spiny bone projections or sharp bone margins are present. This ensures optimal healing. Intraseptal alveoloplasty removes labial undercuts, reduces maxillary overjet, and recontours cortical bone by removing interradicular bone and collapsing the cortical plates, thus preserving denture stress-bearing area.

Which of the following components of hydrocolloid materials can inhibit thee setting of gypsum on the surface?

Borax The water content of the hydrocolloid impressions can inhibit the setting of the gypsum at the surface. Borax is used as a filler in agar and is known as gypsum retarder that can prevent the surface of the gypsum cast from completely setting.

Statement 1: Though the cause of trigeminal neuralgia is unknown, it is thought to be a result of focal demyelination of the trigeminal nerve root. Statement 2: Some common medications for the treatment of trigeminal neuralgia include anticonvulsants and tricyclic antidepressants

Both statements are correct Trigeminal neuralgia is thought to be caused by pressure on the root area of the trigimenal nerve that elicits demyelination. It is the demyelination that then causes unusual activity in the nerve. Medications such as carbamazepine and gabapentin (anticonvulsants) are classically used to treat this condition. In severe cases, surgical intervention is used to decompress the microvasculature. Unnecessary dental treatment is often initiated in patients with trigeminal neuralgia of intramural sites. IT is therefore critical for dentists to recognize trigeminal neuralgia to avoid this unnecessary treatment like root canals or extractions. Carbamazepine is a sodium channel blocker. It binds preferentially to voltage-gated sodium channels in their inactive conformation, which prevents repetitive and sustained firing of an action potential. demyelinating diseases most commonly result in vision loss, muscle weakness, muscle stiffness and spasms, loss of coordination, change in sensation, pain, and changes in bladder and bowel function.

There are no major differences in enamel and dentin bonding. They are both equally predictable and strong

Both statements are false Both enamel and dentin bonding involve micro mechanical retention. Acid-etched enamel allows for a more uniform and predictable bond strength than dentin bonding. This is primarily due to the variations in the types of dentin (normal, sclerotic, primary, secondary, coronal, or root dentin). Etching dentin leads to the smear layer being removed and would expose dentinal tubules, leading to increased risk of post-operative sensitivity.

Statement 1: Concious sedation is defined as a drug-induced depression of conciousness during which patients cannot easily be aroused Statement 2: During consciousness sedation patients still respond purposefully to repeated or painful stimuli, though they may require assistance to maintain ventilatory function

Both statements are false Deep sedation is defined as a drug induced depression of consciousness during which patients cannot be easily aroused. During deep sedation patients still respond purposeful to repeated or painful stimuli, though they may require assistance to maintain ventilatory function Concious sedation refers to a minimally suppressed level of consciousness in which a patient is able to maintain his or her own airway and respond to physical stimulation or verbal commands

The advantages of restoring the proximal lesion from the lingual with composite for a Class III are: The facial enamel is conserved for enhanced resistance form Color matching of the composite is critical

Both statements are false The advantages of restoring the proximal lesion from the lingual approach include Enhanced esthetics since the facial surface enamel is conserved Shade matching of the composite to the tooth structure may not be as critical Color change of the restoration is less visible When restoring an anterior tooth that has a proximal carious lesion, the lingual access is desirable.

Spherical alloys contain more mercury than admixed alloys. The higher mercury content explains why it is easier to achieve a contact with a spherical alloy. Both statements are true. Both statements are false. Only the first statement is true. Only the second statement is true.

Both statements are false. Spherical amalgam alloys do NOT contain more mercury than the admixed alloys.Spherical alloys are easier to condense into areas of difficult access, such as around pins, because they provide less resistance to condensation pressures. They harden rapidly and are smoother for carving, burnishing, and polishing.Spherical alloys are more difficult to use in making the interproximal contacts tight because of their pliable nature. Admixed alloys are recommended for areas requiring development of proximal contacts.

The Environmental Protection Agency (EPA) regulates the waste in a dental office. Occupational Safety and Health Agency (OSHA) regulates the transport of waste from a dental office. Both statements are true. The first statement is true; the second is false. The first statement is false; the second is true. Both statements are false.

Both statements are false. The Occupational Safety and Health Administration (OSHA) ensures that workers will be provided ergonomic working places. OSHA protects healthcare professionals from occupational hazards by creating standards regarding recognition, control, and prevention of threats in the workplace. OSHA has partnered with the American Dental Association (ADA) to provide better training resources to dental professionals to ensure that dental health hazards are recognized and minimized. The EPA established maximum exposure levels for Hg vapor. They are intended to be highly protective at 0.1 µg/kg body weight per day. The EPA also regulates transportation of dental waste from dental offices.

Follicular cysts may undergo neoplastic transformation and become ameloblastomas, squamous cell carcinomas or mucoepidermoid carcinomas. For this reason, it is imperative to fully remove the cystic lining to prevent such deleterious changes.

Both statements are true A follicular cyst is also known as a dentigerous cyst. They are benign growths originating from the follicle around the developing crown. The lining of such cysts may undergo malignant change and become aggressive ameloblastoma, SCC, or mucoepidermoid carcinoma. For this reason, the lining of the follicle must be completely removed to prevent these dangerous conditions.

Adhesive dentistry now allows for more than just replacing carious and fractured tooth structure. Modern adhesive techniques allow for the correction of anesthetic shapes, positions, dimensions, and closing of a diastema.

Both statements are true Adhesive techniques have been incorporated into the bonding of veneers, inlays, and onlays

A minimum thickness of porcelain for a metal-ceramic restoration is 0.7mm The optimum amount of porcelain thickness for a metal ceramic restoration is 1-1.5mm

Both statements are true Porcelain should be kept to a minimum thickness that is still compatible with good esthetics. Relatively thin porcelain, of uniform thickness and supported by rigid metal, is strongest. The minimum thickness of porcelain should be 0.7mm for a PFM restoration and the desirable thickness is 1.0-1.5mm. When the thickness of porcelain is beyond 2mm there is an increase of porcelain fracture and may weaken the restoration

Definition: Beading is the procedure of scribing a rounded groove (o.5mm) outlining the anterior and posterior borders of the maxillary major connector of a removable partial denture Purpose: Beading an RPD adds strength to the major connector and maintains tissue contact to prevent food impaction

Both the definition and purpose are true The metal margin produced by the 0.5mm beading is easily finished and provides intimate tissue contact, preventing food from easily dislodging the prothesis. Care should be exercised in adapting such a beaded margin to non compressible tissue, such as the median palatal raphe. Beading is readily accomplished with a cleiod carver. A slightly rounded groove is preferred to a V-shaped groove

Permanent mandibular canines and first premolars are of greatest concern in serial extraction cases because the canines often erupt before the mandibular premolars and crowding of anterior teeth may occur if the normal eruption sequence is disrupted. NEITHER the statement NOR the reason is correct. The statement is correct, but the reason is NOT. Both the statement and the reason are correct and related. The statement is NOT correct, but the reason is correct.

Both the statement and the reason are correct and related. Serial extraction procedures are performed to prevent the crowding of teeth. The canines often erupt before the mandibular premolars, and crowding of anterior teeth may occur if the normal eruption sequence is disrupted. In serial extraction, primary canines are extracted when a child reaches age eight or nine to facilitate incisor alignment. The primary first molars are extracted after one year to hasten the eruption of the first premolars. The erupted first premolars are then extracted to facilitate canine eruption and alignment in the arch.

Material safety data sheets (MSDS) are not required under Occupational Safety and Hazard Administration (OSHA)'s Bloodborne Pathogen Standard because they serve no purpose in dealing with bloodborne pathogens. NEITHER the statement NOR the reason are correct. Both the statement and reason are correct but NOT related. The statement is correct, but the reason is NOT. Both the statement and the reason are correct and related.

Both the statement and the reason are correct and related. The MSDS is not regulated based on the Bloodborne Pathogen Standard. The MSDS indicates procedures for the proper management and safe handling of products. The MSDS contains information about the product's physical data, storage, toxicity, health effects, disposal, and other relevant data.

Nystatin is indicated to treat infections caused by the microbe

Candida albicans. Nystatin is an antifungal drug that is most effective against infection caused by Candida species. Candida albicans is an opportunistic fungus that is considered normal human flora, specifically in the mouth, skin, and gastrointestinal and genitourinary tracts. Candida albicans is opportunistic and can cause fungal infections in immunocompromised patients. Nystatin is very effective against Candida albicans and is activated by binding to the cell membrane of fungi and forming porosities, which result in the leakage of K+ and eventually cell death.

A patient undergoing radiation therapy of the head and neck is most likely to suffer from Aggregatibacter actinomycetemcomitans. Candida albicans. Histoplasma capsulatum. Staphylococcus aureus. Streptococcus viridans.

Candida albicans. adiotherapy is one of the treatment modalities used in the treatment of oral cancer.Radiotherapy may create an imbalance in the oral microflora and a marked decrease in the production of saliva and alteration of the oral lining.All these factors contribute to the development of candidiasis, an opportunistic fungal infection caused by Candida albicans.

Which of the following would be a common intraoral condition found in a diabetic patient? Candida infections Geographic tongue Leukoplakia Papillary hyperplasia Lichen planus

Candida infections Diabetics are susceptible to bacterial, viral, and fungal infections, including periodontal abscesses and Candida infections. The following describes the other conditions: Geographic tongue (benign migratory glossitis) typically involves the dorsal and lateral borders of the tongue. The loss of filiform papillae creates erythematous patches surrounded by a white or yellow perimeter. The condition is benign and requires no treatment. Leukoplakia is a white patch or plaque of oral mucosa that cannot be wiped off. Leukoplakias are most commonly associated with tobacco usage. Papillary hyperplasia is generally seen on the palate in the form of denture stomatitis. The palatal mucosa is covered with multiple red and inflamed papillary projections.

Which antibiotic is bacteriocidal and is commonly prescribed to treat pseudomonas infections? Carbenicillin Dicloxacillin Penicillin VK Tetracycline Doxycycline

Carbenicillin Carbenicillin is a bacteriolytic antibiotic belonging to the carboxypenicillin subgroup of the penicillins. Carbenicillin has Gram-negative coverage, which includes Pseudomonas aeruginosa, but limited Gram-positive coverage. Carboxypenicillins are susceptible to degradation by beta-lactamase enzymes.

Long-term exposure to low, chronic doses of x-ray radiation poses which of the following serious risks? Radionecrosis Purpura Diarrhea Carcinogenesis Alteration of oral microflora

Carcinogenesis Repeated low doses of ionizing radiation may induce irreparable DNA damage, which leads to errors in replication and transcription that result in carcinogenesis. Persistent low-dose radiation can induce a type of genomic instability in cells that may trigger viral interactions, leading to premature aging and carcinogenesis. Ionizing radiation results in enhanced frequency of mutation and other genetic changes in the descendants of the irradiated cell after many generations of replication. Gap-junction-mediated cell-cell communication and activation of the p53 damage response pathway may result in carcinogenesis.

In a patient with cusps undermined by decay and heavy occlusal forces, which of the following is the restorative material of choice? Glass ionomer Cast gold Amalgam Composite resin

Cast gold In a patient with undermined cusps and heavy occlusal forces, cast gold is the restorative material of choice.Gold is the most durable restorative material.Cast gold can easily be finished, polished, burnished, and adapted to tooth structure to decrease recurrent caries risk.Cast gold has the highest corrosion resistance of any restorative material. 0 Comments

A 60-year-old female presents to your office with an 8-by-8 mm, bluish-black, soft fluctuant lesion of her lower lip. She states that the lesion is asymptomatic, has exhibited persistent growth over the last 10 years, and bleeds profusely when she bites her lip. Which of the following is the most likely diagnosis? Verrucous carcinoma Nodular melanoma Neurofibroma Cavernous hemangioma Lipoma

Cavernous hemangioma Cavernous hemangiomas are a class of blood vessel malformation characterized by a large, blood-filled space and contain blood, not tissue. Hemangiomas can arise virtually anywhere in the body and are considered benign neoplasms. Unlike capillary hemangiomas, cavernous hemangiomas can be disfiguring and do not tend to regress. Cavernous hemangiomas may lead to spontaneous or traumatic bleeding.

If a urethral swab is cultured and reveals gram negative diplococci, treatment with which of the following would be most appropiate?

Ceftriaxone (class:cephalosporin) The treatment of choice in gonorrheal infections is ceftriaxone CDC recommends a single dose of 500 mg of intramuscular ceftriaxone. Alternative regimens are available when ceftriaxone cannot be used to treat urogenital or rectal gonorrhea. Ceftriaxone works by inhibiting the mucopeptide synthesis in the bacterial cell wall. The beta-lactam moiety of ceftriaxone binds to carboxypeptidases, endopeptidases, and transpeptidases in the bacterial cytoplasmic membrane. These enzymes are involved in cell-wall synthesis and cell division.

The scale used in Black's three number formula for instruments is in which of the following units for the third number?

Centigrades Black created a three-number formula for instruments that have a primary cutting edge at a right angle to the one axis of the blade. The numbers represent the blade's width, length, and angle

What is the first step when delivering a cast gold crown?

Check proximal contacts IT is important to check the proximal contacts firs to ensure that the crown can be seated fully and that an open contact is not present. [next check the occlusion and then cement the restoration if everything is acceptable. After proximal contacts have been established it is important to check the marginal fit [With gold restorations, it is possible to burnish the margins to ensure that the margins are tight and sealed]

Composite is bonded to adhesive by which of the following means? No answers apply Both mechanically and chemically Chemically Mechanically

Chemically The composite-resin restorative material is chemically bonded to the adhesive resin through polymerization through methyl methacrylate (MMA) bonds.Mechanical bonding occurs between the adhesive and tooth substrate, such as dentin or enamel.The hybrid layer, or hybrid zone, is where the low viscosity resins micro-mechanically interlock with dentinal collagen.

During the administration of opioid analgesics, stimulation of which of the following is responsible for the feelings of nausea and vomiting?

Chemoreceptor trigger zone The chemoreceptor trigger zone (CTZ) at the medulla oblongata functions by receiving input signals from hormones or certain blood-borne medications, then relays it toward the vomiting center of the brain to induce vomiting. The neurotransmitters involved in the control of vomiting and nausea may include dopamine, histamine, serotonin, substance P, and acetylcholine. The presence and activation of opioid receptors may be the reason administration of opioid analgesics may cause nausea and vomiting.

Chloral Hydrate

Chloral hydrate is metabolized in vivo to trichloroethanol, which is responsible for secondary physiological and psychological effects.[25] Chloral hydrate is structurally and somewhat pharmacodynamically similar to ethchlorvynol, a pharmaceutical developed during the 1950's that was marketed as both a sedative and a hypnotic under the trade name Placidyl. In 1999, Abbott, the sole manufacturer of the drug in the United States at the time, decided to discontinue the product. After Abbott ceased production, the drug remained available for about a year. Despite the fact that it could have been manufactured generically, no other company in the United States chose to do so. The metabolite of chloral hydrate exerts its pharmacological properties via enhancing the GABA receptor complex[26] and therefore is similar in action to benzodiazepines, nonbenzodiazepines and barbiturates. It can be moderately addictive, as chronic use is known to cause dependency and withdrawal symptoms. The chemical can potentiate various anticoagulants and is weakly mutagenic in vitro and in vivo

Which of the following antibiotics is most commonly associated with aplastic anemia? Chloramphenicol Bacitracin Clindamycin Azithromycin Vancomycin

Chloramphenicol Chloramphenicol is a broad-spectrum bacteriostatic antibiotic associated with a risk of the following: Pancytopenia (bone marrow inhibition) Aplastic anemia Gray baby syndrome Gray baby syndrome describes liver failure in fetuses that results from the administration of chloramphenicol. Chloramphenicol is now most commonly used to treat eye infections.

Which of the following maintains a continuous antimicrobial effect, or substantivity? Alcohol Anionic detergent Cationic detergent Chlorhexidine gluconate Quaternary ammonium compounds

Chlorhexidine gluconate Chlorhexidine gluconate elicits an antimicrobial effect that lasts for 12 hours. Chlorhexidine gluconate is a chemical antiseptic that has an effect against Gram-positive and Gram-negative bacteria. It has a bactericidal and bacteriostatic effect by disrupting the bacterial membrane. As an oral rinse, it helps prevent the formation of bacterial biofilm, preventing the formation of plaque. Alcohol (greater than 70% ethanol or isopropyl solution) is a very effective surface disinfectant but is more often used as an antiseptic, the distinction being that alcohol tends to be used on living tissue rather than nonliving surfaces. Alcohol functions by disrupting the cell membrane and evaporating quickly, so there is limited residual activity. Detergents help loosen and remove microbes from surfaces with rinsing. Once the surface is rinsed, the detergent is no longer present and is not considered substantive. Quaternary ammonium compounds (quats) like benzalkonium chloride are believed to act by disrupting the cell membrane and are lethal to a wide variety of organisms except endospores, mycobacterium tuberculosis, and non-enveloped viruses. Quats are cationic detergents, as well as disinfectants that need to be rinsed off and are not substantive due to the rinsing.

In the case of a diabetic patient with an apical abscess requiring an extraction where penicillin VK is indicated, which of the following antibiotics should be prescribed if the patient is allergic to penicillin?

Clindamycin Clindamycin is a lincomycin antibiotic that can be used if the patient is allergic to penicillin, and it is very effective in fighting Gram-positive bacteria, including Staphylococcus aureus, which is responsible for pus and abscess formation. Erythromycin is a macrolide antibiotic that binds with the 50S subunit of the bacterial ribosome, inhibiting peptide synthesis. It can indirectly inhibit digoxin metabolism and can thus contribute to serious complications. 75% of patients taking erythromycin have nausea and vomiting, and it has numerous other important drug interactions. This drug is not recommended. Cephalosporins are similar in effect and chemical structure to penicillin, so this drug is not prescribed for patients with a penicillin allergy. Sulfamethoxazole-trimethoprim, doxycycline, and vancomycin are other alternative drugs to penicillin.

During the periodontal assessment of this patient, a 4 mm pocket is detected on the distal surface of tooth #27. The gingival margin is located 2 mm occlusal to the cementoenamel junction (CEJ). Which of the following correctly identifies the periodontal condition of this tooth? Periodontal pocket (4 mm) Clinical attachment loss (2 mm) Pseudopocket Periodontal abscess

Clinical attachment loss (2 mm) The probing was noted to be 4 mm deep. The free gingival margin was noted to be 2 mm coronal to the CEJ. Therefore, the depth of the pocket is 2 mm apical to the CEJ. Clinical attachment loss identifies the distance from the CEJ to the base of the sulcus/pocket, which in this case is 2 mm. According to the American Academy of Periodontology, a pseudopocket is a deepening of the gingival crevice resulting primarily from an increase in the bulk of the gingiva without the apical migration of the junctional epithelium or the destruction of the periodontal ligament and alveolar bone. Had the probing depth stopped at the level of the CEJ, the diagnosis would have been a pseudopocket. Periodontal pockets are formed from persistent, chronic gingivitis, which results in loss of connective tissue attachment, bone destruction, and periodontal pocket formation.

Which of the following antifungal compounds is found in troche form? Ketoconazole Terbinafine Clotrimazole Fluconazole Amphotericin B

Clotrimazole A troche is a small medicated lozenge, such as a cough drop. Clotrimazole is an imidazole that inhibits ergosterol synthesis to prevent fungal cell wall production. It is a topical antifungal delivered as a troche that is dissolved in the mouth. Ketoconazole and fluconazole are systemic antifungals. Terbinafine is typically used to treat fungal infections of the scalp, toenails, or fingernails, and it is not used for troches.

Which of the following is the only anesthetic drug that can both induce vasoconstriction of blood vessels and increase systemic blood pressure? Articaine Lidocaine Mepivacaine Carbocaine Cocaine

Cocaine Cocaine causes an increase in the amount of norepinephrine in synapses by blocking the reuptake of norepinephrine along the peripheral sympathetic nerve endings, resulting in an increase in heart rate.Cocaine can also cause an increased blood pressure level through increasing epinephrine levels which induces a rapid heart rate and an increase in cardiac output.Cocaine is also the only anesthetic drug that can induce vasoconstriction of blood vessels which can also increase blood pressure.

Which of the following describes why lead is used in x-ray machines? No answers apply X-ray production Filtration Heat dissipation Collimation

Collimation Lead is used for collimation, or shaping of the beam, due to its ability to block x-ray photons going in unnecessary directions.Collimation allows the clinician to keep the radiation dose as low as reasonably achievable (ALARA) so excess radiation is blocked before it reaches the patient.Aluminum is used to filter out low-energy x-ray beams. Collimation: Collimation restricts the x-ray beam to the area of interest using lead shutters within the x-ray tube. A secondary beneficial effect of collimation is reduction of off focus radiation making it to the film. Because a smaller volume of tissue is being irradiated, less scatter radiation is produced.

Malignant ameloblastoma should be treated by:

Complete resection Malignant ameloblastoma is an odontogenic tumor that has a high potential for taking over the jaw. Even though this is technically not a neoplasm, (An abnormal mass of tissue that forms when cells grow and divide more than they should or do not die when they should. Neoplasms may be benign (not cancer) or malignant (cancer).) the nature of some ameloblastomas is so destructive that it has been termed malignant. A composite resection involves resection of the tumor, surrounding bone, soft tissues and associated lymph nodes. This type of resection is generally reserved for cancerous growths and malignant ameloblastomas [Radiation therapy and chemotherapy are both excellent treatment modalities for certain neoplasms. However, with respect to ameloblastomas, the first treatment should be that of resection, and a determination would be made at that time if further treatment is required.

Cavosurface margins should be beveled in which type of restoration?

Composite resin Class III Composite preparation margins, including surfaces other than the occlusal, should be beveled to increase the surface area of tooth structure to bond to the resin. Beveling these margins also increases esthetics of anterior composite resins [Because amalgam requires a 90 degree cavosurface angle, no amalgam preparation should be beveled. If it did, amalgam would fracture at the margins due to inadequate bulk.]

Which of the following should NOT be included in a written prescription?

Conscription A prescription is a directive written by an authorized medical professional to the pharmacist. A typical prescription includes prescriber information, patient information, superscription, inscription, subscription, transcription, renewal, and signature. Superscription includes the Rx symbol. Inscription contains the name of the drug, the quantity, and the dosage form. Subscription includes the guidelines to be followed by the pharmacist regarding the preparation and distribution of the medication. Transcription is the information regarding the intake of the medication (e.g., how, how much, when, and how often). Conscription, sometimes called the draft in the United States, is the mandatory enlistment of people in a national service, most often a military service.

Which of the following describes the condition in which teeth are joined only by cementum? Hypercementosis Gemination Fusion Dilaceration Concrescence

Concrescence is a condition in which the cementum overlying the roots of at least two teeth connects to join the teeth together.Surgical separation of the teeth may be necessary if one is to be extracted, which may result in damage to the adjacent tooth root.Incidence of concrescence can sometimes be attributed to trauma or crowding of teeth. Most common in the permanent molars Different from fusion because it occurs following eruption and involves only cementum

What is the name of an instrument used to compress amalgam and/or push resin composite materials into all areas of the preparation?

Condensers Condensers are used to compress filling materials into all areas of preparations. Proper condensation of amalgam promotes adaption of the amalgam to the walls of the preparation, and it compacts the material, eliminating voids and reduces the amount of residual mercury in the restoration. Effective condensation of amalgam will increase the strength and serviceability of the restoration

Which of the following causes discontinuity of the occlusal plane when an ankylosed tooth is present? Localized inhibition of alveolar process growth Ankylosed teeth moving apically as the root becomes incorporated into the alveolar bone Differential eruption sequences Continued eruption of non-ankylosed teeth and growth of the alveolar process

Continued eruption of non-ankylosed teeth and growth of the alveolar process Ankylosed teeth usually lie below the occlusal plane because the supporting alveolar process does not grow.Alveolar process growth typically occurs simultaneously with tooth eruption. Ankylosed teeth do not experience alveolar process growth because of their failure to erupt the same way as the other non-ankylosed teeth due to a lack of periodontal ligament.Ankylosed teeth are teeth with roots completely embedded into the bone and lack a periodontal ligament. 0 Comments

Which of the following chemical bonds is considered to be the strongest and is rarely seen during drug interactions? Van der Waals forces Hydrogen Electrostatic Dipole-dipole Covalent

Covalent Covalent bonds are not commonly involved in drug-receptor interactions due to the almost irreversible nature of their bonding. Drug-receptor interactions are normally considered reversible in nature. Covalent bonds are very tight and practically irreversible. Covalent bond formation is a rare occurrence in drug-receptor interactions.

Parasympathetic innervation is carried by which cranial nerves? IV, VII, VIII, X III, VII, IX, X III, V, VII, IX II, V, IX, X II, III, VII, IX

Cranial nerve III, the oculomotor nerve's parasympathetic component, constricts the pupil.Cranial nerve VII contains parasympathetic innervations to the lacrimal gland as well as the sublingual and submandibular salivary glands.Cranial nerve IX brings parasympathetic fibers to innervate the parotid salivary gland.Cranial nerve X brings parasympathetic innervation to the digestive organs and the heart.

Each of the following conditions is a tumor arising from blood vessels EXCEPT one. Which of the following is the EXCEPTION Hemangiopericytoma Hemangioma Cystic hygroma Angiosarcoma

Cystic hygroma Cystic hygromas (lymphatic malformations) are congenital multiloculated lymphatic lesions that can arise anywhere but are classically found in the left posterior triangle of the neck in children. They are considered to be the most common form of lymphangioma. They contain large, cyst-like cavities containing lymph, and they decrease in size as they approach the surface of the skin. Cystic hygromas are benign but can be disfiguring. There are two types of cystic hygromas: macrocystic lymphatic malformations (large cysts) and microcystic (small cysts). A person can have one type of cyst or a mixture of both macro- and microcysts.

The best treatment for necrotizing ulcerative periodontitis includes:

Debride the teeth and root surfaces that are affected by the presence of biofilm and calculus The use of ultrasonic instrumentation may provide greater irrigation and debridement to deep lesions. Often these treatments are painful and therefore should be completed using local anesthetic Because NUP patients are often somewhat immunocompromised, oral hygiene instructions for both regular in office and home care is of the utmost importance [The prescription of antibiotics is NOT universally indicated. Depending on each patient's situation, the clinician may decide to prescribe antibiotics. This may be the case for patients with a fever or severely immunocompromised status

The letter "D" in "DMFS" stands for which of the following? Determined Diagnostic Decayed Demineralized

Decayed Caries prevalence can be recorded as "DMFT" or "DMFS" as follows: Decayed or carious tooth surfaces of all teeth present within the oral cavity Missing tooth surfaces of all teeth present within the oral cavity Filled surfaces of all teeth present within the oral cavity Surfaces Teeth

Enamelplasty on tooth 11 that results in the unilateral loss of canine guidance would cause which of the following? Decreased clearance on the right side and a greater possibility of a nonworking contact Increased clearance on the right side with less possibility for a nonworking contact No effect on the nonworking side clearance

Decreased clearance on the right side and a greater possibility of a nonworking contact Decreased clearance with a greater possibility of nonworking contact may occur.The decrease in clearance occurs due to the placement of the crown on the left upper canine without restoring the canine guidance. The change to group function occlusion will create a decrease in the vertical clearance on the right side because the teeth will not separate as much as when the patient was in canine guidance.During lateral excursions, there is disocclusion of all the teeth except for the canine in an ideal canine-guided occlusion. In an idealized group function occlusion, multiple tooth contacts should occur during lateral excursions on the working side only.

Which of the following is the correct technique to effectively manipulate a Cavitron (ultrasonic scaler) when removing the calculus in the image? Decreased grasp pressure with continuous movement Increased grasp pressure with continuous movement Decreased grasp pressure with slow movement Increased grasp pressure with slow movement

Decreased grasp pressure with continuous movement Ultrasonic instruments like the Cavitron use rapid vibrations of a fluid-cooled tip to fracture and dislodge supragingival and subgingival calculus deposits from the teeth and to cleanse the environment within the periodontal pocket.Tips function most effectively with light lateral pressure against the tooth or calculus deposit and continuous movement in a back-and-forth motion until the deposit is dislodged.

Each of the following physiologic changes occurs due to amyl nitrite administration EXCEPT one. Which is the EXCEPTION?

Decreased intraocular pressure Amyl nitrite is a short-acting vasodilator administered through inhalation to treat angina pectoris. It takes effect within 30 seconds of inhalation and lasts about five minutes. Amyl nitrite can be used to treat angina pectoris and cyanide poisoning. Physiologic effects include dizziness, headache, decrease in blood pressure, increased heart rate, and relaxation of involuntary muscles. Overdose leads to headache, emesis, nausea, dyspnea, syncope, and hypotension.

Which of the following pharmaceuticals is most closely associated with dermal photosensitivity?

Demeclocycline Demeclocycline is a tetracycline antibiotic used in the treatment of Lyme disease, acne, and bronchitis. All members of this drug class may induce photosensitivity, interfere with bone development, and discolor teeth. Demeclocycline is the only tetracycline known to cause nephrogenic diabetes insipidus. Demeclocycline acts by binding to the 30S and 50S ribosomal subunits, impairing protein synthesis by bacteria.

Which of the following communication tools can help increase the general comfort of the patient while at the dentist? Demonstrate control through a direct interviewing style. Discuss financial expectations early in the appointment Provide an immediate evaluation of the patient's oral health, so the patient does not have to wait and wonder "how bad it is." Limit eye contact to avoid being unduly intimate, in order to maintain your professional relationship.

Demonstrate control through a direct interviewing style. A direct interviewing style helps patients relax through a controlled conversation regarding their dental condition. Directive interviews help the dentist control the flow of the discussion and the pace of the interview while also providing quality time for dentist and patient to discuss matters in a systematic and comfortable manner. Direct interviews give patients a sense of comfort, knowing that the dentist is focusing attention toward them. They also decrease the time spent answering unnecessary questions from patients.

A posterior cross bite in a patient with adequate palatal width would be considered a ______. In this situation, the upper molar are ______

Dental condition; inwardly inclined A patient with adequate palatal width should generally have molars with a normal transverse relationship. This is because there is adequate space for the molars to erupt and occlude onto the buccal cusps of the lower dentition. Because the upper molars should have adequate width to occlude with the buccal cusps of lower teeth, the upper molars must have an incorrect incline in order to create the cross bite.Therefore, the upper molars would be inclined inwardly and likely occlude with the central fossa or even lingual cusps of the mandibular teeth

Which of the following drugs is indicated for use in patients suffering from congestive heart failure?

Digitalis Digitalis drugs are used to treat patients with congestive heart failure and cardiac dysrhythmias. Digitalis makes the cardiac muscles contract more strongly and pump more efficiently, improving blood circulation and reducing edema. Symptoms of digitalis intoxication include the following: Nausea Emesis Diarrhea Cardiac arrhythmia Headache

Each of the following agents is an opioid utilized in outpatient anesthesia EXCEPT one. Which is the EXCEPTION? Meperidine Fentanyl Diazepam Oxycontin

Diazepam Diazepam is a benzodiazepine, not an opioid. It possesses anxiolytic, anticonvulsant, hypnotic, sedative, skeletal muscle relaxant, and amnestic properties. Diazepam has several pharmacologically active metabolites, so it has a long pharmacologic half-life. Although it has a long duration of action, this drug is available in low doses to prevent drug accumulation and an unnecessarily prolonged effect. Opioids are molecules that are known to bind to the opioid receptors to cause anesthesia, sedation, respiratory depression, constipation, and a strong sense of euphoria in patients.

Seizures induced by a lidocaine overdose can be treated by which of the following options? Diazepam Flumazenil Epinephrine Naloxone Meperidine

Diazepam Diazepam is a rapid-acting and highly efficacious anticonvulsant drug indicated to treat anxiety, panic, and acute seizure attacks such as those induced by lidocaine. Injecting lidocaine, which may reach a concentration beyond 8-9 μg/mL, is said to pose a greater risk for acute convulsions. Intravascular diazepam administration can stop a seizure attack triggered by lidocaine. The intramuscular injection of diazepam one hour prior to lidocaine injection can help prevent lidocaine-induced seizure attacks.

Which of the following medications is NOT necessary to keep in stock in the office for use in emergency situations?

Diazepam Diazepam is considered a primary emergency support drug, but not necessary to have in the office. It is used as an anticonvulsant when required.

Which component in dentrifices should be limited in order to prevent inactivation of fluoride ions?' Polyacrylic spheres Monofluorophosphate Dicalcium phosphate Anionic detergents

Dicalcium phosphate The interaction of fluoride ions and dicalcium phosphate dihydrate results in the formation of insoluble fluoride compounds that reduce the amount of free and available fluoride ions. Fluoride ions that interact with dicalcium phosphate dihydrate form calcium fluoride. In order to make more fluoride ions interact with the teeth, incorporating dicalcium phosphate dihydrate within fluoride pastes must be limited.

Digitalis should be prescribed to patients suffering from atrial fibrillation and already on quinidine to prevent

Digitalis should be prescribed to patients suffering from atrial fibrillation and already on quinidine to prevent Digitalis and quinidine are both used as anti-arrhythmic drugs. Digitalis regulates the heartbeat of patients with atrial fibrillation by slowing down the ventricular contraction rate. Digitalis is often prescribed for patients suffering from congestive heart failure.

Which substance is characterized as an irreversible cholinesterase inhibitor? Neostigmine Pilocarpine Physostigmine Diisopropyl fluorophosphate

Diisopropyl fluorophosphate Diisopropyl fluorophosphate (DFP) is an organophosphate insecticide that can act as an irreversible cholinesterase inhibitor. This substance can elicit a parasympathomimetic action and act as an irreversible anticholinesterase. DFP became useful in ophthalmology for its ability to induce miosis (pupillary constriction) for the treatment of chronic glaucoma and in neurological experiments focusing on neuropathy.

Which of the following clinical signs is NOT characteristic of opioid overdose? Pinpoint pupils Comatose state Dilated pupils Decreased respiration rate

Dilated pupils Opioids are potent analgesics that can decrease pain perception, decrease pain reaction, and increase pain tolerance. Common side effects of opioids include the following: Sedation Euphoria Respiratory depression Constipation Symptoms of opiate overdose include the following: Pinpoint pupils or pupillary miosis Seizures Muscle spasms Decreased heart rate Decreased respiratory rate Naloxone can reverse the effects caused by opioid overdose.

Which of the following clinical central nervous system (CNS) signs suggests that a patient is suffering from oxygen deprivation? Pinpoint pupils with stronger light reflex Pinpoint pupils without light reflex Dilated pupils with light reflex Dilated pupils without light reflex

Dilated pupils without light reflex Patients suffering from oxygen deprivation demonstrate dilated pupils without light reflex, because the brain stem that controls pupillary response lacks oxygen, causing the cells to die or be damaged. Routine assessment of the pupillary reflex is utilized by physicians for checking brain stem function. Pupils normally dilate equally to allow more light to enter the eyes, allowing vision even in the dark. Pupils constrict when there is too much light needed by the eyes to see. Lack of pupillary reflex or an unusual abnormal pupillary reflex could be a result of brain stem cell damage, optic nerve damage, oculomotor nerve damage, or due to depressant drugs like barbiturates.

What is the most common cause of actinic chelitis?

Direct UV light exposure Actinic chelitis is also known as sailor's lip. This condition is caused by exposure to ultraviolet light and has potential in developing into squamous cell carcinoma. The most common place it occurs is the lower lip due to exposure of the sun. This condition is more common in individuals of a lighter complexion as well as older males.

In a 4 year old patient, a small mechanical pulp exposure that occurs in an asymptomatic vital primary molar would best be treated by:

Direct pulp capping with Mineral Trioxide Aggregate Although the formecresol pulpotomy technique has been recommended for many years as the principal method for treating primary teeth with carious exposures, a substantial shift away from use of this medicament has occurred because of concerns about its toxic effects. It is generally agreed upon that pulp capping procedures should be limited to small exposures that have been produced accidentally by trauma or doing cavity preparation. MTA is very biocompatible and has better sealing abilities than calcium hydroxide

Which statement describes direct-acting cholinomimetics?

Direct-acting cholinomimetics perform well as receptor agonists. They bind and activate the M2 and M3 muscarinic receptors. Drugs like bethanechol act in the management of urinary retention, and pilocarpine is used in treating glaucoma. Bethanechol is predominantly selective for gastrointestinal and urinary bladder smooth muscle. Methacholine has a more prominent action on the cardiovascular system than carbachol. Bethanechol and carbachol almost completely resist the action of acetylcholinesterase.

Intracapsular restraint of mandibular movements is typically caused by which of the following? Myofacial pain disorders Infection Disc interference disorders Ankylosis

Disc interference disorders Disc interference disorders most commonly cause intracapsular restraint of mandibular movement in adults. Disc interference disorders are associated with dysfunctional condyle disc complex and are also termed as internal derangement disorders. They are commonly associated with anterior and medial displacement or dislocation of the articular disc. Signs and symptoms may include the following: Clicking sounds Catching or locking during jaw movements May be associated with pain

Which of the following tetracycline-class antibiotics is excreted the slowest? Chlortetracycline Doxycycline Tetracycline Oxytetracycline

Doxycycline Doxycycline has the longest half-life of the tetracycline group and is used both prophylactically and therapeutically. Doxycyline is used to prevent malaria and to treat Lyme disease, Rocky Mountain spotted fever, and acne. Taking doxycycline with dairy, antacids, calcium supplements, iron products, or laxatives containing magnesium may decrease the effectiveness of the drug.

Which of the following is the drug of choice for treating Rickettsia rickettsii infection?

Doxycycline Doxycycline is the first-line treatment for R. Rickettsii infections which is a unicellular , gram negative bacteria coccobacillus. Failure to receive treatment may lead to end-organ failure.

Which of the following would NOT be recommended to help alleviate this patient's xerostomia? Drink plenty of water. Utilize salivary substitutes. Drink plenty of fruit juices. Suck on tart, sugarless candy that contains xylitol.

Drink plenty of fruit juices. or patients with xerostomia, instructions should be given to include the following: Drink plenty of water. Suck on tart, sugarless gum or candy with xylitol or ice chips. Avoid products containing alcohol and caffeine. Avoid juices and soft drinks to reduce the risk of dental caries. Utilize salivary substitutes (Biotene, Xero-lube, and Salivart).

Which of the following is the MOST reliable diagnostic indicator of pulp vitality?

Duration of response during thermal test The duration of response during the thermal test is more reliable than the electric pulp test in diagnosing pulp vitality For example, the exact number of the reading is of no significance and does not detect subtle degrees of vitality. The EPT is not infallible and may produce false positives or false negatives 20% of the time.

Partial denture service is logically divided into six phase

Education of patient Diagnosis, treatment planning, design, treatment, and mouth preparation Support for distal extension bases (if applicable) Establishment and verification of occlusal relations and tooth arrangements Initial placement procedures Periodic recall

Which of the following is the most likely adverse effect of internally bleaching an endodontically treated tooth with superoxide? Demineralizeation of the tooth structure Weakening of the remaining tooth structure Reinfection of the canal External cervical root resorption

External cervical root resorption The most common adverse effect of internally bleaching a tooth with superoxide is external cervical root resorption.The presence of a glass ionomer barrier over the root-filling material may help to reduce the instances of external cervical root resorption if it is at least 4 mm in thickness.The walking bleach technique using sodium perborate is NOT associated with external cervical root resorption.Bleaching dentin does NOT weaken the tooth structure.

Each of the following is a mechanism of antihypertensive medications EXCEPT one. Which is the EXCEPTION? Blocking α1-adrenergic receptors Enhancing angiotensin-converting enzyme production Blocking peripheral β-adrenergic receptors Relaxing vascular smooth muscle Decreasing calcium channel conduction

Enhancing angiotensin-converting enzyme production Angiotensin-converting enzyme (ACE) inhibitors decrease ACE production by blocking the conversion of angiotensin I to angiotensin II. Calcium channel blockers are vasodilators. Direct vasodilators dilate the vascular smooth muscle.

Which precaution must be kept in mind when administering local anesthesia to a patient suffering with multiple sclerosis? Anesthetic dosage required for a given procedure is less than for a normal patient. Anesthetic dosage required for a given procedure is more than for a normal patient. Anesthetics do not last as long as they would for a normal patient. Epinephrine in local anesthetics is contraindicated.

Epinephrine in local anesthetics is contraindicated Epinephrine should not be used for patients suffering from multiple sclerosis because epinephrine can aggravate the condition by regulating the activation of blood lymphocytes. Epinephrine is a catecholamine and neurotransmitter responsible for the body's fight-or-flight response. Elevating the levels of epinephrine also increases blood lymphocyte activity, which could cause further damage to the myelin sheaths of the neurons, thereby aggravating the multiple sclerosis condition.

Which of the following erythromycin compounds is enterically coated and has prolonged action? Eryc Erythromycin ethylsuccinate Ilosone Erythrocin

Eryc Eryc capsules are enteric-coated erythromycin created for oral administration. The enteric coating serves as a protective barrier of the erythromycin base against the inactivating action of the gastric acids of the stomach. The small size of the enteric-coated capsules allows them to pass intact as they go through the stomach into the intestines, allowing the erythromycin to be dissolved and absorbed in a uniform manner.

Carcinoma in situ is most often associated histologically with which of the following clinical lesions? White sponge nevus Lichen planus Leukoplakia Hairy leukoplakia Erythroplakia

Erythroplakia Erythroplakias are red lesions that are considered to be one of the most common premalignant lesions in the oral cavity. They are most commonly found on the floor of the mouth, tongue, and soft palate. Histopathological examination of erythroplakias exhibit severe dysplasia, carcinoma in situ, and invasive squamous cell carcinoma. Erythroplakias appear red due to the following premalignant changes: Absence of keratin production Reduced number of epithelial cells Increased vascularity due to inflammation Thin and neoplastic epithelium

One of the earliest signs of gingival inflammation is

Evidence of bleeding upon probing The two earliest signs of gingival inflammation are bleeding upon probing and increased gingival crevicular fluid production. Evidence of a change in consistency of the gingiva, pain, or erythema, all may indicate the presence of gingivitis. However, these signs present themselves AFTER the initial inflammatory signs mentioned above.

Today's resin restorations, as compared to Dr. G.V. Blacks's techniques, are placed in 1mm increments after completing all of the following steps except one?

Extension for Prevention Unlike Dr. G.V. Blacks approach, modern materials have allowed for a more conservative approach to tooth preparation. Resin composite fillings will be shallower, have a narrower outline form, have rounded internal line angles, but do not require extension and prevention.

All are advantages of an internal splint over an external splint EXCEPT one. Which is the EXCEPTION? More tooth structure conserved Increased strength Increased retention Increased rigidity

External splints are always recommended for use when the conservation of tooth structure is of primary importance.Internal splints require removal of tooth structure in order to be used. External splints demonstrate less retention and less rigidity compared to internal splints. External splints typically are fabricated using ligature wires, nightguards, interim fixed prostheses, and composite resin restorative materials. Internal splints, on the other hand, are fabricated using composite resin restorative material with or without wire or fiber inserts.

All of the following factors are inhibited by warfarin (Coumadin) except:

Factor VIII Factor VIII is not inhibited by warfarin Warfarin is an oral anticoagulant that inhibits the vitamin K-dependent synthesis go Factors II (prothrombin), VII, IX, and X. Factor VIII is not inhibited by warfarin

Which of the following medications inhibits H1 histamine receptors and is least likely to cause sedation of the patient?

Fexofenadine Fexofenadine is a second-generation selectively peripheral H1 blocker typically used to treat seasonal allergies and chronic urticaria. Fexofenadine is unable to enter the central nervous system (CNS), unlike diphenhydramine (Benadryl®) and hydroxyzine. Famotidine is also an H2 histamine receptor antagonist.

A 14-year-old boy presents to the dentist's office stating that an area of his left anterior mandible has been slowly and progressively growing over the last two years. Radiographs reveal a radiolucent lesion 2 cm in diameter with a ground glass appearance. Which of the following represents the most probable diagnosis? Paget disease of bone Osteosarcoma Fibrous dysplasia Florid osseous dysplasia

Fibrous dysplasia Fibrous dysplasia is an abnormal bone growth condition where normal bone is replaced with fibrous bone tissue, causing a "ground glass" appearance radiographically. The bones of the skull, thigh, shin, ribs, upper arm, and pelvis are most commonly affected. Seventy percent of cases are monostotic and involve one bone. The condition occurs 50% more frequently in the mandible than in the maxilla. Radiographic features include the following: Usually unilocular Radiopaque (usually) or radiolucent with a ground glass appearance Cortex thinned and may be displaced Displaces anatomical structures and may or may not displace teeth Poorly defined with no corticated border Loss of lamina dura

What is the source of electrons within the X-ray tube?

Filament The basic apparatus for generating X-rays, the X-ray tube, is composed of a cathode and an anode. They lie within an evacuated glass envelope or tube. The cathode in an X ray tube consists of a filament and a focusing cup. The filament is the source of electrons within the X-ray tube. It is a coil of tungsten wire about 2mm in diameter and 1cm or less in length. The filament lies in the focusing cup, a negatively charged concave reflector made of molybdenum. When electrons from the cathode strike the target in the anode, they produce X-rays.

Which instruments are used to finish a composite restoration?

Fine abrasive strip Fine-pointed, multi-bladed tungsten carbide bur Fine "football" shaped finishing diamond bur Flexible, abrasive, impregnanted rubber discs or points [non-striped chamfer bur] [a non-striped chamber bur represents a coarse chamfer bur that is used for making gross preparations of crowns with a chamfer margin

Which of the following statements is correct? Retentive pins should be used in class IV restorations. All answers are correct. Rebonding decreases both marginal integrity and color stability. Finishing diamond burs impart less surface damage than carbide burs. Metamerism aids the dentist when selecting a shade.

Finishing diamond burs impart less surface damage than carbide burs. Finishing diamond burs impart less surface damage as compared to the carbide burs. Retentive pins in class IV restorations are contraindicated because they can show through the thin enamel and give an unaesthetic appearance to the restoration. Rebonding is completed at the end of restorative procedures to seal the surface and does not decrease marginal integrity or color stability. Metamerism is a complication in color perception as various light sources produce different perceptions of color. It creates problems in shade selection.

Which of the following conditions describes a benign, nonpainful appearance of the tongue with multiple grooves and bad breath? Hairy tongue Oral squamous cell carcinoma Benign migrating glossitis Fissured tongue Lingual laceration

Fissured tongue Fissured tongue (scrotal tongue, lingua plicata, plicated tongue, and furrowed tongue) is a benign condition characterized by deep grooves (fissures) in the dorsum of the tongue. These grooves may appear unsettling; the condition is typically painless, although some individuals may complain of an associated burning sensation due to inflammation caused by poor hygiene. It has the following characteristics: Estimated prevalence of 2%-5% May be seen at any age Becomes more accentuated with age Appearance varies in orientation, number, depth, and length of the fissure pattern Usually multiple grooves/furrows 2-6 mm in depth Food particles stuck in fissures may cause halitosis and irritation Hairy tongue (lingua villosa) is a commonly observed condition of defective desquamation of the filiform papillae that causes the tongue to appear "hairy" and can often create a habitat for bacteria to reside and cause bad breath. Benign migrating glossitis (geographic tongue) is a nonpainful inflammatory condition of the mucous membrane of the tongue, usually on the dorsal surface, that affects approximately 2%-3% of the general population.

Which of the following would be a radiographic indicator of the presence of periodontal breakdown when seen on this patient's bitewings? Fuzziness of the interproximal alveolar crests Breakdown of the facial and lingual cortical plates Radiolucent outline near the periodontal ligament space Radiopaque lamina dura outline

Fuzziness of the interproximal alveolar crests Radiographic features of the interdental septum will exhibit the earliest radiographic changes in the periodontal breakdown process. The crest of the interdental septum should normally exhibit a level that is parallel to a line drawn between the cemento-enamel junction (CEJ) of adjacent teeth.

What are the steps of the Snowplow technique and what is it's primary benefit?

Flowable composite, uncured, heavily filled packable composite, cured, elimination of voids The Snowplow technique does utilize a layer of flowable composite, but it is left uncured prior to the placement if packable composite. The packable composite displaces the majority of the flowable composite, but not before it adapts to all of the walls of the preparation allowing for the elimination of voids. When used correctly resin restorations have demonstrated the ability to perform as well as amalgam in posterior teeth for up to 10 years.

Vaginal candidiasis can be treated by which of the following oral antifungal drugs?

Fluconazole Fluconazole is an oral triazole antifungal agent prescribed to treat and prevent superficial and systemic fungal infections by inhibiting the fungal cytochrome P450 enzyme. Inhibition of P450 prevents the conversion of lanosterol to ergosterol, an essential component of the fungal cytoplasmic membrane. Clotrimazole and miconazole are both topical drugs. Griseofulvin is primarily used to treat hair and nail fungal infections. Like amphotericin B and natamycin, nystatin is an ionophore. It binds to ergosterol, a major component of the fungal cell membrane. When present in sufficient concentrations, it forms pores in the membrane that lead to K+ leakage, acidification, and death of the fungus.

A population of children in a rural community exhibit high rates of interproximal caries. Which of the following school-based preventive programs provides the greatest benefit to this population? Xylitol lollipops or candies Sealants Fluoride mouth rinses Brushing and flossing worksheets

Fluoride mouth rinses Fluoride mouth rinses are the best program to institute because they are cheap to implement and carry a high rate of compliance because mouth rinses taste good and take little time to implement. Brushing and flossing worksheets are ineffective because they require homecare and may not be completed. Sealants are ineffective against interproximal caries, but have a high degree of evidence for preventing occlusal decay. Xylitol candies are expensive and take multiple administrations per day to be effective against caries.

The crest of interdental bone should normally:

Follow a parallel line between adjacent cementoenamel junctions Normally, the crest of interdental bone should follow a parallel line between adjacent cementoenamel junctions. If interdental bone were to follow an arc curving coronally, one may suspect a bony pathological process. Nevertheless, such instances should be investigated further. If interdental bone were to follow an arc curving apically one should suspect the early stages of bone loss.

Which of the following materials appears most radiopaque in radiographs? Porcelain Gold Glass ionomer Enamel Composite Acrylic

Gold Metals, like gold, appear the most opaque on radiographs.Acrylic and soft tissues appear radiolucent unless there is an opaquer, like barium, present in the acrylic.Composite resins appearance can range from completely radiolucent to radiopaque.

Which of the following is increased while a patient is being treated with digitalis? Rate of conduction Refractory period of the muscle of the ventricles Refractory period of the muscle of the atrium Force of contraction

Force of contraction Digitalis is considered a cardiac glycoside. It increases the contractile force of the cardiac muscle.

The euphoria experienced from ethanol absorption into the bloodstream is caused by decreased activity in the medullary synapses. decreased inhibitory activity of the frontal cortex. increased activity of the synapses in the limbic system. increased activity of the synapses in the hypothalamic nuclei. increased activity of the synapses in the thalamic nuclei.

Gamma-aminobutyric acid (GABA) and dopamine levels in the frontal cortex are affected when ethanol is found in the bloodstream. The frontal cortex is primarily responsible for impulse control and inhibition. Ethanol alters the response of both GABA and the dopamine receptors. It binds to the GABA, increasing the GABA receptor response, leading to a reduction in the inhibition of socially unacceptable behaviors. Ethanol increases the level of dopamine found near the dopamine receptor. Enhanced activity of dopamine induces an intense feeling of pleasure, happiness, and self-confidence.

Which of the following are treatments for sialoliths?

Gentle massage Moist heat Increased fluid intake Sialogoues (drugs that stimulate salivary flow) Surgical removal

During the intraoral examination, the clinician notes that the dorsal and lateral borders of the tongue demonstrate an appearance of white lines and red patches that are non-ulcerated and do not rub off. The patient notes that she has had episodes of burning sensation on her tongue associated with these patches. Which of the following conditions is most likely associated with these signs and symptoms? Pseudomembranous candidiasis Fissured (scrotal) tongue Median rhomboid glossitis Geographic tongue

Geographic tongue Geographic tongue (benign migratory glossitis) typically involves the dorsal and lateral borders of the tongue. The loss of filiform papillae creates erythematous patches surrounded by a white or yellow perimeter. As the name suggests, the condition is benign and requires no treatment. Median rhomboid glossitis appears as an erythematous area at the midline of the dorsal surface of the tongue, which is devoid of filiform papillae. This condition is often seen in immunocompromised patients and is associated with chronic Candida albicans infection. Fissured (scrotal) tongue is manifested as fissures or grooves on the dorsal surface of the tongue, which can become irritated if food collects in them. The treatment involves brushing the tongue with a soft-bristled toothbrush or tongue scraper. Candida albicans (thrush) is the most common oral fungal infection and appears as white plaques that wipe off to reveal underlying red mucosa. The treatment is to apply topical solutions of antifungal medications (nystatin, clotrimazole).

Each of the following is an indication for using resin composite materials EXCEPT one. Which is the EXCEPTION? Small- and moderate-sized restorations Areas where esthetics is important Conservative tooth preparations Gingival margin not on intact enamel

Gingival margin not on intact enamel Composite resin restorations are contraindicated when gingival margins are NOT intact, this is typically because of concerns regarding fluid contamination.It is very difficult to maintain isolation and make a satisfactory bond with etched dentin.It is advised to place an RMGIC (resin-modified glass ionomer cement) liner before placement of resin material if it is necessary to fill it with composite (like in anterior esthetic cases).Indications of composite restorations are: Small- and moderate-sized restorations Conservative tooth preparations Areas where esthetics is important

Which of the following conditions puts patients, especially the elderly, at an increased risk for caries? Gingival recession Erosion Bruxism Attrition

Gingival recession Gingival recession results in exposure of the root surfaces.Root surfaces are less resistant to caries attack as compared to the enamel.The passive movement of gingival tissue occurs apically with the aging process.

Plaque accumulation is directly related to which of the following? Accumulation of calculus Gingivitis severity Herpes outbreak severity Materia alba level Periodontitis severity

Gingivitis severity The severity of gingival inflammation is directly related to the amount of plaque accumulation.Plaque is a collection of bacterial aggregates that cling to the tooth pellicle.When bacterial toxins are released by the plaque, the gingiva becomes irritated, initiating a defensive reaction through an inflammatory response. Materia alba is soft debris on the tooth surface that is easily removed and does NOT cause gingivitis. Calculus is mineralized plaque that does not directly cause gingivitis but is a predisposing factor for plaque buildup.

The gingival fibers are arranged in the following groups:

Ginvodental group Circular group Transsseptal group Gingival fibers consists of type 1 collagen, which provides the tensile strength to the gingival tissue. The gingvodental fibers are embedded in the cementum just at the base of the gingival sulcus. Circular fibers encircle the tooth in a ringlike fashion. The transeptal group form horizontal bundles extend between the cementum of approximating teeth into which they are embedded.

Which technique should be implemented for a patient presenting with rampant caries in an area in which esthetics are NOT considered important to the patient? Glass ionomer without a bevel Resin-modified glass ionomer with a bevel Resin composite without a bevel Resin composite with a bevel

Glass ionomer without a bevel A glass ionomer without placement of a bevel should be used in a patient with rampant caries in an area in which esthetics is NOT a concern.Glass ionomer cement (GIC) imparts an acceptable esthetics to the restorations. In a case of rampant caries, fluoride release and fluoride uptake qualities of GIC have higher significance than esthetic concerns.Beveling of enamel margins is NOT beneficial in this patient because there is no need to blend the color of the restoration to the tooth and it does not provide a better seal.

Which of the following structures is not a structure of the mandibular arch that may affect denture fabrication:

Hamular notch The hauler notch is an important landmark in maxillary denture fabrication, not a landmark for mandibular denture fabrication. It lies distal to the maxillary tuberosity and marks the posterior limit of the maxillary denture The buccal shelf is important as it serves as the stress bearing area of the mandibular arch. The retromolar pad is an area of thick mucosa distal to the last molar and should extend approximately 2/3rds up the length of the pad The retromylohyoid fossa are located medial to the tongue and lateral to the mandible bilaterally. The fossa are typically undercut in relation to each other, so care must be taken to block out these areas in the impressions prior to custom tray or baseplate fabrication The master notch is located in the most distal portion of the buccal aspect of the mandibular arch. This area must be carefully evaluated to prevent overextension of the denture. Overextension could cause discomfort to the patient or dislodging of the denture on opening.

Research has shown that the best solution to place a tooth in after avulsion to increase the periodontal ligament longevity is:

Hank's balanced salt solution Research has shown that the best transport medium for avulsed teeth is cell culture media such as ViaSpan or Hanks balanced salt solution. If used, it can significantly increase the likelihood of the periodontal ligament surviving several hours

Which of the following conditions contraindicates the use of 100% oxygen therapy to treat a patient who suffers from syncope? Hyperventilation syndrome Vasovagal Neurogenic Orthostatic

Hyperventilation syndrome (HVS) is a psychologically or physiologically based respiratory disorder involving a patient breathing too deeply or too rapidly. HVS may present with chest pain and a tingling sensation in the fingertips and around the mouth and may accompany a panic attack. HVS patients have roughly the same oxygenation in the arterial blood (98% for hemoglobin saturation) and insufficient CO2 in their blood and other tissues. Though oxygen is abundant in the bloodstream, HVS reduces effective delivery of that oxygen to vital organs due to low CO2-induced vasoconstriction and the suppressed Bohr effect. HVS is a condition of insufficient CO2, not O2, so additional oxygen is contraindicated.

The most effective way to decrease radiation exposure during radiographic examination involves the use of:

High speed film F-speed film requires only 40% of the exposure of that required for D-speed film. The use of such film provides the greatest reduction in radiation dose to the patient

Wearing gloves in the operatory prevents

Hinders percutaneous absorption of a patient's microorganisms

Each of the following devices are effective in removing surface plaque EXCEPT one. Which is the EXCEPTION? Home water-irrigating device Toothpick Super-soft toothbrush Dental floss

Home water-irrigating device Oral irrigation devices help remove plaque in shallow periodontal pockets. These devices propel a pulsating, high-pressure stream of water to flush subgingival plaque and debris out of shallow pockets. They are ineffective if the periodontal pocket is too deep to be reached by toothbrush bristles or a toothpick. Dental floss is the most recommended oral hygiene method for proximal plaque removal, but it is not capable of reaching deep into the pocket.

The mechanism of action for which of the following antihypertensive drugs includes stimulating arterial smooth muscle cells to vasodilate in order to decrease the overall blood pressure? Hydrochlorothiazide Valsartan Metoprolol Hydralazine

Hydralazine Hydralazine acts to directly relax the vascular smooth muscles in the arteries and arterioles causing vasodilation, which decreases blood pressure. It is not a primary drug in the management of hypertension, because it also stimulates the baroreceptor reflex, increasing the patient's heart rate and cardiac output. Hydralazine is contraindicated for patients with coronary artery disease, because it could trigger angina pectoris or possibly a myocardial infarction. Valsartan is an angiotensin II receptor blocker. Metoprolol is a cardioselective β1-adrenergic blocker. Hydrochlorothiazide (HCTZ) is a diuretic.

Each of the following is considered an adverse effect of hydrochlorothiazide (HCTZ) EXCEPT one. Which is the EXCEPTION?

Hyperkalemia Hydrochlorothiazide (HCTZ) is a thiazide diuretic that inhibits the kidneys' ability to retain water, reducing the blood volume and decreasing the venous blood return to the heart and cardiac output. Patients on HCTZ often require potassium supplements due to the hypokalemia often induced by this medication. Common HCTZ side effects include the following: Hypokalemia (low potassium) Hypomagnesemia (low magnesium) Hyponatremia (low sodium) Hypercalcemia (high calcium) Hyperuricemia leading to gout Hyperglycemia Hyperlipidemia

Each of the following is an effect of histamine EXCEPT one. Which is the EXCEPTION? Activation of C-nerve fibers Hypertension Increased capillary permeability Increased gastric acid secretion Bronchiolar constriction

Hypertension Histamine release causes hypotension due to relaxation of vascular smooth muscle and increased capillary permeability. Stimulation of H1 receptors in bronchiolar smooth muscle causes constriction. Stimulation of H2 receptors stimulates gastric acid secretion. Histamine-selective C-fibers are responsible for creating the "itching" sensation.

Which of the following is the most common drug reaction seen in a cross-reactivity of penicillin with cephalosporin?

Immediate-type hypersensitivity reactions Patients experiencing a penicillin allergy also have a tendency to elicit cross-allergic reactions with cephalosporin drugs. An immediate, or type I, hypersensitivity reaction is a type of rapidly acting allergic response of the body over a certain kind of allergen. An immediate hypersensitivity reaction may result in severe systemic reactions like anaphylaxis, which can cause difficulty in breathing and death. Administration of epinephrine and antihistamines may help slow down and prevent this allergic reaction. Patients demonstrating a rash or hives following penicillin administration is a contraindication for use of cephalosprin antibiotics, but it is not a severe contraindication, and they may still be used in certain situations. Although cephalosporins are mostly cross-reactive with penicillin allergic patients, they do not always cause hypersensitivity. Penicillins demonstrate cross-allergy with first-generation cephalosporins and a negligible cross-allergy with second-generation cephalosporins. Laboratory and cohort studies confirm that the R1 side chain is responsible for this cross-reactivity. Overall cross-reactivity between penicillins and cephalosporins is lower than previously reported (from 10% to 1%), though there is a strong association between amoxicillin and ampicillin with first- and second-generation cephalosporins that share a similar R1 side chain.

Which of the following factors during a free gingival graft will increase the likely success of the graft? Donor tissue containing periosteum Immobilization of the graft at the recipient site Presence of a periosteal fenestration The thickness of the blood clot between the recipient and donor tissue margin

Immobilization of the graft at the recipient site Immobilization of graft enables the formation of blood vessels and better diffusion of nutrients from the recipient site to the graft. Donor tissue can gain attachment to the recipient site as long as the graft is adapted and placed properly at the recipient site. The formation of new capillaries from the recipient site to the donor site will only occur within two to three days after the surgical procedure in the process known as capillary inosculation.

Which mistake should be avoided to prevent production of porcelain that is porous? Inadequate firing temperature Excessive firing temperature Inadequate condensation of porcelain Moisture contamination Improper degassing

Inadequate condensation of porcelain Inadequate condensation of porcelain allows for voids between particles, causing porosities during sintering.Controlled and gentle mechanical condensation of porcelain allows the particles to settle and become closer, eliminating voids.Surface tension condensation allows the drawing of the porcelain particles together by draining the excess fluid from the mixture.

When preparing a partial veneer crown for tooth 8, the proximal grooves are placed parallel to which of the following? Long axis of the tooth Long axis of the clinical crown Incisal two-thirds of the facial axial wall Incisal third of the facial axial wall

Incisal two-thirds of the facial axial wall Placement of the proximal grooves parallel to the incisal two-thirds of the facial surface ensures that the partial veneer crown has only one path of insertion.Proximal grooves and flares should be made continuous with the facial wall to provide retention, resistance, and structural durability.Proximal grooves created in a proper way also relieve the tooth of unsupported enamel.

The most effective treatment for an acute apical abscess can be achieved by:

Incision and drainage Incision and drainage is a surgical opening created in soft tissue for the purpose of releasing exudate or decompressing the area of swelling. This technique will help relieve the pressure that builds up in an acute apical abscess Root canal therapy will remove the source of infection, but the effect is not as immediate as incision and drainage

Which of the following describes the mechanism of action of acetylsalicylic acid (aspirin)? Stimulation of μ-opioid receptors Blocking of sodium channels in nerves Inhibition of cyclooxygenase Inhibition of antithrombin III

Inhibition of cyclooxygenase Cyclooxygenase (COX) is an enzyme involved in the synthesis of prostaglandins including the following: PGE2 PGF2α PGE2 and PGF2α mediate pain within the body. Aspirin inhibits COX-1 and COX-2.

A febrile patient presents with a submandibular fluctuant mass. The abscess is determined to be of odontogenic origin. Which of the following adjunctive therapies should be completed along with pulpectomy and canal debridement? Incision, drainage, and an antibiotic sensitivity test Aspirin to decrease the temperature Administration of penicillin VK to reduce the swelling only Application of heat and cold to the area to improve circulation

Incision, drainage, and an antibiotic sensitivity test The increased number of bacteria built up by the infection resulted in its spread into the submandibular space, causing the patient's fever. Incision and drainage allow the aggregated infection and pressure that build up within the submandibular space to be relieved. Antibiotic sensitivity testing helps determine the putative etiologic bacteria and guide the clinician to the appropriate antibiotic treatment.

When fabricating a complete denture, which of the following anatomical structures is used as a guideline to provide the most esthetic result for the anterior-posterior location of the maxillary anterior teeth? Middle of the nasal cartilage Anterior edentulous ridge Incisive papilla Vibrating line Anterior labial frenum

Incisive papilla The incisive papilla is a stable landmark that does not change its position within the palate.The incisive papilla is used as a guide in placing and positioning of the maxillary anterior teeth during denture tooth placement.The incisive papilla is expected to be lingually positioned in between the maxillary central incisors.The most labial surface of the maxillary central incisor denture tooth should be placed approximately 8-10mm anterior to the incisive papilla.

An edentulous patient appears to have wrinkles around the corners of the mouth, angular cheilitis, and lack of a vermilion border. While fabricating a complete denture, which of the following should be done to correct this? Decrease the vertical dimension of the occlusion. Increase the vertical dimension of the occlusion. Increase the interocclusal distance. Move the anterior teeth facially. Use bilateral balanced occlusion.

Increase the vertical dimension of the occlusion. Unsupported lips and absence of vermilion border, together with a relatively short lower one-third of the face, is indicative of an incorrect vertical dimension of occlusion.Presence of labial commissure and deep labiomental groove are also indications of a short, and incorrect, vertical dimension of occlusion.Increasing the vertical dimension of occlusion can help achieve adequate lip support, an esthetically pleasing and properly proportioned facial profile, and satisfactory phonetics.Too high of a vertical dimension of occlusion may cause straining of the muscles of mastication and facial pain.

Ethanol ingestion results in each of the following EXCEPT one. Which is the EXCEPTION?

Increased gastric acid secretion Ethanol ingestion causes decreased gastric acid secretion. Ethanol ingestion causes blurry vision and delays in pupil reaction time. Pupil constriction occurs when the sphincter pupillae muscles elicit a delayed reaction to stimuli when there is a depression of the sympathetic nervous response brought about by excessive consumption of ethanol. Ethanol consumption makes a person less responsive to surroundings. A person under the influence of ethanol experiences central nervous system (CNS) depression and diuresis.

Each of the following is a biologic change that occurs with advancing age and affects the pharmacodynamics and pharmacokinetics of pharmaceuticals EXCEPT one. Which is the EXCEPTION?

Increased plasma protein binding Decreased renal excretion occurs as people age, causing increased longevity inside the body for agents that are renally excreted. Some drugs may have increased half-lives due to reduced functionality of organs metabolizing the drug, such as the liver, as we age. A decreasing rate of biotransformation also occurs as we get older because of the inadequate production of enzymes that activate the drug.

If a flap is created to assist in removal of a partial. Bony impacted third molar, it would be necessary to reposition the flap after the procedure is complete. Failure to do this will result in:

Increased post operative pain Failure of wound closure by primary intention Increased wound dehiscence Flap closure does not necessarily affect the development of bone necrosis. This is more directly impacted by blood flow to the bone and pre-existing risk factors such as exposure to bisphosphanates or other osteoporosis cancer medications that impact bone metabolism

Which of the following physiologic effects can occur after the administration of a cholinomimetic drug?

Increased salivation Stimulation of nicotinic or muscarinic receptors can initiate and increase salivation. Cholinomimetic drugs are designed to directly or indirectly inhibit acetylcholinesterase. Cholinomimetic drugs are similar in action to acetylcholine and can cause stimulation of the nicotinic and muscarinic receptors.

When mixing water and powder to pour an impression with dental stone, what would the effect be of increasing the water to powder ratio?

Increased setting time As the water to powder ratio increases, the setting time increases, the strength of the gypsum product decreases, and the setting expansion decreases.

The best way to produce a radiographic image with low contrast is by doing which of the following? Increasing the kilovoltage peak (kVp) Increasing the milliamperage (mA) Increasing the focal spot-skin distance Decreasing the filtration

Increasing the kilovoltage peak (kVp) Longer gray scales of contrast can be achieved by increasing the kilovoltage peak.Increasing the kilovoltage causes the radiographic image to have low contrast and longer gray scales.Low kilovoltage creates low energy x-rays responsible for producing high-contrast and shorter gray scale.

Which of the following is an antihistamine that can also be injected as local anesthetic solution if a patient reports being allergic to ester and amide anesthetics?

Injectable 1% diphenhydramine hydrochloride can be used as a safe and effective local anesthetic for patients having allergic reactions from an ester or amide derivative anesthetic. Diphenhydramine can be applied topically as an antipruritic and anesthetic drug. Its ability to provide local anesthesia is probably because of its chemical structure, which is similar to some of the known local anesthetic drugs.

Dextroamphetamine belongs to which of the following drug classes?

Indirect sympathomimetics Indirect sympathomimetic drugs like ephedrine, amphetamine, and cocaine work by blocking the norepinephrine transporter, inhibiting the reuptake of norepinephrine. Indirect-acting sympathomimetic drugs like monoamine oxidase inhibitors (MAOIs), reuptake inhibitors, and release stimulants can increase the amount of endogenous catecholamines in the body. These drugs have been used to treat low blood pressure, cardiac arrest, and delaying premature labor. Direct-acting[edit] Adrenergic receptor agonists[edit] Main article: Adrenergic agonist Direct stimulation of the α- and β-adrenergic receptors can produce sympathomimetic effects. Salbutamol is a widely used direct-acting β2-agonist. Other examples include phenylephrine, isoproterenol, and dobutamine. Indirect-acting[edit] Main article: Monoamine releasing agent Dopaminergic stimulants such as amphetamine, ephedrine, and propylhexedrine work by causing the release of dopamine and norepinephrine, along with (in some cases) blocking the reuptake of these neurotransmitters. The mechanism of action of amphetamine is complemented by the inhibition of the reuptake and of monoamine oxidase which acts synergistically to produce a significant increase the monoamine concentration.1 This activity is not done as an inhibitor per se but more as a competitive substrate and thus, amphetamine is known to be a weak dopamine reuptake inhibitor, moderate noradrenaline reuptake inhibitor and very weak serotonin reuptake inhibitor. From this specific action, the l-isomer is known to be significantly less potent.5

Blood dyscrasias are most commonly caused by which of the following nonsteroidal anti-inflammatory drugs (NSAIDs)? Ketorolac Indomethacin Ibuprofen Aspirin Acetaminophen

Indomethacin Blood dyscrasias are any disorder or disease of the blood, like leukopenia, aplastic anemia, and neutropenia. They are rarely caused by NSAIDs but are sometimes caused by the use of indomethacin. Indomethacin is an NSAID that provides anti-inflammatory, antipyretic, and analgesic effects. It is proven to be useful in managing conditions like chronic arthritis. The use of indomethacin is limited due to its ability to inhibit the aggregation of platelets. This inhibition of the aggregation of platelets can cause tendencies of increased bleeding time. Therefore, utmost caution must be observed when indomethacin is used for patients suffering with coagulation defect.

Which of the following is the mechanism of action by which epinephrine blocks the effects of histamine?

Induction of physiologic effects opposite to histamine The physiologic response of the body to epinephrine is the opposite response to the effects of histamine release. Histamine release causes contraction of the smooth muscle of the lungs, bronchoconstriction, swelling, difficulty breathing, and anaphylaxis. Adrenaline release or epinephrine administration produces bronchodilation, blood vessel constriction, and an increase in cardiac output of the heart. Epinephrine is the best drug for managing anaphylaxis due to its ability to counteract the effects that cause anaphylaxis.

A dentist provides treatment of an examination, radiographs and hygiene therapy for a new patient. The fees are billed through to the insurance company, and 80% of the fees are covered. The dentist then decides not to collect the remaining 20 percent from the patient, as a nice gesture for a new patient. What best describes this situation?

Insurance fraud and unethical behavior It is unethical to not collect the remaining fees once insurance has paid their percentage. In this case when treatment cost $100, insurance would cover 80% 0r $80 of treatment. If that were the only fee collected, then technically the $80 would 100% of the fee. Therefore, the insurance should have paid only 80% of the $80 ($64) [A professional courtesy would explain this discrepancy to the insurance company)

Which of the following interdental aids would be best for the patient to utilize to effectively remove plaque from the interproximal embrasure spaces of their periodontally involved mandibular anterior teeth that lack defined papillae? Interdental brush Superfloss Dental floss Dental tape

Interdental brush Interdental (interproximal) brushes provide excellent access to root concavities, furcation areas, and proximal surfaces where papillae do not fill the interdental spaces. Dental floss and dental tape are most commonly used for situations where there is still a papilla. Superfloss is indicated to clean underneath fixed partial dentures. Floss picks are indicated for patients that still have a papilla but are not coordinated enough to use standard floss or dental tape.

A patient presents to the office with pain associated with the right preauricular region of the face. The patient has a maximum opening of 45 mm and presents with a "joint click" associated with the right mandibular condyle. Of the following, which is the most likely diagnosis? Myofascial pain dysfunction syndrome Coronoid hyperplasia Internal derangement with reduction Auriculotemporal syndrome

Internal derangement with reduction Temporomandibular dysfunctions occur with many variations, the most common being internal derangement. Internal derangement is a condition within the temporomandibular joint that involves displacement or slipping of the articular disc. Internal derangement with the presence of reduction denotes that the articular disc is moving and sliding in and out of its normal limiting structure as the condyle head functions during the jaw opening and closing movements. The movement of the condyle against an articular disc that slides back and forth causes the clicking sound that characterizes a temporomandibular joint disorder. In severe conditions, patients may experience altered or limited mouth opening and locked jaw.

In which of the following locations does new plaque principally accumulate after patients brush their teeth? Occlusal Lingual Interproximal Buccal

Interproximal Newly formed plaque may form on all tooth surfaces, but it typically stays longer in areas rarely disturbed by oral hygiene practices. It usually accumulates along areas toothbrushes hardly reach (e.g., the interproximal area). Plaque may form along the buccal and lingual aspects, but proper brushing easily cleans these areas. The occlusal area may demonstrate some plaque accumulation, but plaque there is usually dislodged during mastication and brushing.

Which type of suture is indicated in an immediate denture case to promote the best wound healing for the alveolar ridge? No use of sutures Vertical mattress Horizontal mattress Continuous Interrupted

Interrupted Explanation Interrupted sutures are more stable and provide better adaptation of the wound edges.Interrupted sutures are placed individually.Continuous sutures pose a greater possibility of coming loose under frictional and occlusal forces from the tissue side of immediate dentures.If continuous sutures unwind and become loose, the stabilization of the wound is altered.Disturbing the closure of the wound could result in the reopening of the wound.

Which of the following drugs is most potent against a Mycobacterium tuberculosis infection? Methicillin Penicillin VK Isoniazid Vancomycin

Isoniazid Isoniazid is one of the most potent and most common antibiotics prescribed to treat Mycobacterium tuberculosis infections, for the following reasons: It is very effective against actively multiplying bacteria. It can be prescribed alone or co-administered with other antibiotics. Isoniazid treatment takes six to twelve months to completely eradicate tuberculosis. Isoniazid is usually administered orally or intramuscularly.

Which of the following drugs is a β-adrenergic agent that produces the greatest bronchodilation effect?

Isoproterenol Amphetamines are psychostimulant drugs that stimulate the central nervous system (CNS). Their overuse can lead to addiction. Methoxamine is a sympathomimetic used for the management of hypotension due to its vasopressive activity. Norepinephrine is a catecholamine that has vasopressive properties and is used to raise blood pressure to normal levels in cases of acute hypotension. Phenylephrine is an adrenergic drug with strong vasoconstrictive properties. It is usually used as a nasal decongestant. Isoproterenol is a sympathomimetic that is similar to epinephrine in its cardiac excitatory action but different from epinephrine in its vasodilation property. It is a potent bronchodilator when used in sulfate and hydrochloride forms. Isoprenaline is a β1 and β2 adrenoreceptor agonist and has almost no activity on alpha adrenergic receptors.[3] Its agonist effects at TAAR1 provide it with pharmacodynamic effects that resemble those of the endogenous trace amines, like tyramine.[7] Isoprenaline's effects on the cardiovascular system (non-selective) relate to its actions on cardiac β1 receptors and β2 receptors on smooth muscle within the tunica media of arterioles. Isoprenaline has positive inotropic and chronotropic effects on the heart. β2 adrenoceptor stimulation in arteriolar smooth muscle induces vasodilation. Its inotropic and chronotropic effects elevate systolic blood pressure, while its vasodilatory effects tend to lower diastolic blood pressure. The overall effect is to decrease mean arterial pressure due to the β2 receptors' vasodilation.[8] The isopropylamine group in isoprenaline makes it selective for β receptors. The free catechol hydroxyl groups keep it susceptible to enzymatic metabolism.[9]

Which of the following is true regarding erythema migrans?

It appears on the anterior 2/3rds of the tongue Erythema migrans is a common benign condition that primarily affects the tongue. It is often detected on routine examination of the oral mucosa and occurs more in females at a 2:1 ratio. The characteristic lesions of erythema migrans are seen on the anterior 2/3rds of the dorsal tongue mucosa. They appear as multiple, well-demarcated zones of erythema, concentrated at the tip and lateral borders of the tongue

Which of the following properties of erythromycin explains its propensity for interaction with other drugs?

It decreases metabolism of drugs that interact with cytochrome P450. Erythromycin tends to inhibit metabolism of drugs within the liver through cytochrome P450 inactivation. It stops the enzymatic reaction for other drugs, making their metabolism in the liver slower, leading to possible increased toxicity of other drugs. Toxic megacolon is a side effect of inflammatory bowel diseases like Crohn disease and ulcerative colitis, which prevents pharmaceutical absorption. Erythromycin and clarithromycin inhibit cytochrome P450 enzymes, and have been implicated in clinically significant interactions.

Which is the primary reason for administration of prednisolone during an operative procedure? All answers are correct. It reverses an already inflamed pulp. It decreases the likelihood of pulpal inflammation. It serves as a hemostatic agent.

It decreases the likelihood of pulpal inflammation. Prednisolone can decrease pain stimuli by reducing the production of prostaglandins responsible for the mediation of pain and inflammation.Prednisolone is an anti-inflammatory steroid drug that functions as an effective inhibitor of phospholipase A2, which is responsible for promoting inflammation.Prednisolone has the ability to down-regulate some pro-inflammatory cytokines and reduce the pulpal inflammation during operative procedures.

Which of the following does NOT describe tramadol (Ultram®)? It is metabolized into a more active form. It binds to μ-opioid receptor. It inhibits reuptake of norepinephrine. It inhibits reuptake of serotonin. It is a peripherally acting analgesic.

It is a peripherally acting analgesic. Tramadol is a centrally acting synthetic analgesic. It is indicated as a pain reliever for moderate-to-severe pain. Tramadol increases the release of serotonin as well as inhibits the reuptake of norepinephrine and serotonin, thereby modifying the ascending pain pathway. Tramadol and codeine are both metabolized in the liver following the same hepatic pathway. About featured snippets• Feedback People also ask What is tramadol mechanism of action? Tramadol is a centrally acting analgesic with a multimode of action. It acts on serotonergic and noradrenergic nociception, while its metabolite O-desmethyltramadol acts on the µ-opioid receptor. Its analgesic potency is claimed to be about one tenth that of morphine.Jun 16, 2014 Tramadol - WHO | World Health Organization https://www.who.int › quality_safety › 6_1_Update PDF Search for: What is tramadol mechanism of action? Is tramadol an SSRI or SNRI? Is tramadol an opioid or Nonopioid? Tramadol is now available in more than 100 countries and is the most commonly prescribed opioid worldwide 4. In the US, tramadol prescriptions more than doubled between 2007 and 2015, becoming the second most prescribed opioid. Tramadol modulates the descending pain pathways within the central nervous system through the binding of parent and M1 metabolite to μ-opioid receptors and the weak inhibition of the reuptake of norepinephrine and serotonin.

Which statement describes the lincosamide clindamycin? It is effective against Gram-negative bacteria only. It does not penetrate well into bony tissue. It is usually given in combination with erythromycin. It inhibits cell wall synthesis. It is effective against most anaerobes.

It is effective against most anaerobes . Clindamycin is a lincosamide antibiotic used to treat anaerobic bacterial infections, and its use is associated with pseudomembranous colitis. Clindamycin is prescribed to treat anaerobic bacterial infections and some protozoal diseases, such as malaria, and is also a common topical treatment for acne. The potency of oral clindamycin leads to widespread loss of normal gut flora, allowing Clostridium difficile to take over the gut, causing pseudomembranous colitis. Clindamycin may prolong the effects of neuromuscular-blocking drugs such as succinylcholine.

Each of the following statements describes biotransformation EXCEPT one. Which is the EXCEPTION? It generally involves modifications of the chemical structure of the drug. It principally occurs in the microsomal enzyme system of the liver. It usually converts a drug into its lipid-soluble, non-ionized form. The rate may vary significantly between species.

It usually converts a drug into its lipid-soluble, non-ionized form. Biotransformation involves the conversion of a lipid-soluble, non-ionized form of a drug into its more water-soluble metabolites. It increases the absorption of a drug into the bloodstream, enhancing the bioavailability of the drug. It also renders non-polar compounds polar so they are not reabsorbed in the renal tubules and thus excreted.

What type of seizure is classified as a partial seizure?

Jacksonian Partial seizures by definition only involve one side of the brain at the time of onset. There are three general types of partial seizures: simple, complex, or partial evolving into generalized seizures. One type of partial seizure is the Jacksonian seizure. In this type of seizure, consciousness is not impaired. [Generalized seizures involve both sides of the brain. There are many types of generalized seizures including: Tonic clonic Absence: impaired consciousness Myoclonic: sudden, brief muscle contractions Atonic: sudden loss of muscle tonic] A Jacksonian seizure is a type of focal partial seizure, also known as a simple partial seizure. This means the seizure is caused by unusual electrical activity that affects only a small area of the brain. The person maintains awareness during the seizure. Jacksonian seizures are also known as a Jacksonian march.

Which of the following analgesics can be given orally or by intramuscular (IM) injection? Ibuprofen (Motrin®, Advil®) Acetaminophen (Paracetamol®, Tylenol®) Ketorolac (Toradol®) Naproxen sodium (Aleve®) Lidocaine (Xylocaine®)

Ketorolac (Toradol®) Ketorolac is an anti-inflammatory that can be administered through intravenous, intramuscular, and oral routes. It is primarily used to relieve moderate levels of pain and prescribed as a postoperative pain reliever or after any other painful procedure. The administration of a ketorolac injection together with ketorolac tablets should not exceed or go beyond five days to avoid the increased risk of experiencing severe adverse effects.

Which of the following is affected by taking expired tetracyclines, an effect known as the Fanconi syndrome? Pancreas Kidney Brain Lungs Heart

Kidney Taking expired tetracyclines results in damage to the proximal tubule, resulting in renal tubular acidosis and loss of the following in urine: Glucose Amino acids Excessive phosphate and bicarbonate

Which of the following characterizes the therapeutic index (TI) of pharmaceutical agents? ED50/LD50 ED1/LD99 LD99/ED1 LD50/ED50 LD99/LD50

LD50/ED50 The therapeutic index is a numerical comparison of a drug's therapeutic effect over its toxic or lethal side effect. The therapeutic index is measured by dividing the drug's lethal dose, or the dose in which it produces toxic effects, over its median effective dose. (LD50 ÷ ED50) Drugs having a low therapeutic index provide a narrow margin of safety, which means the drug may pose a great risk compared to its medical value. The median lethal dose (LD50) is the amount of an ingested substance that kills 50% of a test sample. The median effective dose (ED50) is the dose of a drug that produces a measurable effect in 50% of the people who take the drug. 0 CommentsSort by

One of the major contraindications to use of composite resin as a restorative material is:

Lack of adequate isolation Operator factors-lack of attention to detail, technical ability, and knowledge of the material's uses and limitation are also major contraindications to use composite resin material

Which of the following muscles move the mandibular condylar head?

Lateral pterygoid The lateral or external pterygoid muscle is responsible for the movement of the condyle. The muscle itself consists of an upper and lower head. These two heads move both the condyle and articular disc and provide a variety of movements dependent on either contraction of a singular head or both simultaneously

Accidental radiation exposure is prevented by covering x-ray film with which of the following materials? No correct answer Lead Europium Cardboard Plastic Aluminum

Lead X-ray film manufacturers use lead foil to prevent film from being accidentally exposed to radiation before it is used.Plastic, aluminum, and cardboard do not block radiation enough to be considered for this task.

A 40 year old woman was seen by a dentist for a regular check up. Upon a clinical examination, the dentist noticed a spider web like appearance of white lines present bilaterally in her inner cheeks. The woman stated it was not painful nor bothersome. Which of the following diagnosis most likely fits with this patient's condition?

Lichen Planus Lichen Plans is a disease of the mucous membrane and skin that occurs most often in women in their 30s and 60s and is due to an autoimmune process of unknown cause. A distinct feature of this disease is a white keeratoic strain. There are many types of this condition (place form, bullous, reticular-spider web, etc.)

Which of the following agents can penetrate mucosal tissues up to 3 mm, allowing it to function as a topical anesthetic?

Lidocaine is proven to be an effective topical anesthetic drug that can effectively numb the mucosal surfaces of the oral cavity. It is a strong anesthetic drug usually delivered in gel, liquid, lotion, spray, or cream form. Lidocaine topical anesthetic can penetrate approximately 2-3 mm deep into the mucosal layer when applied properly.

What is the magnitude and duration of force that equilibrates the soft tissue pressure of the lip, cheek, and tongue against the teeth during speaking?

Light force and very short duration [during mastication very heavy and very short]

During fabrication of a complete denture, the coronoid process affects the final denture in which way? Affect the position and arrangement of posterior teeth Determine the location of the palatal seal of the maxillary denture Limit the amount of distal extension of the mandibular denture Limit the thickness of the denture flange in the maxillary buccal vestibule

Limit the thickness of the denture flange in the maxillary buccal vestibule The thickness of the denture flange in the maxillary buccal vestibule needs to be reduced due to the presence of the coronoid process of mandible.This is required for the stability and retention of the denture in the edentulous maxilla. Dislodgement of maxillary denture may result otherwise.

Which of the following actions is NOT performed by a magnetostrictive ultrasonic instrument? Lavage Cavitation Vibration Linear movement Acoustic turbulence

Linear movement Magnetostrictive ultrasonic instruments (Cavitrons) are used to remove heavy deposits of calculus and stain via an elliptical motion. Cavitrons work by doing the following: Lavage: flushes the pocket Cavitation: ultrasonic movement creates air bubbles that knock deposits off tooth structure Vibration: mechanically removes deposits and debris Acoustic turbulence: agitation observed in fluids by mechanical vibrations that disrupts bacterial cell walls Piezo instruments use a linear movement to create a back-and-forth motion to create their ultrasonic movement.

Radiographically, the areas of acute supportive osteomyelitis may show which of the following characteristics?

Little radiographic change Acute supportive osteomyelitis shows little to no radiographic change since this condition is acute in nature. Radiographic changes take time to occur since they must follow the real destruction of bone, which takes at least 10-12 days to be apparent on a radiograph [Increased radiolucency develops during the course of chronic osteomyelitis as the body's inflammatory response resorbs bone.]

Which of the following is a second generation antihistamine?

Loaratidine Loratidine îs a second generation H1 antihistamine. Compared to first generation antihistamines, it does not cross the blood brain barrier, and hence is not sedating First generation antihistamine [promethazine Hydroxyzine Meclizine Diphenhydramine ] The primary mechanism of antihistamine action in the treatment of allergic diseases is believed to be competitive antagonism of histamine binding to cellular receptors (specifically, the H1-receptors), which are present on nerve endings, smooth muscles, and glandular cells.

Each of the following is commonly associated with acute suppurative osteomyelitis EXCEPT one. Which is the EXCEPTION? Loose teeth with suppurative drainage from the periodontium High or intermittent fever Paresthesia or anesthesia of the inferior alveolar nerve Deep and intense pain

Loose teeth with suppurative drainage from the periodontium Loose teeth with suppurative drainage from the periodontium is caused by periodontitis, not acute osteomyelitis. That said, loosening of teeth may develop as the condition progresses from an acute to a chronic phase. Acute osteomyelitis mostly affects children and is usually correlated with a Staphylococcus aureus infection. It may also occur in adults with a compromised immune system or undergoing immunosuppressive or steroid therapy.

Which of the following results in the need for repeated adjustments in the denture when the mouth is in centric relation? Allergy to the relining material Decrease in vertical dimension of occlusion Loss of even contacts in centric relation Presence of a pressure spot in the reline impression

Loss of even contacts in centric relation Loss of even contacts accounts for the need of repeated adjustments in the denture when the mouth is in centric relation.Erythematous areas result in high-pressure spots, making the denture uncomfortable for the patient.For comfortable seating of dthe enture, even contacts are desired in the centric relation.Centric relation is the mandibular jaw position in which the head of the condyle is situated as far anterior and superior as it possibly can within the mandibular fossa/glenoid fossa and is the most retruded position of the mandible.

Tooth #30 demonstrates a class II furcation involvement. Which of the following best describes this condition? Evidence of bone and tissue loss in the furcal area, with entrance clearly visible, and a Nabers probe can pass between the roots through the entire furca Evidence of early bone loss, where a Nabers probe can enter the depression leading to the furcation Severe bone loss in the furcal area, where there is through-and-through involvement with complete loss of bone between the roots, but gingiva is covering the area Loss of some bone between the roots, and a Nabers probe can enter the furcal area but cannot pass between the roots

Loss of some bone between the roots, and a Nabers probe can enter the furcal area but cannot pass between the roots Explanation When loss of attachment extends into the bifurcation or trifurcation, the furcation is classified by degree of involvement. The Glickman furcation classifications are as follows: Class I: Evidence of early bone loss in the furcal area. and a Nabers probe enters the depression, but bone remains between the roots Class II: Evidence of moderate bone loss in the furcal area, and a Nabers probe can enter the furca but cannot pass between the roots Class III: Evidence of severe bone loss in the furcal area, and a Nabers probe can pass between the roots through the entire furca, but gingiva is still covering the opening Class IV: Evidence of severe bone and soft tissue loss in the furcal area, where there is through-and-through involvement with complete loss of bone, and a Nabers probe can pass between the roots through the entire furcation with the entrance clearly visible

Which of the following is NOT a pathological outcome of smokeless tabacoo?

Lung cancer Because smokeless tobacco exerts its effects by coming into direct contact with the oral mucosal membrane and does not need to be inhaled or smoke. It is the inhalation of the cariogenic smoke that increases the potential for lung cancer Smokeless tobacco use can cause leukoplakia, gingival recession, and cardiovascular disease.

This patient notes that she is allergic to latex. Which of the following cells are most active in the histamine allergic reaction? Monocyte Plasma cell Lymphocyte Mast cell

Mast cell The mast cell is the most active during a histamine allergic immune reaction. Mast cells contain histamine and heparin, which they are able to secrete to affect the activity of the body in an immune reaction and which are known for allergy and anaphylaxis. Monocytes are a subtype of white blood cells (leukocytes) and are the largest of all leukocytes. They are considered part of the innate immune system. Monocytes are also responsible for phagocytosis. Lymphocytes are cells of the immune system that include the following: Natural killer cells (NK cells) function in cell-mediated, cytotoxic innate immunity. They recognize stressed cells in the absence of antibodies and major histocompatibility complex (MHC) and do not require activation to kill cells that are missing MHC I "self" markers. T cells for cell-mediated, cytotoxic adaptive immunity serve to activate other immune cells, kill other cells, and shut down an immune response. B cells produce antibodies for humoral antibody-driven adaptive immunity. Plasma cells are a type of white blood cell that secrete large volumes of antibodies.

Which of the following teeth is most resistant to rotational forces during extraction? Mandibular first premolar Mandibular second premolar Maxillary first premolar Maxillary central incisor Mandibular canine

Maxillary first premolar The buccal and lingual placement of the roots and the tendency to fracture decreases the usefulness of rotational extraction forces. Maxillary first premolars are known to fracture during extractions because they have two thin roots. These premolars have variable root forms and normally have two root canals and two roots. Due to the thinness of the tooth root of maxillary premolars, the roots are easily fractured during extraction.

Which of the following is the predominant etiology of fibrous gingival hyperplasia? Puberty Pregnancy Medication Leukemia Diabetes

Medication Fibrous gingival hyperplasia is most commonly caused by medications such as phenytoin, cyclosporin, and nifedipine.These three drugs are known to be anticonvulsive, immunosuppressive, and calcium channel blockers respectively.There is no documentation at present that proves that being diabetic, pregnant, or going through puberty or leukemia may cause fibrous gingival hyperplasia. Calcium channel blockers include amlodipine, verapamil, and diltiazem.

Short duration local anesthetics include

Mepivacaine HCL 3% Prilocaine HCL 4% by infiltration Short acting anesthetics by definition provide plural anesthetic for less than 30 minutes. (Using the same specifications of prilocaine as a nerve block would produce intermediate duration anesthetia. Intermediate anesthetics lasts for approximately 60 minutes and include Lidocaine HCL 2% + 1:100,000 epinephrine Mepivacaine 2% +1:20,000 levonodefrin

In which of the following directions would a mandibular second molar move if the first molar were extracted? Mesially Lingually Distally Buccally

Mesially Teeth tend to move into an edentulous area recently occupied by an adjacent tooth. Edentulous spaces exhibit thinner bone support, so the adjacent second molar may tilt or move mesially into the space.

Which is the term for when different types of light sources make colors appear different? Translucency Metamerism Opalescence Fluorescence

Metamerism Metamerism is the phenomenon where colors appear similar in a particular light source but appear differently when observed with another light source.Fluorescence is when light absorbed by a substance is emitted back in a longer wavelength.Opalescence is the condition in which a material appears yellowish-red when light is transmitted through it. However, the material appears blue in the scattered light that is perpendicular to the transmitted light.Translucency occurs when light is permitted to pass through a material but is diffused within the material, making the image on the opposite side not clearly visible.

Each of the penicillin class drugs can be administered orally EXCEPT one. Which is the EXCEPTION?

Methicillin Methicillin is a narrow-spectrum parenteral beta-lactam antibiotic agent of the penicillin class used to treat infections caused by bacteria beta-lactamase-producing Gram-positive organisms. Methicillin has been replaced by flucloxacillin and dicloxacillin due to its high frequency of adverse effects. The term "methicillin-resistant Staphylococcus aureus (MRSA)" is used to describe Staphylococcus aureus strains resistant to all penicillins. Methicillin required administration via intramuscular or intravenous injection because it was inactivated by gastric acid in the stomach when taken orally. Side effects associated with its use included diarrhea and allergic reactions, such as skin rash and anaphylaxis.

A patient presents with a loss of sensation to the left half of the chin and lip. You would be concerned about:

Metstatic malignancy of the mandible Metastatic malignancy of the mandible that involves the inferior alveolar nerve can produce a distinct pattern of symptoms called "numb chin syndrome. Basically there is an unexplained loss of sensation to the lip and chin. This pattern of paresthesia may also occur in primary malignancy of the jaws as well. Other symptoms that may be experienced are pain, loosening of teeth, and other paresthethias. [Bell's palsy is an idiopathic (unknown cause) paralysis of cranial nerve VII. The facial nerve does not provide sensation to the lip or chin area]

Which of the following represents an effective treatment for mottled enamel (chalky white color mixed with brown spots)?

Microabrasion with HCL/pumice Defects in tooth formation are confined to the enamel and are either hypo calcified or hypo plastic. Enamel hypo calcification is common, appearing as a distinct brownish or whitish area on the facial aspect of the crown. Both the whitish and the brownish spots are amenable, (responsive) to bleaching with pumice, (a light volcanic rock) and acid [Sodium perborate is also present in some tooth bleaching formulas for non-vital root treated teeth. The compound is inserted in the root canal and left in place for an extended period of time to allow it to diffuse into the tooth and bleach stains from the inside out

Which is the most accepted treatment for mottled enamel (Colorado brown stains/hyperfluorosis/chalky enamel/white enamel with brown spots)? Microabrasion with hydrochloric acid (HCl)/pumice At-home vital tooth bleaching Walking bleach after root canal treatment ln-office bleaching with 35% hydrogen peroxide

Microabrasion with hydrochloric acid (HCl)/pumice Mottled enamel is a condition in which a combination of small white, brown, and yellow spots are seen all over the surfaces of the teeth. Microabrasion is a fast, pain-free treatment that is effective in lightening or completely removing the yellow, white, and brown stains seen on mottled enamel. Microabrasion removes a thin layer of stained enamel with the use of pumice and hydrochloric acid. Hydrochloric acid whitens the stains in the tooth surface and has an effect that lasts for a few days after its application.

Which material will provide the most esthetic restoration for a class V preparation on an anterior tooth? Glass ionomer Microfilled resin composite Resin-based glass ionomer Hybrid resin composite

Microfilled resin composite A class V preparation should be restored with microfilled resin composite material whenever esthetics is the main concern.Because of their small particle size, microfilled resin composites have a smooth, highly polishable surface which requires minimal finishing.The low elastic modulus of these materials allows them to flex during tooth flexure, better protecting the bonding interface in class V restorations.Microfilled resins are NOT typically used for restorations that are going to be in contact with other teeth because they have a low resistance to wear. 1 CommentSort by

Post-operant sensitivity is somewhat common after the placement of a composite resin restoration. What is the most likely cause of this sensitivity?

Microleakage Microleakage occurs in these restorations often from polymerization shrinkage of composite resin in improperly placed or layered fillings. This shrinkage creates tiny openings through which oral fluids enter into the tooth and access the dentin-pulp complex. The movement of this fluid is what causes the pain. Do not leave etchant on longer than the recommended time and pain will not occur. Same with light curing

Where do children primarily acquire Streptococcus mutant?

Mouth to mouth transmission from their mothers The succesful transmission to the infant is also related to the size of the inoculum, the frequency of the inoculations, and the size of the ineffective dose

Which of the following antibiotics is considered a third-generation cephalosporin? Cefamandole Clindamycin Moxalactam Cephalexin Amikacin

Moxalactam is a third-generation cephalosporin that has been associated with prolonged bleeding time and several cases of coagulopathy. Moxalactam is no longer marketed in the United States. Other third-generation cephalosporins include the following: Cefcapene Cefdinir Cefixime

The supraspinal analgesic action of morphine is achieved by binding to which of the following receptors? Delta Epsilon Kappa Mu Sigma

Mu Delta (δ), kappa (κ), and mu (μ) are the three basic opioid receptors, each of which has a different mode of action. Mu-receptor activation creates the following: Sedation, itching, analgesia, and euphoria Decreased respiration, constricted pupils or miosis, and reduced blood pressure Decreased bowel movement that eventually leads to constipation Delta receptors are responsible for spinal anaesthesia.

Which of the following is a common lesion of the oral mucosa that results from the rupture of a salivary gland duct and spillage of mucin into the surrounding soft tissues?

Mucocele The spillage of mucin from a mucocele is often the result of local trauma, although there is no known history of trauma in many cases

High doses of cholinergic agents often induce each of the following effects EXCEPT one. Which is the EXCEPTION? Hypersalivation Bradycardia Mydriasis Polydipsia Diaphoresis

Mydriasis Mydriasis is a sympathetic reaction of the pupil, causing it to dilate and allow more light to pass through the eye. In a cholinergic reaction, the parasympathetic nervous system is stimulated, causing an increase in the amount of normal secretions like saliva, tears, and acids in the stomach. The cholinergic effect can also cause a slow heartbeat or low heart rate, also known as bradycardia.

Identify the causes of nonodontogenic pain that may present as a maxillary toothache

Nasal mucosa pain Sinusitis Neurovascular pain Neuralgia Nonodontogenic pain may confuse or frustrate a dentist's attempts to alleviate a patient's pain. Careful consideration and evaluation of a patient's oral condition may rule out odontogenic causes of pain, therefore it is important to be aware of the causes of pain that mimic tooth pain, but are in fact musculoskeletal, neuromuscular, or neuropathic in origin Nasal mucosa pain or pain from the maxillary sinuses may both be referred pain from the maxillary dentition due to the proximity of structures to the tooth apices that may mimic a toothache.

A 41-year-old female reports developing an asymptomatic, rapidly expanding ulcer on her right lateral hard palate that appears crater-like and measures 20 mm in diameter. Which of the following is the most likely diagnosis? Necrotizing sialometaplasia Squamous cell carcinoma Actinomycosis Pleomorphic adenoma Adenoid cystic carcinoma

Necrotizing sialometaplasia Necrotizing sialometaplasia (NS) is a rapidly expanding ulcerative lesion that mostly occurs on the posterior area of the hard palate. NS is usually benign, often painless, and usually self-limiting and resolves in about 6 to 10 weeks. Trauma ischemia caused by trauma to the minor salivary glands located in the posterior hard palate is thought to be the most common cause. Etiologies for NS include the following: Trauma (for instance, during intubation or surgical procedures) Local anesthetic injection Smoking Alcohol Diabetes mellitus Vascular disease (such as arteriosclerosis) Pressure from a dental prosthesis Allergy

How does neostigmine differ from physostigmine?

Neostigmine has an additional direct effect at the neuromuscular junction. Neostigmine is a synthetic drug that is poorly absorbed when taken orally and is useful for managing myasthenia gravis. Physostigmine, unlike neostigmine, is a naturally occurring lipid-soluble drug that is absorbed very well when taken orally. Neostigmine can also directly affect neuromuscular junctions to overcome non-depolarizing neuromuscular blockades

Which cell is least associated with the chronic periodontitis? All answers apply B Lymphocyte Plasma cell T Lymphocyte Neutrophil

Neutrophil Neutrophils (PMNs) are phagocytic cells that are the first white blood cells to arrive to infected and injured tissues.Chronic infections involve macrophages, lymphocytes, and plasma cells.B lymphocytes may become activated and mature into plasma cells, which produce antibodies.Purulence (pus) is composed of dead and dying neutrophils.

Which of the following would be termed regeneration?

New connective tissue attachment The only type of healing that would be considered regeneration is the development of an entirely new connective tissue attachment All other types of healing are considered repair rather than regeneration. -Long junctional epithelium -connective tissue adhesion and root resorption -root resorption and ankylosis

A 50-year-old man, smelling of tobacco pipe smoke, presents for a periodic exam. Upon examination, a raised white lesion covering most of his hard palate and many red papules within the white lesion are noticed. Which of the following is the most likely diagnosis? Hairy leukoplakia Leukoplakia Lichen planus Nicotinic stomatitis Papillary hyperplasia

Nicotinic stomatitis Nicotinic stomatitis is condition that manifests on the hard palate as a white surface lesion associated with pipe and cigar smoking and is not considered premalignant. The palate may appear gray or white and contain many papules that are slightly elevated with red centers. Nicotinic stomatitis, induced by the palate being exposed to very hot conditions, is also associated with the chronic ingestion of hot beverages. It is found most commonly in men >45 years old. There is usually no treatment necessary because cancer risk is not elevated. Immediate cessation of smoking may be advised in order to monitor the lesion and see if differential diagnoses are required. Nicotinic stomatitis should completely resolve on its own after 1-2 weeks upon smoking cessation. If the lesion persists, a biopsy may be necessary to confirm the diagnosis.

Which of the following agents does NOT produce vasoconstriction? Levonordefrin Nitrous oxide Phenylephrine Norepinephrine Epinephrine

Nitrous oxide Nitrous oxide is converted into nitric oxide, which is known to induce vasodilation, not vasoconstriction. Epinephrine, levonordefrine, norepinephrine, and phenylephrine all induce vasoconstriction when administered parenterally.

Each of the following describes an ideal ergonomic position for dental work EXCEPT one. Which is the EXCEPTION? The entire seat or stool surface used to support the operator's weight No more than 10 in. between the operator's nose and the patient's oral cavity Elbows kept close to the body Elbows bent at 90° or greater Back and neck upright

No more than 10 in. between the operator's nose and the patient's oral cavity A good operator position includes at least ten inches or more space between the operator's nose and the patient's oral cavity in order to achieve better visualization, controlled instrumentation, and a convenient working environment. The operator should keep the elbows close to the body, with the back and neck in an upright posture and elbows at 90° or greater. The operator's weight should be supported by using the entire seat or stool surface to maintain balance.

A 35-year-old patient presents to your office with unrestored teeth #1-32, with staining of the occlusal pits and grooves without a detectable catch with the explorer. Which of the following is the best treatment option? No treatment Conservative class I amalgam fillings Preventive resin restoration Topical fluoride placement Bis-GMA sealant Cyanoacrylate sealant

No treatment The presence of stain within the occlusal pits and grooves of a sound tooth does not require any dental treatment.Dark stains observed in pits and fissures of teeth without any detectable catch is known as arrested caries, which occur when a formerly caries-affected area of tooth remineralizes, stopping the caries process.Incipient caries is distinguished by a chalky, white appearance on the smooth surfaces of teeth.Active dental caries can be distinguished during an oral examination as an explorer catch with soft, easy-to-flake-off tooth structure.Caries within the proximal contacts of two teeth can only be diagnosed using radiographs.

In which gland do MOST acinic cell adenocarcinomas occur?

Parotid gland Around 85% of all acidic cell adenocariconomas occur in the parotid gland, a logical finding because this is the largest gland and one that is composed entirely of serous elements

In which of the following cases can you use a conventional powered instrumentation?

None of the above: dental implants, patients with highly infectious diseases, patients with respiratory problems, and cardiac pacemakers placed in the 1980s Conventional powered instrumentation entails the use of sonic, ultrasonic handpicks. Both dental and medical cancers may provide contraindications for this type of treatment. Dental concerns are typically site-specific and the practitioner should evaluate whether the ultrasonic tip will damage any particular margin or restoration

An 11-year-old girl presents to your office four hours after a fall that fractured her maxillary left central incisor, leaving the remaining tooth structure 3 mm supragingival. The exposed pulp is still oozing blood. Which of the following is the treatment of choice? Nonsurgical root canal treatment Apexogenesis procedure followed by a root canal treatment Pulp cap Pulpotomy Pulpectomy and placement of calcium hydroxide

Nonsurgical root canal treatment Only a root canal treatment and exposure of adequate tooth length can properly restore the fractured maxillary left central incisor. A post and core build-up on a root canal treated tooth will help retain the final crown restoration that will reinforce the remaining tooth structure. Non-Surgical Root Canal We invite you to think of non-surgical root canal therapy as a really deep filling. While not exactly the same and certainly a more in-depth procedure, there are basic similarities. Both start off with drilling into the tooth, then cleaning, and then filling the hole. During a non-surgical root canal procedure, we simply drill farther than we would during a filling so that we can access the root canal (the hollow area inside the tooth with nerves) to clean it. In both cases, anesthetic is used to ensure your comfort and in both cases we fill the hole to prevent further infection. Apical, or Root Canal, Surgery Sometimes, however, the infection is either too significant or we simply cannot see the fracture or source of pain in an x-ray, and that is when a surgical root canal (or apicoectomy) is advised. During this procedure, an incision in the gum is made so that the canal can be accessed from the side and infected tissue can be removed safely.

A patient presents with a moderate facial bony undercut in the canine region and a severe undercut in the maxillary tuberosity. Which of the following indicates how the dentist should proceed to fabricate an immediate denture? Only relieve the denture base to bypass the undercuts. Only reduce the tuberosity surgically. Only reduce the facial bony undercut surgically. Reduce the tuberosity and facial bony undercut surgically.

Only reduce the tuberosity surgically. Surgical reduction of the severely undercut tuberosity is indicated in this case.Severe undercut present in the tuberosity region presents serious problems for the placement of an immediate denture.The moderate undercut in the anterior region can be overlooked at this stage.

Which of the following describes why barium salts are added to gutta-percha? No answers apply. Opacifier Reinforcement Plasticizer Preservative

Opacifier Barium is added to gutta-percha to make the material opaque on x-ray films. Gutta-percha is an obturation material that is composed of the following: Zinc oxide (70%) Barium sulfates (5%) Waxes, resins, and gutta-percha (24%) Evaluating obturation requires assessment of the following: Fit of gutta-percha points Density of gutta-percha fill Level of gutta-percha in the cana

Unilateral subcondylar fracture signs and symptoms includes:

Open bite Trismus Ipsilateral occlusal prematurity Inability to achieve maximum intercuspation Ipsilateral deviation of the mandible upon opening Patients who sustain a unilateral subcondylar fracture will typically present with an ipsilateral deviation of the mandible upon opening. The ipsilateral lateral pterygoid muscle (attached to the condyle) will be affected, leading to deviation of the mandible to the affected side when opening.

The route of administration of medication that allows the drug to be absorbed from the stomach and intestine is which of the following?

Oral Enteral administration of medication is performed through the oral route, in which the medicine is expected to be absorbed in the stomach and intestine. Intramuscular drug administration is performed by injecting the drug into the big muscles of the body. The inhalation route of drug administration is performed through the inspiration of medication in the form of gas. Subcutaneous drug administration is achieved through injecting the drug along the fatty subcutaneous layer under the skin. The sublingual route of drug administration is achieved by placing the drug under the tongue.

Which of the following is regarded as the most toxic form of mercury? Each form is considered equally as toxic Inorganic Elemental Organic

Organic Organic mercury is considered to be the most toxic form of mercury. Mercury can form organic compounds, such as methylmercury.Organic mercury compounds can be readily absorbed by many organisms and concentrated as they pass up the food chain. Organically bound mercury in food is the primary source of mercury exposure in most people.The human body is less effective at mercury excretion than absorption. Organic mercury has a tendency to concentrate in the liver, kidney, and brain after beIng absorbed.Higher concentrations of mercury in the human body is harmful. Mercury toxicity can result in bronchitis, pneumonia, kidney disease, and allergic reactions. When inhaled in toxic quantities, it can cause damage to the central nervous system.Dentists are at an increased risk of mercury toxicity because of the excessive use of amalgam fillings in the operatory. The maximum safe dose ingested in a forty-hour work week is 50 mg per cubic meter.The dental operatory should be well-ventilated with seamless floor tiles to decrease mercury retention and inhalation. Use of personal protective equipment is mandatory.

Which statement describes anticholinesterase (ACHE)?

Organophosphates are among the most toxic substances that can easily enter the body. They are active substances in some pesticides that enter a body by ingestion, by skin absorption, and mostly through inhalation.Organophosphates are capable of affecting cholinesterase activity within the blood. Cholinesterases are substances that cause the breakdown of acetylcholine when in the body, stopping the acetylcholine from sending electrical impulses, and therefore regulate movement of the body. With the presence of a cholinesterase inhibitor such as DFP, an organophosphate, acetylcholine is not broken down, resulting in acetylcholine buildup. This causes an uncontrolled and continuous firing of electrical impulses that results in uncontrollable muscle twitching. Severe cases could lead to the inability to breathe due to muscle paralysis and even death.

Which of the following conditions presents radiographically as a "sunburst" lesion and can also cause paresthsia, loosening and displacement of teeth, along with a localized swelling in the posterior mandible? Ossifying fibroma Dentigerous cyst Leukemia Hyperparathyroidism Osteosarcoma

Osteosarcoma Osteosarcomas are classified as a form of bone cancer that predominantly occurs in children and teenagers. The reason for the occurrence of osteosarcoma is still unknown at present, but some studies suggest a genetic etiology. Osteosarcomas usually manifest as a solid, indurated swelling that can be distinguished in radiographs as appearing moth-eaten or having a sunburst appearance. The sunburst appearance is due to the calcified tumor spicules that radiate outward from the bone.

Which of the following oral medications is indicated to treat a staphylococcal infection that is known to produce penicillinase? Streptomycin Oxacillin Moxalactam Penicillin V Penicillin G

Oxacillin Oxacillin is a penicillinase-resistant β-lactam similar to methicillin and is used to treat multi-drug-resistant S. aureus (MRSA) infections. Oxacillin has replaced methicillin in clinical use because of the adverse effects of methacillin. Antibiotic-resistant strains called oxacillin-resistant Staphylococcus aureus (MRSA/ORSA) have become increasingly prevalent worldwide. Penicillinase-resistant antibiotics include the following: Oxacillin Cloxacillin Dicloxacillin Nafcillin Methicillin

Which of the following opioid analgesics has the highest potency and greatest potential to cause dependence and addiction?

Oxycodone Oxycodone is an opiate drug that is mainly used for pain control. Prolonged use is not advised, because it increases the tendency for drug dependence and addiction to occur. Oxycodone is an extended-relief drug that is typically indicated for patients suffering from moderate, severe, and spontaneous pain because it stays in the blood longer and provides relief over a longer period of time. The increased tendency of drug dependence with oxycodone use also increases the risk for withdrawal symptoms if the drug intake is drastically stopped.

____composites were introduced to be inherently more viscous to afford a "feel" on insertion, similar to that of amalgam.

Packable composites Packable composites were developed to increase the viscosity and give the feeling on insertion, similar to that of amalgam. Because of the increased viscosity and resistance to packing, it can sometimes cause lateral displacement of the matrix band.

Patients exhibiting a "cotton wool" appearance of the bone on x-ray and an elevated level of alkaline phosphatase with normal calcium, phosphate, and aminotransferase concentrations in their blood are most probably suffering from which of the following diseases? Ewing sarcoma Paget disease of bone Multiple myeloma Hyperthyroidism Fibrous dysplasia

Paget disease of bone Paget disease of bone is a chronic disorder that can result in enlarged and misshapen bones, pain, fractures, and arthritis. Paget disease of bone is diagnosed by the presence of an elevated level of alkaline phosphatase and normal calcium, phosphate, and aminotransferase levels in the blood. The pathognomonic sign for Paget disease is a radiopaque "cotton wool" appearance of the bone on x-ray. Other signs of Paget disease include the following: Enlarged skull and facial bones Enlarged maxilla and alveolar ridges Displaced teeth Hypercementosis

During the intraoral assessment, the clinician notes a small (less than 1 cm) raised growth present on the retromolar pad distal to tooth #31. Which of the following descriptions would be correct when documenting this discovery? Papule Nodule Vesicle Pustule

Papule A nodule is a growth that is greater than 1 cm in diameter and protrudes above the skin surface. A papule is a growth that measures up to 1 cm in diameter and may be above, level with, or below the skin surface. A vesicle is less than 1 cm in diameter and contains serous fluid. A pustule may exhibit various sizes and contains pus.

The preservative methylparaben most likely demonstrates cross-sensitivity with which of the following agents? Ortho-toluidine derivatives Xylidides Meta-aminobutyric acid esters Para-aminobenzoic acid esters

Para-aminobenzoic acid esters Methylparaben can sometimes induce a cross-sensitivity reaction with the esters of para-aminobenzoic acid (PABA ester). This reaction occurs because the hydroxyl group of paraben is located in the para position, which can eventually react with the para-amino structure of PABA ester. The para position of amino group in PABA ester is a free amino group on the benzene ring that can react with the hydroxyl group of the paraben and cause the cross-sensitivity reaction.

Which of the following is the intended target of H2 histamine receptor blockers? Parietal cells Neutrophils Mast cells Arterial endothelial cells Basophils

Parietal cells Histamine stimulates the H2 receptor of the parietal cells of the stomach and is released by basophils and mast cells. H2 receptor stimulation induces a proton release into the stomach lumen, decreasing the pH. H2 receptor blockers reduce the amount of acid in the stomach.

ASA IV best describes _______?

Patient with severe systemic disease that is incapacitating but that may alter daily activity

Which of the following medications is removed from the body primarily via secretion from the renal tubules? Ciprofloxacin Streptomycin Penicillin G Tetracycline

Penicillin G Penicillin G (pen G) is an antibiotic drug taken parenterally because it is unstable in acids in the stomach. It undergoes rapid excretion through active tubular secretion and filtration in patients with healthy kidney function. The half-life of pen G prior to elimination is said to be about 30 minutes. Pen G is excreted more slowly in neonates and babies with compromised kidney function.

Which of the following is true therapeutically when comparing penicillin G to penicillin V? Penicillin V has more reliable gastrointestinal absorption. Penicillin V has decreased potential for allergic reaction. Penicillin V has a broader antibacterial spectrum. Penicillin V has increased resistance to penicillinase. Penicillin V has slower renal excretion time.

Penicillin V has more reliable gastrointestinal absorption. Benzylpenicillin (penicillin G) is administered parenterally because it is unstable to hydrochloric acid of the stomach and can achieve high tissue concentration within tissues. Phenoxymethylpenicillin (penicillin V) can be taken orally because it is more stable in the acidic environment of the stomach and is absorbed better than penicillin G.

How is penicillin VK eliminated from the body?

Penicillin VK is an antibiotic metabolized in the liver, to be excreted by the kidneys into urine. It is a chemically augmented form of penicillin that combines acid stability with an immediate solubility and faster rate of absorption. It is highly effective against pneumococci and staphylococci, except for strains producing penicillinase and streptococci (groups A, C, G, H, L, and M). Its half-life is approximately 30-60 minutes, and it takes about five hours to clear the body entirely.

For pulpal infections, the class of antibiotic for first line treatment is considered to be the:

Penicillins Most bacterial species responsible for endodontic infections are susceptible to penicillins. Penicillins are regarded as the first line of treatment in root canal infections Penicillin kills bacteria through binding of the beta-lactam ring to DD-transpeptidase, inhibiting its cross-linking activity and preventing new cell wall formation. Without a cell wall, a bacterial cell is vulnerable to outside water and molecular pressures, which causes the cell to quickly die. Cephalosporins are also Beta-lactam antibiotics and have some coverage of oral bacteria. However, they do not have as much coverage or effectiveness as penicillins for oral infections

Corticosteroids can be prescribed for each of the following conditions EXCEPT one. Which is the EXCEPTION? Cardiovascular disease Megaloblastic anemia Peptic ulcers Alcoholism Renalithiasis

Peptic ulcers Corticosteriods can cause peptic ulcers and can therefore worsen the condition of a person who already suffers from them. Corticosteroids have shown a marked increase in gastric acid secretion and a reduction in gastric mucus, as well as gastrin and parietal cell hyperplasia in animal studies. Studies have also shown that corticosteroids delay healing of previous ulcers. Corticosteroids are good for reducing inflammatory reactions in the body. High doses and prolonged use cause the immune system to weaken and cause a tendency to develop peptic ulcers.

Each of the following is a common oral manifestation that can be found in a human immunodeficiency virus (HIV)-positive patient EXCEPT one. Which is the EXCEPTION? Hairy leukoplakia Candidiasis Human papillomavirus (HPV) Perimylolysis Aphthous ulcers

Perimylolysis Perimylolysis describes the decalcification of the teeth from exposure to gastric acid in patients with chronic vomiting. Common oral manifestations of HIV-positive patients include the following: Hairy leukoplakia Herpetic lesions Oral and esophageal candidiasis Kaposi sarcoma Linear gingival erythema Necrotizing ulcerative gingivitis or periodontitis Human papillomavirus Lymphoma Recurrent aphthous ulcers

Each of the following is associated with full-thickness flaps EXCEPT one. Which is the EXCEPTION? Periosteum staying on the bone Dissection of the tissue with a blunt instrument Apical repositioning Osseous recontouring

Periosteum staying on the bone A full-thickness flap includes stripping all of the soft tissue (epithelium, connective tissue, and periosteum) from the bone.Following placement of the incisions, the flap is dissected using blunt instruments like a #9 molt periosteal elevator to separate the soft tissue from the bone.Full-thickness flaps are used when recontouring the osseous structures below or when the gingiva is repositioned apically.

Administering nitrous oxide frequently to a single patient can result in which of the following? Acute narrow-angle glaucoma Deep vein thrombosis Systemic myopathy Peripheral neuropathy

Peripheral neuropathy Nitrous oxide has been demonstrated to produce neurotoxicity similar to other N-methyl-D-aspartate (NMDA) antagonists with prolonged use. Long-term exposure can cause vitamin B12 deficiency, numbness, and reproductive side effects (in pregnant females).

Which of the following gingival hyperplasias will show histological evidence of calcification? Irritation fibroma Peripheral ossifying fibroma Peripheral giant cell granuloma Pyogenic granuloma Traumatic neuroma

Peripheral ossifying fibroma Peripheral ossifying fibromas are an overgrowth of gingival tissue caused by mucosal irritation or trauma and are similar in appearance to pyogenic granulomas and peripheral giant cell granulomas. Peripheral ossifying fibromas range from red to pink and are frequently ulcerated. They may appear sessile or pedunculated with a size of 2 cm or less. Sixty-six percent occur in females with prevalence being highest around 10-19 years of age. Peripheral ossifying fibromas appear microscopically as a combination of a mineralized product and fibrous proliferation. The mineralized portion may be bone, cementum-like, or dystrophic calcifications.

Which of the following drugs will cause the patient to experience gastrointestinal problems and gingival hyperplasia?

Phenytoin Phenytoin is a class 1b antiarrythmic drug that is effective in suppressing seizures. This antielipetic drug will block voltage gated sodium channels. This medication is used to prevent seizures and can treat abnormal heart rhythms. One of the main side effects of this drug are gingival enlargement. Other symptoms include gastrointestinal problems and sometimes behavioral changes such as drowsiness and confusion

The initial assessment of this patient reveals that there is a thin black line of subgingival calculus on the lingualis of the mandibular premolars. Subgingival calculus derives its color from which of the following? Selective bacterial adhesion Pigments from the breakdown of blood components Lack of salivary composition Increased substantivity of acquired pellicle

Pigments from the breakdown of blood components Subgingival calculus is usually darker in color, because pigments from blood breakdown are usually incorporated. Calculus or tartar deposits are most commonly seen on the lingual surfaces of mandibular anterior teeth because of their close proximity to the sublingual and submandibular salivary ducts. The buccal surfaces of maxillary molars are also known to be at risk for calculus deposits because of their proximity to the Stensen duct of the parotid glands.

Each of these drugs can produce xerostomia as a side effect EXCEPT one. Which one is the EXCEPTION? Pilocarpine Benzodiazepines Antihistamines Antidepressants Anticholinergics

Pilocarpine Xerostomia (dry mouth) is characterized by a decreased production of saliva or salivary flow in the oral cavity. Pilocarpine is used to treat xerostomia in patients. The subjective feeling of dryness in xerostomia can cause the following: Dental caries Oral candidiasis Oral dysesthesia Dry mucosa Dysphagia Fissured tongue associated with atrophy of filiform papillae Difficulty in wearing dentures

When curing a composite restoration that is larger than the tip of the curing light, the operator should do which of the following? Position the tip at a sufficient distance from the tooth to illuminate the entire restoration. Place the tip stepwise over each area and expose each area for the recommended time. Move the curing tip over the surface for the recommended time. Center the tip on the surface and cure the entire composite from that particular spot.

Place the tip stepwise over each area and expose each area for the recommended time. Proper exposure time for every portion of a composite resin will ensure that the entire restoration is completely polymerized.Placing the small tip over each area while following the recommended curing time will ensure that no unpolymerized composite will be left behind.Adequate curing of composite restoration is important to ensure the restoration has no weak areas, is properly sealed, and can perform at its best during mastication.

The development of gingivitis in adolescence is principally influenced by which of the following factors? Testosterone Plaque Estrogen Calculus Estradiol

Plaque Plaque is the primary etiology of gingival inflammation because the bacterial aggregates from plaque initiate the inflammatory reaction.Calculus is a predisposing factor for the formation of plaque. Calculus is hard and rough, making it a great niche for bacterial growth. Calculus on its own does not cause gingivitis.There is an increase in sex hormone levels during pregnancy and puberty. The sudden rise of sex hormone levels causes the gingiva to elicit an exaggerated reaction over plaque build up.The increase in sex hormone levels induces vasodilation and increases permeability along capillary walls, which allows more fluids to extravasate and cause severe inflammation. Estrogen is the primary hormone associated with puberty and pregnancy-asscociated gingivitis.

Which acts as the best conditioner to clean the tooth's surface when applying glass ionomer adhesives? Phosphoric acid Polyalkenoic acid (PAA) Ethanol Acetone Isopropyl alcohol No answers apply

Polyalkenoic acid (PAA) PAA is used for glass ionomers because it is a weaker acid that does NOT penetrate into the dentin as much as phosphoric acid.Phosphoric acid creates smaller tubules that extend gingivally; the higher molecular weight polymer in the glass ionomers cannot penetrate these tubules.Ethanol is a solvent primer used to remove excess water after rinsing with the acid.

Cardiac glycosides are known to decrease the concentration of which of the following ions in cardiac myocytes? Chloride (Cl−) Potassium (K+) Calcium (Ca++) Sodium (Na+)

Potassium (K+) Digitalis is a cardiac glycoside known to be potent inhibitors of Na+/K+ ATPase in the cell. When cardiac glycosides inhibit Na+/K+ ATPase, the transition state stabilizes and the sodium ions are not extruded, increasing intracellular sodium concentration. By inhibiting the ion transportation, the concentration of potassium in the cell is decreased. Inhibiting Na+/K+ ATPase causes an increase in the heart's contraction and consequent cardiac output.

Hydrochlorothiazide (HCTZ), a thiazide diuretic, may necessitate a supplementation with Chloride (Cl−) Calcium (Ca++) Potassium (K+) Magnesium (Mg++) Sodium (Na+)

Potassium (K+) Potassium (K+) and hydrochlorothiazide are used in combination in order to reduce the patient's blood pressure and prevent the occurrence of hypokalemia. Hydrochlorothiazide administration causes the body to lose potassium, so hypokalemia often occurs without supplementation. Potassium regulates intracellular osmoregulation and helps reduce unneeded water and salt inside the body.

Which of the following compounds is LEAST likely to provide topical anesthesia to a patient? Procaine (Novocaine®) Carbocaine (Polocaine®) Articaine (Septocaine®) Lidocaine (Xylocaine®) Benzocaine

Procaine (Novocaine®) Procaine is typically no longer used because new and more potent topical anesthetic options are already available for use. Procaine is an ester-type anesthetic with a short duration of action. It is mostly used in infiltration, nerve blocks, and epidural anesthesia. Procaine can induce allergic reactions due to para-aminobenzoic acid (PABA) metabolite that is formed during its metabolism.

The mechanism of action for the tetracycline family of drugs relates to the inhibition of which of the following? Cell lysis Cell wall synthesis DNA/RNA synthesis Protein synthesis

Protein synthesis Tetracycline is a broad-spectrum antibiotic that inhibits protein synthesis. It binds to the 30S subunit of the bacterial ribosomes, preventing the aminoacyl-tRNA from binding to the acceptor site. Tetracycline is considered a teratogen and is contraindicated for pregnant women and for children below eight years of age due to the possibility of tooth discoloration.

A 6 year old patient with an unrepaired ventricular septal defect (VSD) will need a routine teeth cleaning in the near future. Which of the following should be done with regards to this patient prior to their dental procedure?

Prophylactic antibiotics are not necessary in this patient This patient has an unprepared cyanotic congenital heart defect, which is not. Considered a risk factor that warrants prophylactic antibiotic administration. Routine teeth cleanings may lead to bleeding and can be considered a dental procedure that would warrant antibiotic prophylaxis in patients that are high risk for infective endocarditis, but in this case, the patient does not qualify. Unprepared cyanotic congenital heart defects would require prophylaxis In summary, in a number of infectious disease states, tissue hypoxia is a common and prominent microenvironmental feature.

An accidental intravascular administration of epinephrine causes undesirable cardiovascular effects. Coadministration of which of the following drugs would counteract these effects? Phenobarbital and succinylcholine Propranolol and prazosin Atropine and propranolol Atropine and prazosin Amphetamine and propranolol

Propranolol and prazosin Sympathomimetic drugs are vasoconstrictive in nature and increase the blood pressure, heart rate, and force of contraction of the heart. To manage the accidental administration of a sympathomimetic drug, a therapeutic combination of systemic vasodilators and diuretics is considered ideal. Propranolol (a beta blocker) and prazosin (an alpha blocker), when administered together, create vasodilation and block the cardiovascular actions of the sympathomimetic drug.

Which of the following constituents of intravenous (IV) digoxin, IV diazepam, and IV phenytoin is considered the putative agent responsible for inducing thrombophlebitis? Sodium metabisulfite Propylene glycol Methanol Ethyl alcohol Benzoic acid

Propylene glycol The introduction of propylene glycol may cause thrombophlebitis and hemolysis of large veins during intravenous drug injection. Propylene glycol is commonly present in intravenous drugs like digoxin, phenytoin, diazepam, and etomidate and serves to make them more soluble and stabilize. Propylene glycol is regarded as safe, though it may become toxic at high levels in pediatric patients.

Which of the following molecules is derived from the unsaturated fatty acids in cell membranes?

Prostaglandins Prostaglandins are lipid molecules that are enzymatically derived from an arachadonic acid. Endorphins are neurotransmitters that endogenously form morphine from the opioid peptides in the body. Bradykinin is a protein peptide that belongs with the kinin group. Enterochromaffin cells secrete serotonin. Histamine is produced by mast cells and basophils as an immunologic response to a foreign body that enters the body.

Which of the following tests is most valuable for evaluating the surgical risk of a patient on warfarin who needs a tooth extracted?

Prothrombin time (PT) measures the clotting tendency of blood. The normal range is 12-13 seconds. The international normalized ratio (INR) is also used in conjunction with PT. It should come within a range of 0.8-1.2. Warfarin is an anticoagulant that prolongs PT, and the INR comes in the range of two to three. PT is a good diagnostic tool to be used for a patient on warfarin before tooth extraction.

Which of the following types of excursive movements would be interfered with if a mesial marginal ridge of a maxillary left 2nd molar was built up too high during a restoration placement? Select All That Apply Protrusive Working Retrusive Non-working

Protrusive Retrusive An overbuilt mesial marginal ridge of a maxillary 2nd molar tooth will interfere with protrusive and retrusive (anteroposterior) movements.The high spot will hinder the forward movement because it will come in contact with the opposing tooth earlier.It is essential to remove all the high spots after restoring the tooth so that occlusal trauma is not incurred by the tooth and its antagonist.Articulating paper can be used to identify the high spots to carry out their removal.High spots in a restoration will affect the normal functioning and cause pain in the long run.

During the autoclaving process, several indicators are used to ensure full completion of sterilization. These include process indicators and biological monitors. What differentiates biological monitors from process indicators? The biological monitors:

Prove that sterilization was achieved by indicating bacterial spores were killed The differentiating factor between biological markers and process indicators is that biological markers PROVE sterilization has been completed. The indicators used for this are bacterial spores that can only be killed under extremely high temperatures. Process indicators are solely for the purpose of showing that the load has been processed and that is often done by color change on the indicator strip to show the strip was exposed to the appropriate sterilization cycle.

Which location indicates where the initial interferences are encountered when first seating a cast restoration? Occlusal surfaces Proximal contacts Margins Axial walls

Proximal contacts Proximal contacts are where initial interferences occur when seating a cast restoration because these areas are usually overcontoured.Overcontoured proximal contacts are usually related to some errors in fabrication.There are instances when laboratories tend to overcontour the proximal in the hope of achieving tight proximal contacts.A simple chip or a trim along the proximal surface of the tooth adjacent to the cavity preparation on the master cast may provide significant change in the fabricated proximal contact of a cast restoration.Overcontoured proximal areas will result in premature contacts that will prevent proper seating of the restoration. When managing overcontoured proximal contacts, it is best to identify the premature contact area with accufilm and trim the excess material until the proper contour is achieved.

Bitewing radiographs are best used for the detection of which type of carious lesion?

Proximal lesions Proximal lesions are usually found apical to the contact point

Bitewing radiographs are best used for diagnosing which type of caries?

Proximal surface lesions It is difficult to see carious lesions on proximal surfaces because the lesion forms just cervical to the contact area and vision is obscured by the adjacent tooth. If the lesion is discovered clinically, it is usually at a relatively late stage when it has already progressed well into the dentin and is seen as a pinkish grayish shining area on the marginal ridge. Bitewings radiographs are of paramount importance in diagnosing proximal caries in both enamel and dentin.

An oral cytology smear is most indicated for the diagnosis of which condition? Pseudomembraneous candidosis Papilloma Mucous membrane pemphigoid Lichen planus Squamous cell carcinoma

Pseudomembraneous candidosis The clinical appearance of pseudomembranous candidosis alone is sufficient for the diagnosis of candidosis in most cases. Definitive diagnosis of candidosis requires specialized tests, such as a cytologic smear. The membranous slough of pseudomembranous candidosis can often be wiped off with gauze to reveal an erythematous surface underneath. Smears are collected by gentle scraping of the lesion with a spatula or tongue blade, with the resulting debris being directly applied to a glass slide for a cytopathological exam.

The fluoroquinolone antibiotic ciprofloxacin is effective against otitis media in young children. anaerobic bacteria only. oral anaerobes. Pseudomonas aeruginosa.

Pseudomonas aeruginosa. Ciprofloxacin is a second-generation fluoroquinolone antibiotic with a broad spectrum of activity and excellent tissue penetration. It effectively treats Pseudomonas aeruginosa because of its effectiveness against Gram-negative organisms. Ciprofloxacin can be used both orally and parenterally. It is bacteriocidal because it interferes with the rewinding of bacterial DNA after it is copied, stopping DNA synthesis. Coadministration of ciprofloxacin with other drugs primarily metabolized by CYP1A2 results in increased plasma concentrations of these drugs and could lead to clinically significant adverse events of the coadministered drug. Ciprofloxacin is also known for its tendency to cause spontaneous tendon ruptures and may worsen the symptoms of myasthenia gravis. 0 CommentsSort by

What is the slowest and most difficult tooth movement?

Pure translation: A tooth is said to be translated when all points on a body move the same amount in the same direction. Pure translation, or bodily movement, is the slowest and most difficult type of tooth movement. Tipping is the easiest and fastest movement to accomplish, but it is often the least desirable tooth movement. Crown movement and pure rotation are more difficult that tipping and occur more slowly Crown tipping is the tilting of the crown of a tooth without moving the apex of the root.

The major source of terrestrial radiation is:

Radon Radon is a radioactive gas that forms naturally when uranium, thorium, or radium, which are radioactive metals break down in rocks, soil and groundwater. Radon is a naturally occurring radioactive gas produced by the breakdown of uranium in rocks and soils. Radon gas is tasteless, colorless and odorless. The only way to know if it is in your home is to test for it.

Aggressive Periodontitis is associated with

Rapid attachment and bone loss Amount of destruction is not proportional to the amount of plaque present It is usually associated with specific bacteria It is genetic Patients with this periodontal disease usually have a "healthy" appearance. Aggressive periodontal disease usually does not have any signs of other systemic diseases associated with it or have the amount of destruction proportional to the amount of contributing local factors

Each of the following signs or symptoms is associated with typical poisoning by organophosphate pesticides EXCEPT one. Which is the EXCEPTION? Skeletal muscle fasciculation Hyperlacrimation Rash Hypersalivation Diarrhea

Rash Organophosphate poisoning causes several signs and symptoms that occur because of the excessive amounts of acetylcholine along the different receptors and nerve fibers within the body. The excess acetylcholine (Ach) is brought about by blockage of acetylcholinesterase. The nicotinic expression becomes overstimulated due to the increased ACh along the motor nerves, causing muscle cramps, fatigue, fasciculation, muscle weakness, and paralysis. The muscarinic receptors are also affected by organophosphate poisoning.

Which of the following describes the intended function of a home dental water-irrigation system [water pick]? Remove biofilm on the tooth Remove supergingival calculus Reduce pocket depth Reduce gingival bacterial load

Reduce gingival bacterial load Home water-irrigation systems are designed to reduce the amount of bacteria on the gingiva, not the tooth surface. Gram-positive bacteria produce dextrans, levans, and glucans, which let them better adhere to tooth structure. Gram-negative bacteria produce alginates to help them to adhere to soft tissues. Gram-negative bacteria also adhere to Gram-positive bacteria that are adhered to the tooth surface. Spirochetes cannot produce the same adherence factors as Gram-negative and Gram-positive bacteria, so they can be flushed from the pocket In addition to being used in the dental office, oral irrigation can be done at home with a water pick. The device delivers a jet of high-pressure water to eliminate food particles and other debris stuck between teeth. Many individuals prefer a water pick to flossing to clean between the teeth.

Each of the following is an advantage to investigating the biocompatibility of a biomaterial using in vitro testing EXCEPT one. Which is the EXCEPTION? An in vitro test is quick to perform. An in vitro test can be standardized. Relevance to the use of the material is assured in an in vitro test. An in vitro test is excellent for examining mechanisms of interactions.

Relevance to the use of the material is assured in an in vitro test. Relevance to the use of the material is not assured while investigating the biocompatibility of a biomaterial in an in vitro test. The following are advantages of in vitro tests: Quick to perform Can be standardized Excellent for examining mechanisms of interactions

Which of the following is the reason trituration is completed? Activation of the copper in the alloy All answers are correct. Removal of the oxide coating and wetting of each particle of alloy with zinc Removal of the oxide coating and the wetting of each particle of alloy with mercury

Removal of the oxide coating and the wetting of each particle of alloy with mercury The primary objective of the process of trituration of amalgam is to remove the oxide coating and wet each particle of alloy with mercury. Properly triturated amalgam is a homogenous mass with a slightly reflective surface. It flattens slightly if dropped on a tabletop. A correctly mixed amalgam should have sufficient wetness to aid in achieving a well-adapted restoration and not be dry or crumbly.

Temporomandibular joint (TMJ) arthroscopy is indicated for all of the following EXCEPT one. Which is the EXCEPTION? Disk manipulation Lysis of adhesions Repair of disk perforations Therapeutic lavage

Repair of disk perforations Repairing disk perforations requires making a flap incision to access the temporomandibular disk. Lavage of the TMJ, removing adhesions, and disk manipulation can all be completed using arthroscopy. Diagnosis of disk perforations is best completed using T2-weighted magnetic resonance imaging.

Flexural strength describes a material with the ability to do which of the following? Resist fracture during bending Be visually detectable in radiographs Resist the propagation of a crack Sustain deformation without permanent change in size or shape

Resist fracture during bending Flexural strength is the ability of a material to resist fracture while bending and is also known as modulus of rupture, bend strength, or fracture strength.The transverse bending test is most frequently employed to evaluate the flexural strength of a material using a three-point flexural test technique.The flexural strength gives the highest stress experienced within the material at its moment of rupture.The significance of flexural strength is commonly expressed in Class V cervical restorations.

Which of the following describes the likely mechanism of death from an irreversible anticholinesterase overdose?

Respiratory failure due to diaphragmatic and intercostal muscle paralysis Irreversible anticholinesterase overdose inflicts death by causing the diaphragm and other muscles needed during respiration to fail. The diaphragm and intercostal muscles expel air out of the lungs during expiration. The other effects of an anticholinesterase overdose include the following: Bradycardia Death due to bronchoconstriction Gastrointestinal (GI) motility and secretion problems Miosis

Which of the following antibiotics is used to treat mycobacterium infections and is contraindicated with the use of antiretroviral agents?

Rifampicin Rifampicin is considered to be bactericidal and inhinbits DNA-dependent RNA synthesis. This medication is part of the drug treatment used to treat tuberculosis. Mycobacterium infections are primarily treated by rifampicin Hepatotoxicity is the most serious adverse effect associated with this drug Rifampicin also is a cytochrome P450 inducer and alters the metabolism of antiretroviral medications such as clarithromycin and atorvastatin

Which of the following should be performed to verify that an autoclave is working and can still sterilize instruments? Determine whether the autoclave "working" light turns on during a cycle. Examine the sterilizer packages for change in color. Place indicator tape on the instruments. Run the autoclave with a biological monitor test.

Run the autoclave with a biological monitor test. Biological indicators used in biological monitoring tests are the most accepted means of monitoring the sterilization process. They directly determine the presence of the most resistant microorganisms or spores (spore testing). Biological monitoring tests also carry out the determination of the physical and chemical conditions of the sterilization met.

A large (~30 mm) circular radiolucency is noted close to the angle of the mandible overlying the submandibular salivary gland. Which of the following is the likely diagnosis for this asymptomatic lesion? Salivary gland inclusion defect (Stafne defect) Odontogenic keratocyst Ameloblastoma Traumatic bone cyst Focal osteoporotic bone marrow defect

Salivary gland inclusion defect (Stafne defect) The Stafne defect is a depression located along the lingual surface of the mandible that is near the tongue and the submandibular salivary gland. It is a well-demarcated radiolucency that is approximately about 10-35 mm in diameter. A Stafne defect is considered to be an anatomical variant that normally occurs in human population. It is a depression made by ectopically formed salivary gland tissue that is near the submandibular gland. The Stafne defect is not pathologic and is considered a pseudocyst because it does not have fluid and epithelial lining.

Dentin hypersensitivity may be caused by?

Scraping or cutting of dentin Hypertonic solutions Cold application Dentin hypersensitivity pain is explained through the hydrodynamic theory of pain. This suggests that thermal and osmotic stimuli cause fluid to move through the dentinal tubules, which then elicits firing of the pulpal nerves. *Thus the blockage of dentinal tubules is more likely to prevent dentin hypersensitivity since it would limit both the introduction of noxious stimuli into the tubules and the movement of dentinal fluid

During an appointment, you observe the patient exhibiting a blank stare for five to thirty seconds, followed by spastic muscle contractions. Which of the following is the patient most likely experiencing? Seizure Hypoglycemia Syncope Orthostatic hypotension

Seizure The patient has noted a history of seizure on her medical history. Transient loss of consciousness and minor motor movements which last five to thirty seconds are signs and symptoms of a generalized absence (petit mal) seizure. Hypoglycemia is a sign of insulin shock, in which intakes of insulin and food are not balanced. Symptoms include nervousness, sweating, compulsive talking, mental confusion, nausea, convulsions, and coma. Syncope is a sudden, transient loss of consciousness and is the most common medical emergency in a dental setting. Three stages of signs and symptoms of syncope are noted: Presyncope comprises weakness and nausea with a feeling of lightheadedness, tingling in the toes and fingers, and increased pulse rate. The patient may become pale and clammy. Syncope is impaired consciousness. Patients are pale with a weak pulse and shallow breathing. The duration is less than five minutes. Post-syncope, the patient awakens with blood pressure and pulse rate returning to normal. Patients typically feel weak and disoriented. Orthostatic hypotension (postural hypotension) is a form of low blood pressure that happens when the patient stands up after sitting or lying down. The patient may feel dizzy or lightheaded after standing.

Which of the following conditions is an absolute contraindication for the administration of opiods?

Severe cranial trauma Patients suffering severe cranial trauma are advised NOT to take opiates because of the possible increase in intracranial pressure.Opioids have the ability to significantly elevate the intracranial pressure which may cause each of the following in cranial trauma patients: Intracranial bleeding Blood clot Brain damage Shifting of brain structures Herniation of the brain Impeded supply of blood to the brain The authors reported that both opioids cause small and transient increases in intracranial pressure (ICP), regardless of the autoregulatory capacity of the patient.

Which of the following is the primary manifestation indicating an intra-arterial injection of midazolam? Rapid decrease in respiration rate Severe cardiac arrhythmia Severe pain in the extremity Rapid decrease in blood pressure Exaggerated sedation due to the more direct effect of the drug

Severe pain in the extremity Limited reports have been documented with regard to the effects of intra-arterial injection of midazolam. Adverse reaction from accidental intra-arterial injection can be localized within the site of injection or systemic. There are some isolated reports that note seizure activity occurring after intra-arterial injection of midazolam. However, the reason for such reaction is not established. Some accounts of intra-arterial injection of midazolam state that some of the patients experienced moderate to severe pain within the extremity where it was administered.

Which of the following would NOT cause an increase in complication or difficult during extraction of a maxillary second premolar?

Severe periodontal disease Severe periodontal disease would cause extensive alveolar bone loss. Because of this supporting bone loss, often periodontally involved teeth become easier to extract. Care must just be taken to elevate all the way up to the apical portion of the root in order to prevent breakage. [Severe bruxism will often cause the surrounding alveolar bone to become much more dense. These teeth also tend to have a much stronger periodontal ligament attachment, making the extraction process more difficult] [Root candled teeth become brittle over time and the periodontal ligament also tends to become stronger as well. Due to this, the extraction of root candled teeth becomes difficult since they can fracture into many small pieces and create much frustration in their removal]

Which of the following represents why it would be a good clinical decision to replace a silicate cement restoration on the distal of tooth #11? Silicate cement restorations will result in open contacts. Silicate cement restorations on proximal surfaces cause gingival irritation. Silicate cements stain easily and are considered unesthetic. Silicate cement restorations will not maintain the mesio-distal width of tooth #11.

Silicate cement restorations will not maintain the mesio-distal width of tooth #11. Silicate cement restorations are not good for restoring teeth with proximal contacts.Glass particles in silicate cements are easily dislodged from the restorative filling.Silicate cement is brittle and prone to surface crazing and chipping on the margins.Because silicate cements wear easily, they are unsuitable to for restoring: Proximal contacts Stress-bearing areas

The concentration of which of the following ions is most closely associated with an opioid agonist or antagonist's binding affinity with the opioid receptor?

Sodium (Na+) The actions of opioids are primarily inhibitory. Opioids close N-type voltage-operated calcium channels and open calcium-dependent inwardly-rectifying potassium channels. This results in hyperpolarization and a reduction in neuronal excitability. Kappa receptors may act only on calcium channels. The pharmacodynamic effects of opioids are dependent on their ability to inhibit the release of neurotransmitters. Opioids inhibit neurotransmitter release through the following: Reducing entry of calcium ions into the nerve Increasing the outward movement of potassium ions Inhibiting adenylate cyclase Sodium ions regulate the binding affinity of opioid agonists and antagonists to opioid receptors. Binding affinity depends on the following: Receptor type Affinity of sodium with the receptor Whether the substance is an opioid agonist or antagonist

Which of the following materials has a linear coefficient of thermal expansion that is most similar to that of tooth structure?

Stainless steel The linear coefficient of thermal expansion is a physical property of a material that determines the material's expansion or contraction when exposed to temperature variation. Tooth structure has an LCTE of 9-11 ppm degrees Celsius and stainless steel has one of 11 ppm degrees celcius. The more similar a material is to the LCTE of tooth structure, the less likely micro leakage will occur around a restoration

Which of the following instruments should NOT be utilized on the implant replacing tooth 19? Graphite instrument Gold-tipped (titanium nitride) Gracey 13/14 Stainless steel Gracey 13/14 Plastic resin instrument Titanium Gracey

Stainless steel Gracey 13/14 Instruments indicated for implant debridement include the following: Plastic resin instruments Graphite instruments Gold-tipped (titanium nitride) instruments Ultrasonic scaler with plastic tip Instruments contraindicated for use on implants include the following: Ultrasonic scaler with metal tip Stainless steel instruments This is not based on prosthodontics but hygienist anxiousness and most dentists ought to be able not to scratch a metal abutment surface.

Statement 1: The best cause of action would be to recommend the mother replant the tooth, or place it in saline or milk if possible and then come immediately to the dental office Statement 2: The single most important factor to achieve a good prognosis after replantation is the cleanliness of the periodontal ligament, therefore it should be scrubbed clean of debris

Statement 1 is correct, statement 2 is false In a 16 year old patient, the apex of tooth number 8 would be closed. A permanent tooth with a closed apex is a good candidate for immediate replantation. The first thing that should be done immediately in an emergent situation would be to try to replant the tooth. Favorable outcome improves greatly if this is done within an hour of avulsion. If the mother or patient in this situation is not able to do this, then placing the tooth in a storage medium like saline, milk, or even saliva would be the next course of action. The tooth root should not be scrubbed clean as that would damage the periodontal ligament fibers. It would be advisable to make sure the gross debris is gently removed, but the most important factor I'm determining positive outcome remains the speed at which the tooth is replanted (within minutes to an hour)

Dental burs are most commonly made of which material? Iron Steel carbide Tungsten carbide Galvanized iron

Steel carbide Tungsten carbide Most dental burs are made of tungsten carbide. Tungsten carbide burs are most efficient in the removal of soft carious lesions by using them with a touching action. These are less efficient in cutting hard tooth enamel. Tungsten carbide is a metal that is extremely hard (about three times stiffer than steel) and can withstand high temperatures. Because tungsten carbide is so hard, it can maintain a sharp cutting edge and be used many times without becoming dull. However, tungsten carbide burs are brittle and have a tendency to fracture under pressure. They are best operated at high speeds with light pressure. Steel carbide burs dull more quickly, but are less prone to breaking. Carbide burs can be of different types according to their clinical uses: Cutting burs Excavation burs Finishing and polishing burs

What is the name of the technique in which deep caries are excavated from a tooth, a small amount of affected dentin is left (provided that a caries-free dentin-enamel junction [DEJ] is present), and a temporary restoration is placed and then replaced with a permanent restoration following an analysis six months later? Semipermanent caries excavation Stepwise caries removal Indirect pulp cap Direct pulp cap

Stepwise caries removal Stepwise caries removal occurs when decay is excavated around the margins of a preparation, and the tooth structure is remineralized until the permanent restoration is placed later. The stepwise caries removal steps are as follows: Totally remove caries from the margins of the preparation, sparing the pulpal affected dentin but not the infected dentin. Place a CaOH liner and restore with glass ionomer. Six to twelve months later, remove glass ionomer, test tooth vitality, and then place definitive restoration. Indirect pulp capping is different because a permanent restoration is placed at the first appointment and is more aggressive with caries excavation.

Which of the following excision techniques is appropriate for resolving a recurrent ranula? Parotid gland excision Sublingual gland excision Marsupialization Electrosurgery Cryosurgery

Sublingual gland excision Ranulas have a recurrence rate of 25% and are mucoceles found on the floor of the mouth that stem from ruptured salivary glands Complete excision of the sublingual salivary gland is helpful in treating this condition

The relative insolubility in water of which of the following drugs may lead to an increased incidence in renal lithiasis (kidney stones)? Sulfamethoxazole Tetracycline metabolites Erythromycin Clindamycin Acetaminophen

Sulfamethoxazole Sulfamethoxazole is a sulfa drug that is relatively water-insoluble, so it is associated with a high occurrence of kidney stones. Because of the risk of kidney stones, it is usually prescribed with trimethoprim, creating a synergistic combination that allows less sulfamethoxazole to be given, decreasing kidney stone risk. Sulfamethoxazole is most commonly prescribed for streptococcus, staphylococcus, and E. coli infections. Sulfa drugs are structural analogs and competitive antagonists of para-aminobenzoic acid (PABA), which is used in the bacterial synthesis of folic acid. Interfering with folic acid synthesis prevents bacterial DNA replication.

You are planning to work closely with your restorative dentist to determine the final dimensions of the future restorations. Which of the following references will BEST determine the appropriate level of the gingival margin? Surgical stent Cemento-enamel junction (CEJ) of the adjacent teeth Wax-up on the diagnostic casts

Surgical stent After the wax-up is performed, the diagnostic cast is duplicated, and then the restorative dentist fabricates a template, called a surgical stent, which is used to establish the appropriate hard and soft tissue architecture. This template helps the surgeon determine the future gingival margin's ideal position and the final restoration length. The CEJ of the adjacent teeth should only be used as a reference in cases that do not require future restorations or when the only intervention is crown lengthening.If the wax-up is the only reference used, the likelihood of discrepancies between the future gingival margins increases.

Which of the following is the most common diagnosis for severe pain upon percussion of a tooth? Chronic apical abscess Symptomatic apical periodontitis Reversible pulpitis Irreversible pulpitis

Symptomatic apical periodontitis Symptomatic apical periodontitis (SAP) presents with pain upon masticatory forces or percussion. SAP is a periapical diagnosis, whereas irreversible pulpitis is a pulpal diagnosis. SAP is indicative of inflamed periapical tissues and may present with or without a radiolucency associated with the root apices of the tooth. Chronic apical abscess (CAA) is typically regarded as less painful to percussive forces, because the sinus tract associated with it is able to relieve some of the built-up percussive forces.

Which of the following would be the diagnosis for a patient with a tooth that demonstrates spontaneous pain, is not sensitive to hot or cold, but is sensitive to percussion and does NOT have a periapical radiolucency? Asymptomatic apical periodontitis Reversible pulpitis Chronic apical abscess Irreversible pulpitis Symptomatic apical periodontitis

Symptomatic apical periodontitis Teeth exhibiting symptomatic apical periodontitis (SAP) are sensitive to percussion and occlusal forces and do not necessarily demonstrate periapical radiolucency. Teeth exhibiting asymptomatic apical periodontitis (AAP) are not sensitive to percussion or palpation and demonstrate a periapical radiolucency. Irreversible pulpitis (IP) is diagnosed by the presence of a lingering sensitivity after the cold stimulus is removed. IP may or may not elicit spontaneous pain. Reversible pulpitis is characterized by a patient that may be sensitive to cold or sweet, but the pain quickly diminishes once the stimulus is removed. Chronic apical abscess (CAA) always demonstrates a periapical radiolucency and may or may not elicit pain on percussion and palpation. CAA also manifests a sinus tract.

A patient presents with severe intermittent pain on tooth #8 that wakes him up at night. Pulp testing reveals a lingering pain to cold and electric dental pulp vitality test (EPT). Tooth #8 is also very sensitive to percussion and palpation, with no evidence of periapical radiolucency. Which of the following is the pulpal and periapical diagnosis? Symptomatic irreversible pulpitis with symptomatic apical periodontitis Reversible pulpitis with symptomatic apical periodontitis Necrotic pulp with asymptomatic apical periodontitis Reversible pulpitis with a normal periapex

Symptomatic irreversible pulpitis with symptomatic apical periodontitis Irreversible pulpitis is a condition known to elicit spontaneous, lingering, and throbbing pain, sometimes without a stimulus. The continuous pain experienced by the patient is an indication that the pulp is already inflamed and irritated, even without external stimulus.

Each of the following is a characteristic of advanced osteoarthritis of the temporomandibular joint (TMJ) EXCEPT one. Which is the EXCEPTION? Tendency toward apertognathia Crepitus Flattening of articular surface Antegonial notching Tendency toward prognathism

Tendency toward prognathism Advanced cases of osteoarthritis of the TMJ may exhibit crepitus, flattening of the articular surface of the condyle, limited jaw movement, and pain on mouth opening. In some cases, antegonial notching is present. Advanced osteoarthritis may cause apertognathia (anterior open bite). The decrease in condylar height experienced by patients with severe osteoarthritis may make the posterior teeth occlude prematurely, resulting in an anterior open bite. Advanced osteoarthritis can result in a pseudoprognathic-appearing mandible and cause deviation of jaw movement, further aggravating the condition, but this is not considered a true prognathia.

A serious drug reaction may occur involving the cardiovascular system of patients taking erythromycin that may cause death in certain individuals. Which antihistamine drug is contraindicated for use with erythromycin due to that interaction?

Terfenadine Terfenadine is a relatively safe antihistimine that may cause cardiac arrhythmias with overdose. Terfenadine became popular because of its less sedating effect compared to other antihistamines. Erythromycin is a macrolide antibiotic able to inhibit the metabolism of terfenadine, making overdose and cardiac arrhythmia more likely. Terfenadine competes with histamine for binding at H1-receptor sites in the GI tract, uterus, large blood vessels, and bronchial muscle. This reversible binding of terfenadine to H1-receptors suppresses the formation of edema, flare, and pruritus resulting from histaminic activity Argentina's national medical authority, ANMAT, has prohibited the manufacture and marketing of products containing the antihistamines astemizole and terfenadine because of the risk that these active ingredients can produce arrythmia. These ingredients have already been banned in most worldwide markets.

Which of the following antibiotics is considered bacteriostatic? Tetracycline Streptomycin Vancomycin Penicillin Cefuroxime

Tetracycline Tetracycline antibiotics are considered bacteriostatic. Coincident bacteriocidal and bacteriostatic antibiotic use is contraindicated. Remember the bacteriostatic antibiotics using the mnemonic "We're ECSTATiC about bacteriostatics": Erythromycin Clindamycin Sulfonamides Tetracycline Azithromycin Trimethoprim Chloramphenicol Remember the bacteriocidal antibiotics with the mnemonic "Very Finely Proficient At Cell Murder": Vancomycin Fluoroquinolones Penicillins Aminoglycosides Cephalosporins Metronidazole

Which of the following antibiotic agents is found in considerably lower concentrations in serum than in crevicular fluid? Tetracycline Penicillin Metronidazole Oxacillin Clindamycin

Tetracycline Tetracycline is a broad-spectrum bacteriostatic antibiotic that functions by inhibiting protein synthesis. The most important feature of tetracycline is that it attains a higher level of concentration in gingival crevicular fluid than in serum. It is used to treat periodontal diseases caused by pathogens like Aggregatibacter actinomycetemcomitans and Porphyromonas ginigivalis because of the high concentration in crevicular fluids.

Which of the following is the principal benefit of intravenous sedation? Fewer side effects The ability to titrate doses Slower biotransformation and prolonged action A smooth and more gradual onset of sedation

The ability to titrate doses Intravenous (IV) sedation is advantageous for both the doctor and the patient, because the flow of medication can be easily controlled and titrated to safe levels, preventing any unnecessary overdose. Because titration of the drug is possible with an IV route, the flow of the drug is controlled in direct consideration with the patient's immediate response. Intravenous sedation is fast-acting and is directed to the bloodstream, which reduces the required drug dose needed to provide an effect, as compared to other routes of administration. IV sedation also allows the patient to recover faster after the drug is titrated down and slowly removed from the bloodstream

Regarding endodontic access, convenience form refers to:

The allowance for an outline form to facilitate unstrained instrument placement and manipulation. Convenience form basically provides modifications for the ideal outline form as would be required for each clinical situation

When carving a freshly condensed amalgam, the amalgam begins to chip and flake away. Which explains this occurrence? The amalgam was condensed after its working time had elapsed. The alloy was improperly mixed and was not fully wetted with mercury. The amalgam was contaminated by moisture. A low-copper alloy was used.

The amalgam was condensed after its working time had elapsed. Overcondensation of amalgam alloy will result in chipping away of the filling material during carving.Prolonged condensation pressure will result in a decrease in strength of the amalgam restoration.Amalgam alloys typically take roughly 24 hours to set completely, so undue pressure or disturbance during this time will result in the restoration breaking.

Which of the following describes what happens to the caries risk of a mouse that is fed by a gut tube so that no carbohydrates were allowed in the oral cavity? The caries rate stays the same as if the mouse was fed directly. The caries risk of the mouse increases. The caries risk of the mouse decreases.

The caries risk of the mouse decreases. Caries risk is assessed by the evaluation of factors such as amount, frequency, and duration of carbohydrate ingestion.The presence of fermentable food substrates like carbohydrates in the mouth is necessary for caries generation.Cariogenic bacteria like Streptococcus mutans and Lactobacillus casei ferment carbohydrates into acid, cause demineralization to occur with pH <5, and secrete collagenases that break down demineralized tooth structure. Pits and fissures promote deposition of food debris and decrease cleanability, increasing caries risk.Increased duration and frequency of carbohydrates in the oral cavity increases the risk for caries formation.

Pseudomonas infections indicate prescription of which of the following penicillins? Nafcillin Benzathine penicillin Phenoxymethyl penicillin Ticarcillin Amoxicillin

Ticarcillin Pseudomonas are bacteria that most commonly cause nosocomial infections. Carbenicillin and ticarcillin are two penicillin-derived antibiotic drugs that are effective against pseudomonas and enterobacter infections. Pseudomonas easily multiply in the most uncommon places, which include soaps, tubs, sinks, and stocked distilled water.

Which of the following statements about the clinical crown is true? The clinical crown is also known as the anatomical crown. The clinical crown is the portion of the crown that is coronal to the free gingival margin. The clinical crown is the portion of the crown coated with enamel that terminates at the cemento-enamel junction (CEJ).

The clinical crown is the portion of the crown that is coronal to the free gingival margin. The height of the anatomic crown is measured from the CEJ to the incisal edge, while the height of the clinical crown is measured from the gingival margin to the incisal edge. A comparison of these two measurements will determine whether short clinical crowns are a result of incisal wear or a coronal position of the gingival margin. Coronal position of the gingival margin with respect to the CEJ may be the result of delayed passive eruption or gingival enlargement.

If during excavation of caries, the caries extend beyond the external outline of the preparation and enamel becomes undermined, which of the following could be modified to increase access to the caries?

The convenience form is the form that facilitates and provides sufficient visibility, accessibility, and ease of operation in preparing and restoring the tooth. Occasionally to obtain this form, extension of distal, medial, facial, or lingual walls may be required to gain sufficient access to the deeper portion of the preparation

The location of the neuromuscular bundle from the mandibular foramen to the mental foramen is most accurately described by which of the following?

The location is varied The mandibular incisive canal, mental foramen, and associated neuromuscular bundle exists in different locations and possess many variations. Individual, gender, age, race, assessing technique used, and degree of edentulous alveolar bone atrophy influence these differences. Therefore, the dentist should identify these anatomical landmarks prior to the placement of an implant.

Which of the following medications is contraindicated for patients with congestive heart failure?

The following medications reduce cardiac contractility and exacerbate congestive heart failure: Most Antiarrhythmics (except class III) Calcium channel blockers NSAIDs (except Aspirin) Beta blockers—specifically carvedilol, metoprolol, and bisoprolol—are part of the foundation of care for chronic heart failure. Aspirin is used in patients with congestive heart failure (CHF) due to its antiplatelet activity. Lisinopril is an angiotensin-converting enzyme (ACE) inhibitor and does not adversely affect CHF patients. Furosemide is a loop diuretic that may decrease the edema experienced by CHF patients. Valsartan is an angiotensin-receptor blocker (ARB) and does not adversely affect CHF patients.

When sub gingival margins are required, they should be located at or above the level of:

The gingival sulcus Studies have shown that the restorative margins that are placed below the depth of the gingival sulcus could compromise periodontal health. These restorations are said to violate biological width, the dimension of the junctional epithelium combined with the dimension of the connective tissue attachment. If biological width is violated, crown lengthening procedures may be indicated to prevent periodontal disease and restoration failure The cementoenamel junction is not an accurate landmark for margin guidelines as its position in relation to the gingiva can vary

The lowest point on the mandibular symphysis is:

The menton is the lowest point on the mandibular symphysis

The clinical feature that distinguishes periodontitis from gingivitis is:

The prescience of clinically detectable attachment loss

When making a radiograph, the number of electrons produced determines which of the following? The quality of x-rays produced The quantity of x-rays produced Both the quantity and quality of x-rays produced

The quantity of x-rays produced The quantity of x-rays in the beam depends on the quantity of electrons that the filament releases after being heated.Crookes tubes are able to create free electrons through the ionization of the air present within the tube with the use of a high-voltage direct current.The high-voltage current excites and accelerates the electrons that originated from the cathode. The excitation of electrons causes them to move rapidly, hit the anode, and produce x-rays.The control over the x-rays produced is based on the peak kilovoltage (kVp) and milliamperage (mA) of the x-ray machine.

One advantage of _____ is that it can be reversed one quarter to one half turn after insertion to full depth to reduce stress created at the apical end of the pinhole.

The standard pin

With respect to degenerative changes in the temproromandibular joint as a result of osteoarthritis, positioning appliances are not always advantageous. This is due to the fact osteoarthritis produces chronic, progressive derangements to the joint

The statement and the reason are both correct and related Osteoarthritic changes in the TMJ are some of the most difficult to treat because of the chronic nature of the condition. Positioning appliances provide limited benefits, but cannot overcome the degenerative process. The goal of treatment is to reduce the load on the joint and to provide supportive therapy for symptoms. In many cases the process can be self-limiting, so some clinicians choose only to provide supportive therapy

Which of the following factors typically determines whether or not a dental base material is indicated before a definitive restoration is placed? The tooth that is being restored The biocompatibility The strength The thickness of the remaining dentin

The thickness of the remaining dentin The thickness of the remaining layer of dentin that protects the pulp is the most important thing to consider when selecting the type of base material because some materials can cause pulpal irritation as it approximates the pulp.Dental bases are used in order to provide pulpal protection, specifically on teeth having deep cavities.Dental bases may provide pulpal protection, thermal insulation, and provide sedative effects that give the pulp enough time to recover.The strength of the material and its biocompatibility are also important things to consider when choosing a dental base, but they will only be considered after evaluating the thickness of the remaining dentin.

Which of the following factors does NOT affect the duration of a drug's anesthetic action?

The type of injection administered has an effect on duration. Nerve blocks provide a longer duration of anesthesia to the pulp and soft tissue than infiltrations do. The accuracy of deposition of anesthetic also has an effect on duration. This does not cause a significant difference with certain injections, such as infiltrations, but it is substantial for nerve blocks, such as the inferior alveolar nerve block. Injecting closer to the nerve will provide a longer-lasting anesthetic action. The status of the tissues is influenced by several factors such as pH and vascularity. Inflammation, infection, or pain can reduce duration because there will be more rapid absorption of the anesthetic. Anatomic variation, such as the denser bone or greater distances to the nerves of interest, may notably reduce the duration of the anesthetic. Needle bevel orientation has no effect on anesthetic success or duration

The principal function of endorphins is to inhibit the communication of pain signals. Endorphins may also produce a feeling of euphoria very similar to that produced by other opioids.[4]

The word endorphin is derived from ἔνδον / Greek: éndon meaning "within" (endogenous, ἐνδογενής / Greek: endogenes, "proceeding from within"), and morphine, from Morpheus (Ancient Greek: Μορφεύς, romanized: Morpheús), the god of dreams in the Greek mythology. Thus, endorphin is a contraction of 'endo(genous) (mo)rphin' (morphin being the old spelling of morphine). Endorphins are released from the pituitary gland, typically in response to pain, and can act in both the central nervous system (CNS) and the peripheral nervous system (PNS). In the PNS, β-endorphin is the primary endorphin released from the pituitary gland. Endorphins inhibit transmission of pain signals by binding μ-receptors of peripheral nerves, which block their release of neurotransmitter substance P. The mechanism in the CNS is similar but works by blocking a different neurotransmitter: gamma-aminobutyric acid (GABA). In turn, inhibition of GABA increases the production and release of dopamine, a neurotransmitter associated with pleasure

What is the conversion, in milliliters, for one tablespoon?

There are 15 mL in a tablespoon. A tablespoon is the largest type of spoon in the United States and some parts of Canada.The most common function of a tablespoon is its use as a measure of volume in cooking and dispensing of cough syrup to children. 1 tbsp is equal to 3 tsp.

Which of the following would be the null hypothesis for a study comparing periodontal disease in patients who brushed their teeth and rinsed with a chlorhexidine placebo with a group that brushed their teeth with a chlorhexidine rinse afterwards? There is a difference between the periodontitis of the chlorhexidine group and the placebo group. There is a difference between the placebo rinse and the chlorhexidine rinse. There is no difference between the placebo and the chlorhexidine rinse. There is no difference between the periodontal disease of patients brushing their teeth and rinsing with a placebo versus those brushing and rinsing with chlorhexidine.

There is no difference between the periodontal disease of patients brushing their teeth and rinsing with a placebo versus those brushing and rinsing with chlorhexidine. The null hypothesis states that there is no difference between the results of the control and experimental groups. Chi-squared tests measure the significance of difference between two sets of data. The chi-squared value determines whether the null hypothesis should be accepted or rejected at a determined confidence level. The null hypothesis is a typical statistical theory which suggests that no statistical relationship and significance exists in a set of given single observed variable, between two sets of observed data and measured phenomena.

Each of the following are sources of dimensional change for impression materials EXCEPT:

Thermal Expansion There are five major sources of dimensional change: polymerization shrinkage, loss of a by-product, thermal contraction, imbibition, (Imbibition is a special type of diffusion that takes place when water is absorbed by solids-colloids causing an increase in volume.) and incomplete recovery of deformation because of viscoelastic behavior. Thermal expansion is not a factor while making impressions, because the ambient (Ambient temperature is the air temperature of any object or environment where equipment is stored.) is lower than body temperature. It is thermal contraction that affects the accuracy of impressions when they are removed from the warm oral environment to room temperature when the impression is set. [Imbibition occurs when the impression is exposed to water or disinfectants or high humidity environments over a period of time. Imbibition is the uptake of fluid by the material causing the material to deform to accommodate the excess moisture.

The earliest and most common symptom associated with an inflamed pulp is:

Thermal sensitivity is the earliest and most common symptom of an inflamed pulp. As caries enters the dentin, it begins with a lateral spread at the DEJ. This is due to the increased organic content and the involvement of many dentinal tubules. The Tomes fibers react, causing fatty degeneration then, later, decalcification (sclerosis). As caries progresses, destruction of dentin is followed by the bacterial invasion of the tubules and complete destruction of dentin. Once odontoblasts are involved, pulpal changes occur. Initially there is vascular dilation and local edema. The earliest common symptom of this edema is thermal sensitivity (usually increased and persistent pain on application of cold). Remember: The only reliable clinical evidence that secondary dentin has formed is decreased tooth sensitivity (usually seen a few weeks after placement of a filling). When dentinal tubules become completely calcified, the dentin is insensitive.

Which most accurately describes osseointegrated dental implants? They possess a direct structural and functional connection with bone only at the radiographic level of detection. They create a pseudo-periodontal ligament. They create a junctional epithelium with the surrounding tissue. They are attached directly to living bone as determined by the analysis by radiographs and light microscopy.

They are attached directly to living bone as determined by the analysis by radiographs and light microscopy. Osseointegrated implants are implants that are able to create a direct interface with bone cells after they grow and interlock, without any interposing layer of soft tissue. In the process of osseointegration, the bone directly attaches to the implant material because the titanium oxide fuses with the surrounding bone. In osseointegration, no scar, fibrous, or connective tissues get in between the bone and the implant material. Osseointegration can be verified through microscopic and radiographic images as well as by tapping the implant with a mirror and hearing a sharp metallic ring (not a dull thud). The use of titanium for osseointegration was discovered by Per-Ingvar Branemark in 1952.

Which of the following statements most accurately describes why highly polar pharmaceuticals work more rapidly?

They are less lipid-soluble. The low lipid-solubility of highly polar medications makes them less permeable to cell membranes and increases their absorption in the bloodstream. High absorption in the bloodstream increases their bioavailability and efficacy, as well.

Each of the following are characteristics of addition silicone (polyvinyl siloxane) impressions EXCEPT one. Which of the following is the EXCEPTION? They can be poured 24 hours after being made and still remain accurate. They produce ethyl alcohol during the setting reaction. They remain accurate after the first pour. They can rebound from undercuts without permanent deformation.

They produce ethyl alcohol during the setting reaction. Addition silicones are a class of impression materials that remain accurate for several days, even after the initial pour.Addition silicones can accurately copy the details of tooth preparations while displaying low dimensional change once the material has set.Condensation silicones (not addition silicones) release alcohol as a by-product during the setting reaction.

Each of the following statements regarding sealants is true EXCEPT one. Which is the EXCEPTION? They are considered a noninvasive procedure. They perform as a physical barrier. They should be used on all patients' teeth. They prevent biofilm from collecting within a pit or fissure.

They should be used on all patients' teeth. Sealants should not be used on all patients' teeth. Sealants are indicated for either preventive or therapeutic uses, depending upon patient's caries risk, tooth morphology, or presence of incipient enamel caries. Only caries-free pits and fissures or incipient lesions in enamel not extending to the dental-enamel junction (DEJ) are recommended for treatment with pits and fissure sealants. Sealants have these advantages: They perform as a physical barrier. They prevent biofilm from collecting within a pit or fissure. They sealant placement is considered as a non-invasive procedure.

Women taking oral contraceptives are more susceptible to which of the following potentially fatal conditions? Uterine neoplasia Thromboembolic disorders Hypotension Hepatotoxicity Clitoral enlargement

Thromboembolic disorders Thromboembolisms may occur in women taking oral contraceptives due to an increase in the activity of fibrinogen and coagulation factors VII, VIII, and X due to the increase in the blood levels of estrogen and progesterone. Oral contraceptives also induce increased resistance to activated protein C. Thrombus formation may block blood flow and result in several debilitating conditions like myocardial infarction and strokes.

Which of the following is the material most commonly used to manufacture dental implants? Stainless steel Titanium Porcelain Hydroxyapatite

Titanium Titanium is a biocompatible metal commonly used to fabricate dental implants. Titanium is used because it is able to undergo osseointegration, meaning that there is a direct structural and functional connection between ordered, living bone and the surface of an implant. Some bio-ceramic implants have been experimented with but are not widely used.

Which is the goal of alveoloplasty procedures? To make sure that the canine eminences are sharp to increase retentiveness To make the residual ridge as broad as possible even if undercuts are still present To make sure that undercuts that interfere with the denture seating are removed To make sure that the alveolar ridges are narrow to accentuate the ridge height

To make sure that undercuts that interfere with the denture seating are removed Alveoloplasty procedures are performed to recontour or reshape the alveolar bone to make it more ideal for the fabrication and use of prosthodontic appliances. Alveoloplasty procedures promote better denture support and stability by creating an ideal alveolar ridge form. Alveoloplasties help to establish healthy and stable alveolar ridges by removing the undesirable undercuts that impede proper seating of dentures; they also round off knife-edge alveolar ridges to provide better support when used.

Which is the best reason why the liquid component of a cement should be dispensed immediately before mixing is commenced? To reduce the temperature influence of the mixing slab To allow tempering of the powder by the mixing slab To prevent evaporation of the volatile components To avoid absorption of moisture from the air To avoid spreading over a large area of the slab

To prevent evaporation of the volatile components The liquid component of a cement is very volatile and dispensing it too early before mixing commences will lead to evaporation of some of it, causing incomplete wetting of the cement powder during spatulation.Having deficient liquid in proportion to cement powder results in a grainy consistency and premature setting of the cement mixture.

The main cause of halitosis is:

Tongue coating Tongue coating (TC), a grayish-white deposit on the tongue, is the main cause of intra-oral halitosis (IOH), a socially unacceptable condition. Halitosis tongue coating samples with a higher microbial richness than healthy samples

Which of the following is the most common iatragenic side effect of sodium hypochlorite? Reaction with chelating agents Toxicity to vital tissue Corrosiveness to endodontic files Tendency to bleach tooth structure Smell

Toxicity to vital tissue Sodium hypochlorite is a good disinfectant that is used in dentistry as an endodontic canal irrigant at diluted concentrations. It is often used to disinfect gutta-percha points prior to placement and obturation of the canal. In undiluted or excessive amounts, sodium hypochlorite is toxic to vital tissues and can cause periapical tissue damage and pain.

Penicillin's inhibition of which of the following enzymes is responsible for inhibiting cell wall synthesis?

Transpeptidase Transpeptidase is responsible for catalyzing the peptide cross-linking of the peptidoglycan in bacterial cell walls. Penicillins and cephalosporins inhibit the transpeptidase enzyme. β-lactamase is an enzyme responsible for cleaving penicillins, making them ineffective against the bacteria.

An apically repositioned flap on palatal mucosa is achieved by which of the following? Utilizing only a continuous suture type for closure of the surgical site Utilizing only a sulcular incision in the initial incision Trimming the flap margin to the proper length during the procedure Avoiding an initial incision that contacts bone

Trimming the flap margin to the proper length during the procedure Apically repositioned flap margins on the palatal aspect need to be placed exactly to its predetermined position during the procedure because the palatal mucosa has no unattached mucosa that can be displaced.The palatal mucosa is firmly attached to the palatal bone and due to its thick nature displacement is not possible.An apically repositioned flap usually requires displacement of the gingiva to a more apical position, requiring vertical releasing incisions except when the flap is made on the palate.

Cigarette smoking decreases gingival inflammation and bleeding upon probing.

True

Post obturation pain is though to be caused by:

Unknown The etiology for post-obturation pain is largely unknown. Inadequate obturation is not an etiology for post-obturation pain. In fact, the level of obturation tends to cause pain when it is overfilled or overextended. Residual bacteria may be the culprit for residual infection but it is not a known cause of post-obturation pain Lateral canals are very difficult to clean and shape. Lack of cleaning and shaping directly of the lateral canals is not likely a cause of post obturation pain.

Techniques for administering local anesthetic in a six year old should include

Using age appropriate language Using topical anesthetic Shielding of the child's eyes from the syringe DO NOT use a rapid injection technique as this would increase the pain response

Which of the following methods best establishes plaque control? Using antibiotics (penicillin, tetracyclines) Using chemical means (mouthwashes) Using mechanical means (brushing and flossing) Using a surfactant to prevent bacteria from adhering to teeth

Using mechanical means (brushing and flossing) Mechanical disruption (brushing and flossing) is the best way to stop the growth of plaque and other bacterial pathogens. Initial bacteria colonize on the tooth pellicle. Disrupting newly formed plaque helps prevent calculus and dental caries. Antibiotics kill bacteria, but they are not intended for plaque control. Excessive antibiotic use may cause bacterial death in the oral cavity, leading to an imbalance in normal mouth flora and allowing fungi such as Candida to take over.

During treatment, the patient complains of extreme sensitivity on tooth 14. Which of the following cranial nerves carries pain impulses from this tooth? V1 V3 V2 XII X

V2 The trigeminal nerve is cranial nerve V, which divides into three branches: V1, the ophthalmic branch, is a sensory nerve that leaves the skull through the superior orbital fissure. It supplies the skin above the eye, the forehead, and the skin around the eyes. V2, the maxillary branch, is a sensory nerve that leaves the skull through the foramen rotundum. It supplies the maxillary teeth, the oral and nasal cavity, the skin of the cheek, and the temporal region. V3, the mandibular branch, is a sensory and motor nerve that leaves the skull through the foramen ovale. It supplies the lower teeth, the anterior portion of the tongue, the skin of the lip and chin, and the muscles of mastication. Cranial nerve X is the vagus nerve. It has both sensory and motor actions and supplies the muscles of the pharynx and larynx, the muscles of the soft palate, the smooth muscle and glands of the body, the cardiac muscle, and the base of the tongue and epiglottis. Cranial nerve XII is the hypoglossal nerve, which is a motor nerve. It serves the muscles of the tongue and exits the base of the skull through the hypoglossal canal.

An index is a scale by which factors are measured numerically according to specific criteria. Which of the following properties of an ideal index ensures that the index measures what it is intended to measure?

Validity Validity: the index as a condition must measure what it is intended to measure

Which of the following is the most dangerous effect of a digoxin overdose?

Ventricular fibrillation

Which of the following is the most dangerous effect of a digoxin overdose?

Ventricular fibrillation Digoxin toxicity can induce ventricular fibrillation, leading to incoordination of ventricular cardiac muscle contraction. A very narrow window between digoxin's therapeutic effect and its toxic effect (narrow therapeutic index) makes toxicity fairly common. Toxicity creates various electrolyte abnormalities and imbalances, affecting the normal rhythm of the heartbeat. Toxicity also leads to bradycardia, a prolonged PR interval, accelerated rhythm, and possible bidirectional ventricular tachycardia.

Which of the following cancer diagnoses has the best 5-year survival rate? Stage 1 well-differentiated squamous cell carcinoma involving the right lateral tongue Stage 1 well-differentiated squamous cell carcinoma involving the soft palate Stage 1 well-differentiated squamous cell carcinoma involving the floor of the mouth Verrucous carcinoma of the floor of the mouth Verrucous carcinoma of the right lower posterior vestibule

Verrucous carcinoma of the right lower posterior vestibule Verrucous carcinoma is a slow-forming tumor that is usually seen along the mandibular vestibule, buccal mucosa, floor of the mouth, and mandibular gingiva. It is usually seen among people who have a long history of chewing tobacco and appears as a white, papillary, exophytic mass that is nonulcerated. Prognosis of vestibular verrucous carcinoma is better than in those found in the floor of the mouth because there are many other important intraoral structures underneath the floor of the mouth to which the carcinoma may extend. With complete surgical removal of the carcinoma, patients become disease free over a 5-year period approximately 90% of the time. Cancers of the tongue and soft palate typically have a worse prognosis than those found in the vestibule because there are a lot of important structures there, as well as better blood flow, which may lead to quicker tumor growth and spread.

A unilateral angle mandibular fracture that results in upward and medial displacement of the proximal segment during contraction of the muscles attached to the ramps (medial pterygoid, temporal, and master) is also known as a:

Vertically unfavorable and horizontally unfavorable fracture If the horizontal direction is downward and posterior, the active pull of the posterior elevator muscles (masseter, pterygoids, temporals muscles) will displace the proximal segment superiorly

A 58 year old male patient presents with a small blister like lesion of circumscribed border. It is slightly elevated from the surrounding mucosa and appears to contain clear fluid. The appropriate pathological term for this lesion would be a:

Vesicle A vesicle is a small blister, or a small, circumscribed elevation of skin or mucosa containing serous fluid Pustule: a small, cloudy, elevated, circumscribed, pus containing vesicle on the skin or mucosa Papule: a small, elevated, circumscribed, solid, palpable mass of the skin or mucosa Nodule: a large, elevated, circumscribed, solid palatable mass of the skin or mucosa

Which is least likely to occur following an intra-arterial injection of anesthetic? Aspirated blood is bright red in color. Injection of the test dose elicits acute pain. The needle moves with the rhythm of the heartbeat. Penetration of the vessel is met with resistance and elicits pain. Vessels tend to collapse and obstruct.

Vessels tend to collapse and obstruct. No obstruction or collapse of arteries is experienced during administration of an intra-arterial injection. Resistance is met during administration due to blood flow. Aspirated blood is bright red in color due to the high concentration of oxygen in blood passing through the arteries. Intra-arterial injections are painful in nature.

Each of the following is an adverse effect of long-term tetracycline therapy EXCEPT one. Which is the EXCEPTION?

Visual disturbances Tetracycline use is not associated with visual disturbances. Tetracycline is incorporated into the tooth structure during the mineralization of the tooth because it forms a chelate with calcium. Long-term tetracycline use is associated with the following: Diarrhea Photosensitivity Superinfection

Which of the following CANNOT be used to accurately calculate pediatric drug dosages? Clark rule Vital signs Body weight (mg/kg) Body surface area

Vital signs Inaccurate pediatric dosage has been found to be the leading cause of medical problems in children. Calculation of pediatric dosage is often based on the following: Age Body weight Body surface area Stage of development of the child The following two methods are used to calculate the pediatric dosage: Clark rule: adult dose × (weight in lbs. ÷ 150) = child's dose Young rule: adult dose × (age ÷ (age + 12)) = child's dose

Which of the following has the greatest impact on the success of a pulpotomy performed on a primary molar? Age of the patient G. V. Black classification of coronal caries Absence of root resorption Vitality of the tooth pulp

Vitality of the tooth pulp The success of a pulpotomy procedure lies on the ability of the remaining healthy pulp to remain vital and not irreversibly inflamed after the procedure. Partial pulpotomy is a procedure that removes only the portion of the coronal pulp that is infected and inflamed, which keeps the radicular portion vital. Covering the pulpotomized tooth with a stainless steel crown (SSC) may increase the chance for success of treatment.

When differentiating between an acute periodontal abscess and an acute apical abscess, which test is the most useful?

Vitality testing Generally, periodontally involved teeth will remain vital while endodontically involved teeth do not. For an acute apical abscess to be present, the pulp must have become non-vital or necrotic

A moderately developmentally disabled five-year-old child cries excessively and physically resists during an emergency dental visit. Which of the following methods of patient management is used by the dentist in this situation? Voice control Physical restraint Hand-over-mouth technique General anesthesia

Voice control The voice control method should be used in a moderately developmentally disabled five-year-old child who is crying excessively and resisting physically during an emergency dental visit.Voice control will suffice in making the child cooperate.General anesthesia is not an option during an emergency visit.Physical restraint or hand-over-mouth technique can worsen the situation.Most states prohibit the use of hand-over-mouth exercise (HOME) technique.

Trabeculation

What does Trabeculated mean in medical terms? That thickening of the bladder walls is called trabeculation. When your bladder walls get too thick, they lose the ability to expand and contract, making it hard for your body to expel urine. Bladder trabeculation can be caused by other health conditions, such as blood clots.

Relining a removable partial denture (RPD) is indicated when which of the following is present? When the acrylic resin teeth are abraded and there is a loss of the vertical dimension of occlusion When the distal extensions are depressed Soreness on the crest of the alveolar ridges Loss of retention

When the distal extensions are depressed Indirect retainers do not seat properly when the distal extensions are depressed due to the loss of support along the distal alveolar ridge. The presence of bone resorption causes instability to the removable partial denture. The improper seating of indirect retainers indicates that there is already a noticeable amount of bone loss along the residual bone underlying the distal extension base and that the denture has to be relined. The indirect retainers are designed to provide stability, limit the movement of the denture toward the tissue, and prevent the destructive antero-posterior twisting of dentures during function. Indirect retainers may act as a fulcrum when the distal extension base is no longer adapted to the residual ridge and cause the instability and displacement of denture against forces acting on it.

Bitewing radiographs are indicated on a clinically caries-free child when which of the following occurs? When the permanent first molars have erupted During the first dental visit When the spaces between the posterior teeth have closed When the entire primary dentition is completely erupted

When the spaces between the posterior teeth have closed The best time for making the first bitewing radiograph of a clinically caries-free child is after the closure of spaces between the posterior teeth.Bitewing radiographs are best used for the evaluation of interproximal spaces of posterior teeth.Periapical radiographs are most commonly used for assessing the presence or absence of lesions around the apex of teeth.Primary lesions most commonly occur in the bifurcation or trifurcation areas in primary teeth, so bitewing radiographs are considered the gold standard for primary posterior teeth.

When designing a mandibular removable partial denture (RPD), a lingual bar is contraindicated if which of the following is present? When Kennedy class III conditions are present When there is insufficient space to the floor of the mouth When the remaining anterior teeth are mobile When the remaining teeth are spaced widely

When there is insufficient space to the floor of the mouth The space between the floor of the mouth and the marginal gingiva is the primary factor when determining the type of major connector to be used for the lower arch. The lingual bar major connector requires a minimum of 4 mm in cervicoocclusal height and should be approximately 4 mm away from the marginal gingiva. Because of a very limited space, a lingual plate of adequate width is preferred over a lingual bar.

Primate spaces

Within a primary dentition, primate spaces may occur between the primary teeth; a space is noted between the maxillary lateral incisor and the canine, and between the mandibular first molar and canine. Primate spaces are located between the lateral incisors and canines in the upper arch, and between the first molars and canines in the lower arch, whereas developmental spacing (also called interdental spacing) is found between the incisors in both the upper and lower arches.

Which of the following agents is prescribed to treat human immunodeficiency virus (HIV) patients? Acyclovir Zidovudine Ribavirin Amantadine lsoniazid

Zidovudine Zidovudine (AZT) is a nucleoside analog reverse-transcriptase inhibitor (NRTI) antiretroviral drug used for the treatment of HIV/aquired immunodeficiency syndrome (AIDS). AZT is the first U.S. government-approved treatment for HIV therapy and can also be used to prevent HIV transmission, such as from mother to child during the period of birth or after a needle-stick injury. If used by itself in HIV-infected patients, AZT safely slows HIV replication but generally does not stop it entirely. Acyclovir is a guanoside analogue that is used in the treatment of herpes simplex infections. Amantadine is an anti-Parkinson drug that is no longer recommended for use as an antiviral agent against influenza. Isoniazid is an antibiotic used to treat M. tuberculosis infections.

During a large mesio-occlusal-distal (MOD) caries removal, the patient loses consciousness. The doctor's first step is to assess the patient's airway and pulse. pupil size. blood pressure and airway. blood pressure and pulse.

airway and pulse. When a person suddenly loses consciousness, it is important to assess the ABCDs: Airway: Check for obstruction. Breathing: Check for rise and fall of the chest. Circulation: Check the patient's pulse. Diabetes: Check blood sugar if the patient is diabetic. It is important for doctors not to panic when a patient loses consciousness; they must think clearly and decisively to properly restore the patient to health and consciousness.

Which of the following are NOT clinical characteristics of condyloma acuminatum?

[Genital warts (condyloma acuminata) are caused by the human papillomavirus (HPV), which has over 100 different strains.] Tends to be smaller papilloma The condyloma tends to be larger than the papilloma. The average lesion size is 1.0 to 1.5 cm, but oral lesions as large as 3 cm have been reported. [Condylomata are usually diagnosed in teenagers and young adutlts] [this typical condyloma appears sessile, pink and well-demarcated] [The condyloma is characteristically clustered with other condylomata]

ulcerative colitis

a chronic condition of unknown cause in which repeated episodes of inflammation in the rectum and large intestine cause ulcers and irritation

Methacholine is related to acetylcholine as an agonist. an antagonist. a congener. an isomer. a vehicle.

a congener. Congeners are substances that are closely related in composition, function, or origin. Methacholine is a synthetically made choline ester that is more predisposed to function as a muscarinic receptor agonist within the parasympathetic nervous system. Methacholine and acetylcholine are both cholinergic agonists, but methacholine's duration of action is longer than that of acetylcholine.

A central nervous system drug that induces sleep resembling natural sleep from which a patient can be aroused is

a hypnotic. Hypnotic drugs are psychoactive medications that are mainly used to induce sleep and are often used to treat sleep disorders like insomnia as well as surgical anesthesia. Prolonged use of hypnotic drugs can cause some form of dependence or habit, so their use must be limited to a short period of time. Benzodiazepines and non-benzodiazepine drugs are the most common hypnotic drugs present on the market today. Benzodiazepines work by enhancing a very important neurotransmitter called GABA (gamma-aminobutyric acid) at the GABA A receptor. This results in the sedative, hypnotic (sleep-inducing), anxiolytic (anti-anxiety), anticonvulsant, and muscle relaxant properties for which the drugs are prescribed.

In comparison to codeine, morphine is

a more potent analgesic Morphine is a very potent substance, considered the gold standard of opioid analgesics in alleviating moderate to severe pain. It is very useful in medicine because its analgesic property is used for managing pain caused by myocardial infarction and labor. Codeine is less potent compared with morphine and is used to alleviate mild and moderate pain.

An electric dental pulp vitality test (EPT) is contraindicated just after a traumatic injury mainly because the pulp always tests negative. the EPT can give a false-positive reading. the pulp is necrotic and should undergo a root canal procedure. a negative response may be temporary because the blood supply was interrupted. a negative response may be temporary because the neural transmission was interrupted

a negative response may be temporary because the neural transmission was interrupted. Electric pulp testing within an hour of a luxation injury is contraindicated because of the inconsistent findings. A positive response from electric pulp testing is indicative of retained vitality and does not differentiate between the following: Normal pulps Reversible pulpal inflammation Irreversible inflammation A negative response may not always indicate pulpal necrosis, because many teeth take about two months to respond to electric pulp testing again. Electric pulp testing gives the status of the pulpal nerve fibers, not the blood supply of the tooth. Pulpal response to electric current only suggests the presence of some viable nerve fibers that are capable of responding to electric current.

The first-line treatment indicated to reverse tricyclic antidepressant overdose is

activated charcoal. Initial treatment of an acute overdose includes gastric decontamination of the patient by administering activated charcoal lavage, which absorbs the drug in the gastrointestinal tract. Physostigmine is a reversible cholinesterase inhibitor that can reverse the effect of tricyclic antidepressant overdose in five to twenty minutes. However, it increases the risk of cardiac toxicity and may induce seizures. Tricyclic antidepressant (TCA) drugs enhance the anticholinergic activity of the body. Physostigmine is also used to treat atropine overdose, myasthenia gravis, and glaucoma.

Which of the following is an inflammatory reaction to pulpal infection and necrosis characterized by rapid onset, spontaneous pain, tenderness of the tooth to pressure, pus formation and swelling of associated tissues?

acute apical abscess

The drug category contraindicated in glaucoma patients is

anticholinergics. Anticholinergic agents like atropine are contraindicated in patients predisposed to narrow-angle glaucoma, because they prevent evacuation of aqueous humor through the canal of Schlemm and elevate intraocular pressure by increasing pupillary diameter. Atropine causes pupil dilation by paralysis of the cilliary muscle, preventing lens accommodation, and prevents action of acetylcholine on muscarinic receptors M1-M5, causing an increase in sinoatrial (SA) node output and an increase in heart rate. Glaucoma is a pathology in which an increased fluid pressure in the aqueous humor of the eye causes pain and can result in blindness if not treated.

The therapeutic function of nalidixic acid is best characterized as an antiviral agent. antifungal agent. antityphoid agent. antituberculous agent. antiseptic for the urinary tract.

antiseptic for the urinary tract. Nalidixic acid is a quinolone antibiotic effective against Gram-positive and -negative bacteria and is most commonly used to treat urinary tract infections. Nalidixic acid is considered bacteriostatic in low concentrations and bacteriocidal in higher concentrations. Nalidixic acid is no longer used because newer agents with fewer side effects, like convulsions or hyperglycemia, are available.

The therapeutic function of ethambutol is best characterized as an antiseptic for the urinary tract. antityphoid agent. antiviral agent. antifungal agent. antituberculous agent.

antituberculous agent. Ethambutol is a bacteriostatic antimycobacterial drug prescribed to treat tuberculosis and is usually co-administered with isoniazid, rifampicin, or pyrazinamide. Ethambutol is bacteriostatic against actively growing tuberculosis (TB) bacilli and functions by obstructing the formation of the cell wall.

The therapeutic function of isoniazid is best characterized as an antityphoid agent. antituberculous agent. antifungal agent. antiviral agent.

antituberculous agent. Isoniazid is the first-line medication in the prevention and treatment of tuberculosis. It is a pro-drug that must be activated by bacterial catalase-peroxidase enzymes. Isoniazid inhibits the synthesis of mycolic acid component of the mycobacterial cell wall.

Methomoglobinemia

autosomal rec. disorder: maccumulation of methemoglobin in blood, unable to clear abnormal blue protein from blood causing skin to appear blue. "blue Fugates of Troublesome Creek, KY'. enzyme diaphorase (chrom 22) is lacking. treated by methylene blue injections Methemoglobinemia is a condition with life-threatening potential in which diminution of the oxygen-carrying capacity of circulating hemoglobin occurs due to the conversion of some or all of the four iron species from the reduced ferrous (Fe2+) state to the oxidized ferric (Fe3+) state. Ferric iron is unable to bind and transport oxygen. Increased levels of methemoglobin results in functional anemia

The drug category contraindicated in glaucoma patients is belladonna alkaloids. catecholamines. epinephrine. norepinephrine. oxytocin.

belladonna alkaloids. Belladonna alkaloids like atropine are derived from the Atropa belladonna, or deadly nightshade, plant and function as anticholinergic drugs. Glaucoma is a pathology in which an increased fluid pressure in the aqueous humour of the eye causes pain and can result in blindness if not treated. Belladonna alkaloids are contraindicated to patients predisposed to narrow-angle glaucoma, because they prevent evacuation of aqueous humor through the canal of Schlemm and elevate intraocular pressure by increasing pupillary diameter. Atropine causes pupil dilation by paralysis of the ciliary muscle, preventing lens accommodation. It prevents action of acetylcholine on muscarinic receptors M1-M5, causing an increase in sinoatrial (SA) node output.

Acrylic teeth are used primarily in removable partial dentures because they are wear-resistant. stain-resistant. aesthetically preferred. better retained in acrylic resin.

better retained in acrylic resin. Acrylic teeth are retained better than other denture teeth (porcelain and metal), because they are able to bond to the acrylic resin of the denture base. Acrylic teeth are not stain-resistant and are not as color-stable as porcelain teeth. Acrylic teeth are also less wear-resistant than porcelain or metal teeth. Porcelain teeth are considered to have higher long-term aesthetics, because they have the same translucency as acrylic teeth, but they also have more stain and wear resistance and make the patient have less of a "bruxism look" over time.

Tetracycline MOA

binds to the 30S ribosomal subunit to inhibit protein synthesis,

The _____ of the gingival marginal trimmer corresponds to the nib of the _____? blade; hatchet shank; explorer blade; periodontal probe shaft; forceps blade; condenser

blade; condenser Explanation The blade of the gingival marginal trimmer corresponds to the nib of the condenser. Gingival marginal trimmers are cutting instruments, and condensers are non-cutting instruments. In non-cutting instruments, the blade is replaced by a nib or point. The working ends or nibs of the condensers may be of any shape, but they are usually round with flat ends. Triangular, rectangular, or diamond-shaped nibs are also used.

The mechanism by which propranolol relieves angina is by

blocking the β-adrenergic receptors of cardiac muscle. Propranolol is the first nonselective beta blocker developed and is used to treat hypertension, anxiety, and panic attacks. Propranolol blocks the action of epinephrine and norepinephrine on both β1- and β2-adrenergic receptors and exhibits little intrinsic sympathomimetic activity. Due to its high penetration across the blood brain barrier, lipophilic beta blockers such as propranolol and metoprolol are more likely to cause sleep disturbances such as insomnia, vivid dreams, and nightmares.

Anterior occlusion is determined by both horizontal and vertical overlap of anterior teeth. the vertical overlap of anterior teeth only. the horizontal overlap of anterior teeth only. cusp inclination. cusp height.

both horizontal and vertical overlap of anterior teeth. Anterior occlusion is determined by both the horizontal and vertical overlap of anterior teeth.Overbite is the vertical overlap of maxillary incisors over the mandibular incisor during maximum intercuspation. It ranges from 3-5 mm, depending on tooth size and the occlusion type of the patient Overjet is the horizontal overlap of maxillary incisors over the mandibular incisors during maximum intercuspation. It ranges from 2-4 mm, depending on the type of skeletal facial profile and the occlusion type of the patient

Composite resin restorations release fluoride. chemically bond to dentin. can be placed and finished at the same appointment. have a thermal coefficient of expansion similar to amalgam.

can be placed and finished at the same appointment Composite resin restorations are popularly used because of their: High esthetic value Ease of manipulation Durability Glass ionomer cement is a restorative material that: Releases fluoride into the surrounding tooth structure Chemically bonds to the tooth structure Is less resistant to wear than composite

The most medically accepted definition of alcohol abuse is when someone ingests enough alcohol to raise their blood alcohol content over the legal limit. becomes intoxicated on a regular basis. experiences symptoms of withdrawal when alcohol is not ingested. consumes alcohol enough that it puts their relationships, job, or health in jeopardy.

consumes alcohol enough that it puts their relationships, job, or health in jeopardy. Alcohol abuse is defined as "alcohol intake that causes harm to the user, to social and interpersonal relationships, and to the capability to perform duties at work." Alcohol abuse manifests differently for both acute and chronic effects on the central nervous system, including poor judgment, tipsiness, irritability, and sleeping difficulties. If not prevented, alcohol abuse may lead to alcohol dependence.

Which of the following injuries would cause a patient to deviate toward the side of injury when protruding? Select all that apply. damage to the lateral pterygoid muscle ankylosis of the condyle condylar hyperplasia unilateral condylar fracture

damage to the lateral pterygoid muscle• ankylosis of the condyle• unilateral condylar fracture The mandible will also deviate toward the side of injury with: Ankylosis of the condyle: the most common cause of TMJ ankylosis is trauma A unilateral condylar fracture The mandible will deviate away from the affected side with: Condylar hyperplasia: malocclusion is also a common occurrence with this injury Remember: The lateral pterygoids (right and left) acting together are the prime protractors of the mandible. Important: In addition to opening and protruding, the lateral pterygoids move the mandible from side to side. For right lateral excursive movements, the left lateral pterygoid muscle is the prime mover and vice versa. A patient who sustained a subcondylar fracture (unilateral condylar fracture) on the left side would be unable to deviate the mandible to the right (as stated above, the mandible will deviate toward the side of injury with a unilateral condylar fracture, this patient would not be able to deviate the mandible to the right). This is normally treated by a closed procedure involving intermaxillary fixation. This procedure immobilizes the concomitant fractures and corrects the displacement of the jaws associated with the condylar fracture, thereby correcting the shift of the midline toward the side of the fractured condyle and the slight premature posterior occlusion on that side.

The most important reason zinc oxide eugenol cements are better than zinc phosphate cements for crown delivery is because zinc oxide cements demonstrate All answers are correct. decreased film thickness. decreased solubility in saliva. decreased potential to irritate the pulp.

decreased potential to irritate the pulp. Zinc oxide eugenol cement is a dental material capable of providing sedative effects to the pulp. Eugenol, "oil of cloves," provides soothing relief to hyperemic pulp.Zinc phosphate cement uses phosphoric acid for its chemical reaction and is irritating to the pulp.

The mechanism by which succinylcholine inhibits neuromuscular transmission is

depolarizing the motor endplates of skeletal muscle. Succinylcholine is a depolarizing neuromuscular-blocking drug that causes depolarization along the motor endplate. It acts like acetylcholine because it binds to the acetylcholine receptor and causes muscular twitches or contractions during phase I. In phase II, due to the repetitive excitation and depolarization, the motor endplates become resistant to acetylcholine activation. The resistance of motor endplates from acetylcholine activation prevents neuromuscular transmission, preventing muscle contraction.

The quaternary ammonium group in the structure of an anticholinergic agent allows for it to act as a ...

direct nicotinic antagonist. Nicotinic antagonists are a type of anticholinergic agent that inhibit the action of acetylcholine (ACh) at nicotinic ACh receptors and are used for peripheral muscle paralysis during surgery. Quaternary ammonium compounds can result in a range of health effects, from mild skin and respiratory irritation to severe caustic burns on the skin and gastrointestinal lining, hypotension, and death. They are the chemical group thought to be responsible for anaphylactic reactions that occur with use of neuromuscular-blocking drugs for general anesthesia in surgery.

The most superior of all other retrofilling material - mineral trioxide aggregate (MTA) has all of the following advantages, EXCEPT two. Which TWO are not properties of MTA?

easy to manipulate• short setting time The main ions found in MTA are calcium and phosphorus. MTA has a high pH so it induces hard tissue formation. MTA has superior sealing ability and is not adversely affected by blood contaminants. It also causes only low levels of inflammation because it forms fibrous connective tissue and cementum when in contact with the periodontium. Note: MTA is difficult to manipulate and has a long setting time. Despite these disadvantages, it's the material of choice today. Note: MTA has good antibacterial properties, probably related to high pH. A retrofilling (also called a reverse filling or retrograde amalgam filling) is placed to seal the apical portion of the root canal. This procedure is used when an apicoectomy alone will not yield a good result. Whenever there is any chance whatsoever that an apical seal may be faulty, a reverse filling material must be placed. For example, if the root canal appears calcified, it would be impossible to obturate most of the canal and get a seal. If just the root apex were cut off (apicoectomy), the incompletely filled canal might act as a source of reinfection. To prevent this after the root tip is resected, the foramen is found, enlarged, and filled with a zinc-free amalgam to create a seal. An apicoectomy (root-end resection) is a procedure where the buccal tissue is flapped back, the buccal bone around the apex is removed, the root apex is removed, and the area is curetted out. Indications for apicoectomy: 1) A reverse filling needs to be placed 2) It is necessary to gain access to an area of pathosis 3) The poorly filled apical portion of the root is to be removed to the level of canal obliteration. Teeth that have posts in them and need to be retreated are the most common reason for an apicoectomy and a retrograde filling. Remember: Periapical curettage is the same procedure as an apicoectomy (as far as flap and removal of buccal bone) but without removing the root apex. Removal and examination of the diseased tissue and determination of the extent of the lesion are the objectives of apical curettage.

Another term for the maximal effect of a drug is

efficacy. Efficacy is the maximal or ceiling effect of a drug. It is the capacity of a drug to produce its pharmacologic effect in the body. In other words, it is the maximum achievable result from a drug. Specificity describes when a molecule has a certain affinity for a specific receptor or reacts with certan substances (e.g., antibody or antigen). Potency describes a measure of drug activity expressed in terms of the amount required to produce an effect of given intensity. Agonism describes when a molecule binds to and activates a receptor. Duration describes the length of time the drug has a pharmacological effect.

The most accurate description of the purpose of biofeedback is that it is meant to distract and engage the patient in active coping tasks. relax and semi-hypnotize the patient. enable the patient to gain control of certain physiological functions. reduce cognitive dissonance. stimulate the sympathetic nervous system.

enable the patient to gain control of certain physiological functions. Biofeedback is a type of therapy in which patients are taught ways to achieve better health through controlling certain regulatory and involuntary processes that occur within their body. Biofeedback is conducted in three ways: Electromyography functions by measuring the tension within a muscle. Thermal biofeedback focuses on the skin's temperature. Neurofeedback promotes a deeper understanding of how the brain waves of the patient function. Calmness and relaxation is said to be the key to practicing and successfully initiating biofeedback therapy. Through biofeedback, there is a possibility of controlling some of the body's involuntary processes, such as blood pressure.

Internal bleaching of an inadequately obturated tooth can lead to Select All That Apply external cervical root resorption. fracture. symptomatic apical periodontitis. asymptomatic apical periodontitis. discoloration.

external cervical root resorption. symptomatic apical periodontitis. Careful examination of a tooth is necessary before internal bleaching, since the method requires healthy periodontal tissues and a root canal that is properly obturated to prevent the bleaching agent from reaching the periapical tissues, and an effective gingival barrier which prevents the bleaching agents from leaching into the periodontium. Bleaching agents may leech into the root canal treated tooth and beyond, into the apical portion, causing periapical swelling or apical pain. It may also cause external cervical root resorption. The underlying mechanism for this effect is unclear, but it has been suggested that the bleaching agent reaches the periodontal tissue through the dentinal tubules and initiates an inflammatory reaction. This risk may be prevented or reduced by having a well-condensed obturation in the middle/coronal third of the root, and by placing a barrier on top of the obturation to further seal the dentinal tubules. The goal of canal obturation is to produce a hermetic seal of the apical terminus and prevent re-infection of the periapical structures.

Elastomeric impression material works best when the tooth preparation is covered with a thin layer of water-soluble lubricant. covered with a thin layer of petroleum gel. covered with a surface tension-reducing agent. free of surface moisture.

free of surface moisture. The four types of elastomeric impression materials are polysulfides, condensation silicones, addition silicones, and polyethers.Each material differs in the setting mechanism and the physical and chemical properties.Surface moisture is not good for elastomeric impression materials because the moisture decreases the accuracy of the impression.

Porcelain fused to metal (PFM) crowns with a high-noble base typically have the highest percentage of platinum. palladium. silver. gold. chrome-cobalt.

gold. High-noble alloys contain a minimum of 60% noble metals, with gold approximating 40% of the noble metal content.High-noble metals are highly resistant to corrosion and oxidation.High-noble metals used most in the field of dentistry include gold, palladium, and platinum.

In reference to the Spaulding classification system for contaminated items, semi-critical items should be reprocessed by washing with a chlorhexidine gluconate disinfectant. exposure to ethylene oxide. a spray-wipe-spray technique. immersion in a tuberculocidal disinfectant. heat sterilization if the material is heat stable.

heat sterilization if the material is heat stable. Semi-critical items are instruments that do not penetrate soft tissues or bone but are used inside the mouth and contact the mucosa. These instruments must be sterilized after every use or must undergo high-level disinfection if they are not heat stable.

The cells most likely to be damaged first by radiation are neural. muscular. hematopoietic. cartilagenous. endothelial.

hematopoietic. Hematopoietic cells are sensitive to radiation because they are actively dividing and undergoing several cellular processes to form many different cell types, like red and white blood cells and their derivatives.Hematopoietic cells give rise to several types of cells in the body, including myeloid and lymphoid cells.Hematopoietic cells produce blood cell components through continuous cellular processes that can be disrupted by ionizing radiation.

Cycloplegia, or paralysis of the ciliary muscle of the eye, can be caused by pilocarpine. homatropine bromide. amphetamine. ephedrine. acetylcholine.

homatropine bromide. Homatropine bromide is a short-acting chemical drug utilized to cause mydriasis and cycloplegia. It is very useful as a pupil dilator during retinoscopy and as a treatment for uveitis. Homatropine is also used to temporarily paralyze accommodation of the eyes, but it cannot achieve a complete cyclopegia of the eye.

Xerostomia is most likely to occur after taking chloral hydrate. fexofenadine hydroxyzine. phenobarbital.

hydroxyzine. Hydroxyzine is a drug that primarily acts as an anxiolytic and antihistamine. It is also used as a tranquilizer in dental procedures in which the patient manifests uncontrolled anxiety and tension. Hydroxyzine use can cause xerostomia or dry mouth due to its antimuscarinic effect.

In the normal patient, an increase in stress will lead to a/an _____ in ACTH production and large doses of circulating systemic steroids will _____this production. decrease, repress increase, stimulate increase, repress decrease, stimulate

increase, repress The ACTH stimulation test is performed to examine the response of the adrenal gland to an exogenously administered dose of ACTH. Normal patients have a doubling of the serum cortisol level after a dose of ACTH. The serum cortisol level should rise to >20 mg/dL if there is adequate adrenal function. An inadequate response suggests adrenal gland hypofunction. Note: Cosyntropin (Cortrosyn®) is an ACTH analogue that stimulates the adrenal gland and its ACTH receptors. About 20 mg of hydrocortisone is secreted by the adrenal cortex daily. During stress, the cortex can increase the output to 200 mg daily. Remember: Patients taking steroids or people with disease of the adrenals will have decreased ability to produce more glucocorticoids (hydrocortisone) in times of stress (extractions). The reason for this is as follows: Secretion of glucocorticoids is stimulated by ACTH, a hormone produced in the anterior pituitary. The pituitary responds to stress by increasing ACTH output and, therefore, glucocorticoid production increases. A relative lack of glucocorticoids will also increase output of ACTH. An overabundance of circulating systemic steroids will inhibit production of ACTH. Large doses of steroids repress ACTH production, which leads to atrophy of adrenal cortex. Corticotropin-releasing hormone (CRH) is released from the hypothalamus. CRH stimulates the release of ACTH from the anterior pituitary. ACTH stimulates cortisol release from the zona fasciculata of the adrenal glands, causing negative feedback to CRH and ACTH.

Administering a selective β1 adrenegic agonist to a patient will elicit

increased cardiac output. The heart has both β1 and β2 adrenergic receptors, but β1 adrenergic receptors are the predominant receptor present. β2 receptors are predominantly present in vascular smooth muscle. β1 adrenergic agonists cause an increase in heart rate and contractile force.

The substance ethanol

is partially absorbed in the stomach and partly in the intestine. Roughly 20% of ethanol absorption occurs in the stomach, and 80% takes place in the jejunum and ileum of the small intestine. Less than 1% of ethanol is absorbed in the mouth. Alcohol absorption occurs more quickly with an empty stomach, because the pyloric sphincter has no food to hold inside the stomach and allows the alcohol to pass through, absorbed in the small intestines.

Uncomplicated respiratory tuberculosis is best treated with a combination of isoniazid and rifampin, because they function by different mechanisms. streptomycin and chloramphenicol, because both are hydrophobic. griseofulvin and streptomycin, because both prevent protein production. ethambutol and isoniazid, because both inhibit bacterial folic acid synthesis.

isoniazid and rifampin, because they function by different mechanisms. Isoniazid inhibits the synthesis of mycolic acid, a component of the mycobacterial cell wall. Rifampicin inhibits bacterial DNA-dependent RNA polymerase, stopping bacterial RNA production. Tuberculosis and inactive meningitis are treated with a cocktail of pyrazinamide, isoniazid, ethambutol, rifampicin , and streptomycin ("PIERS").

The primary reason to use an antibiotic disk sensitivity assay is that the results can be determined within minutes. it can distinguish between bacteriostatic and bacteriocidal effects on the bacteria in question. it allows the prescriber to know which meds will cause more gastrointestinal (GI) upset by destroying normal gut flora. it allows the prescriber to test common antibiotics to see which are effective against the infection in question.

it allows the prescriber to test common antibiotics to see which are effective against the infection in question. An antibiotic disk sensitivity assay helps guide the physician in choosing the appropriate antibiotic for a certain bacterial infection. It is a type of test conducted in a laboratory that examines the effect of several types of antibiotics against bacterial strains. The effects are evaluated until the most effective antibiotic for a specific bacterium is determined. Having an antibiotic disk sensitivity assay reduces the risk of prescribing inappropriate antibiotics and of prolonged untreated bacterial disease.

When a maxillary third molar is displaced into the infratemporal fossa, it is usually displaced through the periosteum and located _________ to the lateral pterygoid plate and __________ to the lateral pterygoid muscle with displacement.

lateral, inferior The infratemporal fossa is an irregular space behind the maxilla. Its roof is formed by the greater wing of the sphenoid. The lateral pterygoid plate of the sphenoid is medial. Laterally, it is limited by the coronoid process and ramus of the mandible. The infratemporal fossa communicates with the pterygopalatine fossa through the pterygomaxillary fissure, which is a cleft between the lateral pterygoid plate and the maxilla. It communicates with the orbit through the inferior orbital fissure, which is found between the maxilla and the greater wing of the sphenoid. The pterygopalatine fossa is a small space behind and below the orbital cavity. It lies between the pterygoid plates of the sphenoid and palatine bone below the apex of the orbit. Clinical: If there is good access and adequate light, a single cautious effort to retrieve the tooth with a hemostat can be made. If the effort is unsuccessful, or if the tooth is not visualized, the incision should be closed, the patient should be informed, and prophylactic antibiotics should be prescribed. A secondary surgical procedure is performed 4-6 weeks later after lateral and posteroanterior radiographs are taken to locate the tooth in all three planes. After adequate anesthesia, a long needle is used to locate the tooth. Careful dissection is performed along the needle until the tooth is visualized and subsequently removed. Note: If no functional problems exist after displacement, the patient may elect not to have the tooth removed. Proper documentation of this is critical.

A 32-year-old patient presents with a tender swelling in the submandibular triangle. The most likely etiology is sialolith obstruction of salivary flow. lymphadenopathy. an infected thryroglossal duct cyst. a mucocele.

lymphadenopathy. Tender swelling localized within the submandibular triangle is indicative of lymphadenopathy. Lymphadenopathy is a condition in which the lymph nodes become swollen, tender, and enlarged. It could be an indication of infection, malignancy, or an autoimmune disease. Enlarged lymph nodes within the submandibular triangle are palpable during bidigital jaw palpation.

Referred preauricular pain is most common with maxillary molars. maxillary premolars. mandibular molars. mandibular premolars.

mandibular molars. The pain that originates from mandibular molars is sometimes referred to the ear, because both are innervated from one source, the mandibular nerve (V3). The nerves from the mandibular molars transmit pain stimuli to the mandibular nerve, which also provides sensory innervation to some parts of the external ear. Because of the same origin of innervation and the proximity of the external ear to the mandibular molars, the pain perceived within the molar area easily travels and radiates into the external ear.

Occult lesions are pathologies that manifest no signs or symptoms. present with confusing signs. cause the mandible to swell. cause pain.

manifest no signs or symptoms. Occult lesions are lesions that are not easily detectable by clinical methods alone.Occult lesions are problematic to distinguish, because even with a direct observation, the lesion would be difficult to visualize because it appears normal.Most of the time, occult lesions are asymptomatic and do not show signs of disease progression.Occult lesions are also difficult to find, even with the use of conventional radiographs. It is best to evaluate such lesions histologically and through magnetic resonance imaging (MRI).

After drawing a line to connect the three main canal orifices, which of the following molars appear to have formed a molar triangle?

maxillary first molar• maxillary second molar Both maxillary first and second molars form a molar triangle when a line is drawn to connect the three major canal orifices, i.e., distobuccal, mesiobuccal, and palatal. Following are the morphological features that need to be considered during preparation of an access cavity for the maxillary molar canals: Maxillary first molar: It is the largest tooth in volume. The pulp chamber is widest in the buccolingual dimension and has four pulp horns, namely mesiobuccal, mesiopalatal, distobuccal, and distopalatal. The cervical outline of the pulp chamber is rhomboid, with rounded corners in few cases. The palatal root offers the easiest access as it is the longest and has the largest diameter of the three roots; the other two being mesiobuccal and distobuccal roots. A line drawn to connect the three main canal orifices forms a triangle. The mesiobuccal and distobuccal roots often possess one or two canals; a third canal may be seen in the mesiobuccal root. Maxillary second molar: A line drawn to connect the three major canal orifices generally forms a flat triangle or an almost straight line. The three roots of the maxillary second molar are shorter in length, less curved, and either fused or placed closed to each other as compared to the roots of the maxillary first molar. A single canal is usually present in each root; however, multiple canals can also be observed. The access preparation is:rhomboid when four canals are presentrounded triangle with the base to the buccal when three canals are presentoval outline that is widest buccolingually when two canals are present

All of the following muscle/nerve pairings are correct EXCEPT one. Which one is the EXCEPTION? lateral rectus muscle / abducens nerve superior oblique muscle / trochlear nerve medial rectus muscle / abducens nerve inferior rectus muscle / oculomotor nerve

medial rectus muscle/abducens nerve The medial rectus is innervated by the lower division of cranial nerve III (oculomotor). ... The lateral rectus is innervated by cranial nerve VI (abducens nerve), and the superior oblique is innervated by cranial nerve IV (trochlear).

Administration of amyl nitrite for a prolonged period of time may result in

methemoglobinemia. Methemoglobinemia is a blood disorder characterized by levels of methemoglobin exceeding 1%. Ferrous, the reduced form of iron found in the heme group, combines with oxygen to produce oxyhemoglobin. Oxyhemoglobin distributes oxygen to the different tissues in the body and is changed back to hemoglobin after delivering the oxygen. Hemoglobin with the presence of a ferric heme group is called methemoglobin, which cannot transport oxygen. The blood normally has 1% methemoglobin. Causes of methemoglobinemia include exposure to environmental factors such as benzocaine, benzene, and nitrites. Methemoglobinemia may be acquired.[6] Classical drug causes of methemoglobinaemia include various antibiotics (trimethoprim, sulfonamides, and dapsone[7]), local anesthetics (especially articaine, benzocaine, prilocaine,[8] and lidocaine[9]), and aniline dyes, metoclopramide, rasburicase, umbellulone, chlorates, bromates, and nitrites.[10] Nitrates are suspected to cause methemoglobinemia.[11] In otherwise healthy individuals, the protective enzyme systems normally present in red blood cells rapidly reduce the methemoglobin back to hemoglobin and hence maintain methemoglobin levels at less than one percent of the total hemoglobin concentration. Exposure to exogenous oxidizing drugs and their metabolites (such as benzocaine, dapsone, and nitrates) may lead to an increase of up to a thousandfold of the methemoglobin formation rate, overwhelming the protective enzyme systems and acutely increasing methemoglobin levels.[citation needed] Infants under 6 months of age have lower levels of a key methemoglobin reduction enzyme (NADH-cytochrome b5 reductase) in their red blood cells. This results in a major risk of methemoglobinemia caused by nitrates ingested in drinking water,[12] dehydration (usually caused by gastroenteritis with diarrhea), sepsis, or topical anesthetics containing benzocaine or prilocaine. Nitrates used in agricultural fertilizers may leak into the ground and may contaminate well water. The current EPA standard of 10 ppm nitrate-nitrogen for drinking water is specifically set to protect infants.[12] Benzocaine applied to the gums or throat (as commonly used in baby teething gels, or sore throat lozenges) can cause methemoglobinemia.[

The maxillary first molar is innervated by which of the following nerves? Select all that apply. anterior superior alveolar middle superior alveolar posterior superior alveolar greater palatine ascending pharyngeal

middle superior alveolar• posterior superior alveolar The posterior superior alveolar (PSA) nerve block, otherwise known as the tuberosity block or the zygomatic block, is used to achieve anesthesia for the pulps of the maxillary third, second, and first molars (entire tooth = 72%; mesiobuccal root of the maxillary first molar not anesthetized = 28%). Target area: PSA nerve — posterior, superior, and medial to the posterior border of the maxilla. Note: Potential for hematoma formation. The middle superior alveolar (MSA) nerve block is useful for procedures where the maxillary premolar teeth or the mesiobuccal root of the first molar require anesthesia. Target area: maxillary bone above the apex of the maxillary second premolar. Note: The MSA nerve is present in only about 28% of the population. The anterior superior alveolar (ASA) nerve block or infraorbital nerve block provides profound pulpal and buccal soft-tissue anesthesia from the maxillary central incisor through the premolars in about 72 % of patients. Target area: infraorbital foramen (below the infraorbital notch). Remember: In order to extract the maxillary first molar, you must numb both the PSA and MSA nerves as well as the greater (anterior) palatine nerve for palatal anesthesia (soft tissue).

The most common type of ameloblastoma is ___________ multi cystic, and the most common location for an ameloblastoma to be found is the _____________ Ameloblastomas are most commonly found in the posterior mandible, including the posterior body, ramps, and angle.

multi cystic, posterior mandible

Which nerve may, in some cases, also serve as an afferent nerve for the mandibular first molar, which needs to be considered when there is failure of the inferior alveolar local anesthetic block? posterior superior alveolar nerve glossopharyngeal nerve facial nerve mylohyoid nerve

mylohyoid nerve Just before entering the mandibular canal, the inferior alveolar nerve gives off a motor branch known as the mylohyoid nerve. The inferior alveolar nerve travels along with the inferior alveolar artery and vein within the mandibular canal and divides into the mental and incisive nerve branches at the mental foramen. The inferior alveolar nerve provides sensation to the mandibular posterior teeth. The mylohyoid nerve pierces the sphenomandibular ligament and runs inferiorly and anteriorly in the mylohyoid groove and then onto the inferior surface of the mylohyoid muscle. The mylohyoid nerve serves as an efferent nerve to the mylohyoid muscle and the anterior belly of the digastric muscle. This nerve may, in some cases, also serve as an afferent nerve for the mandibular first molar. The mylohyoid muscle is an anterior suprahyoid muscle that is deep to the digastric muscle. In addition to either elevating the hyoid bone or depressing the mandible, the muscle also forms the floor of the mouth and helps elevate the tongue.Note: The sublingual gland is located superior to the mylohyoid muscle. 1. When placing the film for a periapical view of the mandibular molars, it is the mylohyoid muscle that gets in the way if it is not relaxed.2. When the floor of the mouth is lowered surgically, the mylohyoid and genioglossus muscles are detached.3. An injection into the parotid gland (capsule) when attempting to administer an inferior nerve block may cause a facial expression—paralysis of the forehead muscles, the eyelid, and the upper and lower lips on the same side of the face that the injection was given. Important: If the parotid capsule injection happens, care must be taken to protect the eye from injury and drying using lubrication and an eye patch.4. Remember: The bone of the maxilla is more porous than that of the mandible. Therefore, it can be infiltrated anywhere.

The impurity that is theoretically found in nitrous oxide that is considered slightly toxic is

nitric oxide Nitrous oxide is a gas that creates analgesia and dissociated anesthesia and is commonly used in dentistry. Nitrous oxide was first used by Horace Wells for his own tooth extraction by his assistant. Nitrous oxide is used in general anesthesia to enhance the effect of other drugs being administered by allowing the concentration of other drugs to be decreased without decreasing their effect. When nitrous oxide reacts with oxygen and is converted into nitric oxide, it causes vasodilation. Nitric oxide (nitrogen oxide[6] or nitrogen monoxide) is a colorless gas with the formula NO. It is one of the principal oxides of nitrogen. Nitric oxide is a free radical:

Diisopropyl fluorophosphate (DFP) and organophosphate pesticides function through

noncompetitively inhibiting cholinesterases. Cholinesterases are substances that allow for the breakdown of acetylcholine when in the body, stopping the acetylcholine from sending electrical impulses and therefore regulating movement of the body. With the presence of a cholinesterase inhibitor such as DFP and organophosphate, acetylcholine is not broken down, resulting in acetylcholine buildup. This causes an uncontrolled and continuous firing of electrical impulses that results in uncontrollable muscle twitching. Severe cases could lead to both the inability to breathe due to muscle paralysis and even death.

When ethanol is consumed while a patient is taking barbiturates, the resulting reaction is considered? competitive inhibition. uncompetitive inhibition. non-competitive inhibition. potentiation. negative synergism.

potentiation. Potentiation occurs when one drug administered simultaneously with another drug interacts to produce a synergistic effect. Barbiturates are drugs known to cause depression of the central nervous system (CNS). Due to their ability to make people calm, relaxed, and sleepy, they are also called sedative-hypnotics. Alcohol, like barbiturates, also depresses the CNS. When alcohol is given in combination with barbiturates, further depression of CNS can occur, which may lead to unconsciousness or death.

Which of the following are the morphological characteristics of maxillary first premolar?

presence of pulp horn under each cusp larger palatal orifice compared to the buccal orifice pulp chamber wider buccalingually than mesiodistally The maxillary first premolar exhibits the following morphological features: Average length of the maxillary first premolar is 21.5 mm. Usually presents two roots but variations are possible. Irrespective of the number of roots, only two root canals (buccal and palatal) are present in more than two-thirds of the cases. A developmental depression or furcation groove is an important anatomic feature observed on the palatal aspect of the buccal root. Pulp horns are present under each cusp (buccal and palatal pulp horns). The buccal pulp horn is larger compared to the palatal pulp horn. In younger teeth, buccal pulp horn is more prominent compared to the palatal pulp horn. In cross-section, the pulp chamber is narrower mesiodistally than buccopalatally and is ovoid in shape. The buccal orifice is slightly smaller than the palatal orifice. Points to be noted while preparing an access cavity for a maxillary first premolar: The access preparation is oval or slot-shaped; however, it is triangular with the base on the buccal aspect in cases where three canals are present. The shape of the access cavity preparation for endodontic treatment runs ovoid buccopalatally, which is different from GV Black's cavity preparation for a class I occlusal restoration that runs ovoid mesiodistally and encompasses all pits and fissures. The buccal extension of the boundary of the access cavity is two-thirds to three-fourths up the incline of the buccal cusp, whereas the palatal extension is almost halfway up the incline of the palatal cusp. The buccal canal orifice is located just below the buccal cusp, and the palatal canal orifice is located below the palatal cusp. Due to the mesial concavity of the root, overextension on the mesial side is avoided to prevent perforation.

The American Heart Association's 1997 recommendations for the prevention of bacterial endocarditis suggests antibiotic premedication should be used for

restorative dentistry with or without retraction cord. The American Heart Association's recommendations suggest antibiotic prophylaxis for patients with high- and moderate-risk heart conditions prior to having dental treatments with significant bleeding in the mouth. Restorative dentistry often causes gingival bleeding in cases of Class II, III, IV, or V restorations, especially when retraction cord is necessary. The exception may be when only Class I restorations are planned. Subgingival placement of the antibiotic strips may render bleeding within the gingival sulcus, depending on the condition of the gums and the depth of insertion, but not always. The orthodontic adjustments and placement of rubber dams do not usually cause any significant amount of bleeding in the process. Local anesthetic injections that do not penetrate too deeply into the mucosa would also not cause a significant amount of bleeding.

The oral antibiotic most likely to cause the failure of oral contraceptives is tetracyclines. macrolides. cephalosporins. penicillins. rifampin.

rifampin. Rifampin is an antibiotic that affects the metabolism of oral contraceptive drugs within the body. Rifampin induces the cytochrome P-450 enzyme, so drugs that use this metabolic pathway undergo accelerated elimination. Oral contraceptive failure during rifampin treatment can be prevented by adjusting the dosage of oral contraceptive drugs. Rifampin problems can be prevented by using drugs that are not metabolized by P-450 or by choosing non-hormonal birth control methods.

The ratio of the median lethal dose (LD50) to the median effective dose (ED50) indicates a drug's

safety. The therapeutic index is a ratio of the dose of a drug that causes lethal effect to the minimum dose of the drug that can elicit a therapeutic effect, or LD50 ÷ ED50. The therapeutic index is important to check the margin of safety of using a certain drug. A high therapeutic index value states that the drug is effective at a minimum dose, and its lethal effect occurs at higher doses. Having a low therapeutic index suggests that the drug has a narrow margin of safety.

A physician may enhance renal excretion of an acidic drug by administering

sodium bicarbonate. Alkalinization of urine through the use of sodium bicarbonate decreases acidic drug reabsorption and increases renal excretion of the drug. Acidic urine allows weak acid drugs to be easily reabsorbed. If the drug is a weak base, making the urine more alkaline helps the drug be reabsorbed. Excretion of drugs can be greatly affected by urinary pH.

Neuromuscular blockade of the phrenic nerve can be caused by extremely high doses of

streptomycin. Streptomycin is a bacteriocidal aminoglycoside intravenous (IV)/intramuscular (IM) antibiotic. Streptomycin has been reported to possess neuromuscular-blocking properties. Extremely high doses of streptomycin have allegedly resulted in lethal neuromuscular blockades of the phrenic nerve. Streptomycin is typically prescribed for tuberculosis, infective endocarditis, and plague. Streptomycin irreversibly binds to the 16S rRNA and S12 protein within the bacterial 30S ribosomal subunit. As a result, this agent interferes with the assembly of initiation complex between mRNA and the bacterial ribosome, thereby inhibiting the initiation of protein synthesis. Certain antibiotics can induce neuromuscular paralysis, but the mechanism of this action is largely unknown.

Which lymph nodes directly receive lymph from the tip of the tongue? submental lymph nodes submandibular lymph nodes parotid lymph nodes none of the above

submental lymph nodes The deep cervical lymph nodes are located along the length of the internal jugular vein on each side of the neck, deep to the sternocleidomastoid muscle. The deep cervical nodes extend from the base of the skull to the root of the neck, adjacent to the pharynx, esophagus, and trachea. The deep cervical nodes are further classified as to their relationship to the sternocleidomastoid muscle as being superior or inferior. The deep cervical lymph nodes are responsible for the drainage of most of the circular chain of nodes, and they receive direct efferents from the salivary and thyroid glands, the tongue, the tonsil, the nose, the pharynx, and the larynx. All these vessels join together to form the jugular lymph trunk. This vessel drains into either the thoracic duct on the left, the right lymphatic duct on the right, or it independently drains into either the internal jugular, subclavian, or brachiocephalic veins. Some regional groups of lymph nodes: Parotid lymph nodes - receive lymph from a strip of scalp above the parotid salivary gland, from the anterior wall of the external auditory meatus, and from the lateral parts of the eyelids and middle ear. The efferent lymph vessels drain into the deep cervical nodes. Submandibular lymph nodes - located between the submandibular gland and the mandible; receive lymph from the front of the scalp, the nose, and adjacent cheek; the upper lip and lower lip (except the center part); the paranasal sinuses; the maxillary and mandibular teeth (except the mandibular incisors); the anterior two-thirds of the tongue (except the tip); the floor of the mouth and vestibule; and the gingiva. The efferent lymph vessels drain into the deep cervical nodes. Submental lymph nodes - located behind the chin and on the mylohyoid muscle; receive lymph from the tip of the tongue, the floor of the mouth beneath the tip of the tongue, the mandibular incisor teeth and associated gingiva, the center part of the lower lip, and the skin over the chin. The efferent lymph vessels drain into the submandibular and deep cervical nodes.

Crystalluria is less likely to occur with sulfonamide treatment if accompanied by coadministered corticosteroids. coadministered probenecid. sulfonamide cocktail method. concurrent ammonium chloride.

sulfonamide cocktail method. Adequate fluid intake may lessen the chances of urolithiasis while taking sulfonamides. The sulfonamide cocktail method is the use of sulfonamide antibiotics in conjunction with other agents to decrease the amount of sulfonamide needed and thereby prevent crystallurea and stone formation. Sulfonamides should be taken with approximately 2,400 mL of water to facilitate better filtration and excretion in the kidneys. Sulfonamides are contraindicated for patients with impaired kidney function. Occasionally sulfonamide crystals are found in the urine of patients being treated with sulphonamides. When deposited in the urinary tract they can cause haematuria and other complications.

Antihypertensive drugs often cause postural hypotension because they interfere with

sympathetic control of vascular reflexes. Orthostatic hypotension, or postural hypotension, is a medical condition consisting of a sudden decrease in blood pressure when a person stands up. Baroreflex and autonomic pathways normally ensure that blood pressure is maintained despite various stimuli, including postural change. α-adrenergic (sympathetic) activity is the predominant pathophysiologic mechanism of orthostatic hypotension in hypertensive patients, especially with patients taking receptor-blocking medications.

Eccrine sweat glands are primarily innervated by

sympathetic-cholinergic. Sweat glands are classified as either eccrine or apocrine glands. Eccrine sweat glands are coiled, unbranched glands that serve in thermoregulation and function to cool down the body through the evaporation of sweat. Eccrine glands are innervated by the sympathetic nervous system, primarily by cholinergic fibers. Apocrine glands secrete sweat and oily compounds that act as pheromones when they are stimulated by adrenaline.

Thromboemboli formation is prevented by aspirin inhibiting

synthesis of thromboxane by COX. Thromboxane is a vasoconstrictor responsible for platelet aggregation and for thromboemboli. Aspirin causes the irreversible inactivation of the enzyme cyclooxygenase (COX), which is responsible for the production of prostaglandin and thromboxane. Thromboxane synthetase is the enzyme that mediates the reaction of thromboxane formation.

If a 500 mg dose of a drug demonstrates a therapeutic efficacy for four hours, and the half-life of the drug is six hours, a 1,000 mg dose would be effective for

ten hours. A drug with a therapeutic efficacy of four hours is above the minimal effective concentration for four hours. Doubling the dose from 500 mg to 1,000 mg keeps the blood level of the drug above the minimum effective concentration for a period of time longer than four hours. The elimination half-life (t½) is six hours, so 500 mg of the drug will be in the bloodstream for six hours after a dose of 1,000 mg. The effectiveness of the drug is 4 + 6 = 10 hours.

A split-thickness or mucosal skin graft used in a vestibuloplasty receives its nourishment and oxygenation primarily from the exposed periosteum that forms the graft bed. the vasculature in the subepithelial or submucosal layer moved to the site with the graft. the mucosa surrounding the graft. the exposed bone directly beneath the graft.

the exposed periosteum that forms the graft bed Mucosal skin grafts receive nourishment from the vascular supply coming from the exposed portion of the periosteum where the graft is adapted and placed. The graft is initially adapted and attached to the recipient site through fibrin layer formation. Nutrients for the graft are taken from the recipient bed through a process known as plasmatic imbibition. The end capillaries of the graft and the recipient bed then form a vascular network, which will provide nourishment to the whole graft later. Formation of new vessels by means of neovascularization will then enable better blood flow into the graft. 0 Comments

The root canal orifices in the mandibular first molar are smaller than those of the mandibular second molar. The access cavity in a two-canal mandibular second molar is rectangular and wider mesiodistally than buccolingually.

the first statement is false, second is true The salient features of the morphologic characteristics of mandibular first and second molars are as follows: Mandibular first molar The mandibular first molar is the first permanent posterior tooth to erupt and often requires endodontic procedures later in life. The mandibular first molar has an average length of about 21.9 mm and usually has two roots (mesial and distal). The mesial root is wider than the distal. The tooth consists of four pulp horns (mesiobuccal, mesiolingual, distobuccal, and distolingual) and three canal orifices (mesiolingual, mesiobuccal, and distal). The mesial root usually consists of mesiobuccal and mesiolingual canals. Occasionally a middle mesial canal is observed between the mesiobuccal and mesiolingual canals. The distal root can have one or more than one canals (distal canal if only one canal present and distolingual, distobuccal, and middle distal if more than one is present). The access preparation in mandibular first molar is rhomboid or trapezoid in shape irrespective of the number of canals. Mandibular second molar The mandibular second molar has an average length of about 21.4 mm and usually has two roots (mesial and distal) in close proximity to each other. The root apices are close together, and in some cases only one root is present. Compared to the mandibular first molar, the mandibular second molar is smaller and more symmetric. The pulp chamber and the root canal orifices of the mandibular second molar are also smaller compared to the mandibular first molar. Mandibular second molars usually have two canals, but may present only one or up to six canals. The mesial canal orifices of the mandibular second molar are in close proximity of each other and may sometimes be connected by a semicircular slit, a variation of the C-shaped canal. The shape of the access cavity for a two-canal mandibular second molar is rectangular, wide mesiodistally and narrow buccolingually. If only one canal is present, the access cavity is oval and lined up in the center of the occlusal surface. Note: Compared to other teeth, mandibular second molars show a higher incidence of having a C-shaped canal orifice.

In mandibular incisors, dimensions of the pulp canal remain similar at all levels. Mandibular central incisor is the smallest tooth in the arch.

the first statement is false, the second is true Mandibular incisors The root morphology and the access preparation techniques for the lateral and central mandibular incisors are very alike. Their small size (mandibular central incisor is the smallest tooth in the arch) and internal structure makes access cavity preparation very difficult. Contrary to the maxillary incisors, the mandibular incisors possess a wider pulp outline labiolingually.

percussion

the sharp striking of one thing against another

Prolonged use (more than 14 days) of clindamycin is discouraged because

there is a high incidence of pseudomembranous colitis. Clindamycin is a lincosamide antibiotic used to treat anaerobic bacterial infections, and its use is associated with pseudomembranous colitis. Clindamycin is prescribed to treat anaerobic bacterial infections and some protozoal diseases, such as malaria, and is also a common topical treatment for acne. The potency of oral clindamycin leads to widespread loss of normal gut flora, allowing Clostridium difficile to take over the gut, causing pseudomembranous colitis. Clindamycin may prolong the effects of neuromuscular-blocking drugs such as succinylcholine.

A laceration is a soft tissue injury defined by a break or tear in epithelial and sub epithelial tissues. It is considered to be one of the most frequent types of soft tissue injury and is often caused by sharp objects or torn by the force of a blow. Patients will require suturing and the evaluation of the vasculature and other adjacent anatomic structures Abrasion is a superficial wound and is caused by friction between an object and the soft tissue surface. The epithelium becomes denuded and may involve deeper layers. Commonly abrasions are known as scrapes.

true

Alveolar fractures are associated with: Pulp necrosis Splitting the segment and involved teeth to adjacent teeth and other facial injuries Alveolar fracture injuries are treated immediately by splinting the alveolar segment and involved teeth to the adjacent intact teeth with supporting bone

true

Oral health is a state of being free from chronic mouth and facial pain, oral and throat cancer, oral sores, birth defects such as cleft lip and palate, periodontal gum disease, tooth decay and tooth loss, and other diseases and disorders that affect the oral cavity.

true

Periapical scars are areas of sense fibrous connective tissue that form as a result of the termination of the healing process. They can appear as a radiolucent lesion around the apex of a tooth that has previously had a periodical lesion (and has subsequently undergone endodontic therapy). The remaining answer choices-focal sclerosis osteomyelitis, osteosclerosis, and cementoblastoma appear radiographically as opaque lesions

true

Which strain of bacteria is most commonly associated with dental-related subacute bacterial endocarditis?

α-hemolytic streptococci The bacteria most commonly associated with subacute bacterial endocarditis (SBE) linked to dental procedures is α-hemolytic streptococci. Species of α-hemolytic strep (viridans strep) include the following: S. mitis S. mutans S. salivarius S. sanguinus

Primary retention form: locks or retains the restorative material in the tooth Primary resistance form: helps the restoration and tooth resist fracturing as a result of occlusal forces Convenience form: features that are those that make the procedure easier and more accessible Outline form: the initial extension of the tooth preparation should be visualized preoperatively by estimating the extent of the defect, the preparation form requirements of the material, the need for adequate access

true

Scleroderma is an autoimmune, multi-organ disease of adults especially women. It may occur concomitantly with other autoimmune diseases such as lupus erythematous, rheumatoid arthritis, sjrogen's syndrome, dermatomyitis. For reasons not understood, dense collagen is deposited in the tissues of the body in extraordinary amounts. The skin develops a diffuse, hard texture, and its surface is usually smooth

true

The crowns of the primary molars are wider mesiodistally in comparison with crown length than are those of the permanent teeth The roots of primary anterior teeth are narrow and long compared with crown width and length The primary teeth are usually lighter in color than the permanent teeth The crowns and roots of primary molars are more slender mesiodistally at the cervical third than are those of the permanent molars

true

The thin periodontal biotype (also known as the scalloped biotype) consists of a collection of several characteristics. Including Delicate and friable soft tissue Adjacent teeth contact areas decidedly towards incised or occlusal thirds Teeth triangular in shape Steeper posterior cusps Underlying osseous form scalloped, often with dehiscences and fenestrations present The thick periodontal biotype (also known as the flat biotype) consists of characteristics that are distinctly different from that of the thin biotype. These include Reaction to insult by increasing pocket depth Denser, more fibrotic tissue Teeth square in shape Flatter posterior cusps Contact areas of adjacent teeth are more towards the apical

true

Penicillin derivatives can lead to hypersensitivity reactions of type I only. type IV only. types I, II, and IV. types I, II, III, and IV.

types I, II, III, and IV. The four hypersensitivity reaction types are as follows: Type I, allergy (immediate): e.g., anaphylaxis and asthma; mediated by IgE and IgG Type II, cytotoxic, autoimmune: e.g., hemolytic anemia and Graves disease; mediated by IgM, IgG, and complement Type III, immune complex: e.g., serum sickness, rheumatoid arthritis, and lupus; mediated by IgG and complement Type IV, delayed hypersensitivity: e.g., contact dermatitis, transplant rejection, and multiple sclerosis; mediated by T-cells

When chlorpromazine (CPZ) is administered to a patient who then stands up, the patient may experience dizziness due to

α-adrenergic inhibition. Chlorpromazine affects various types of receptors along the central nervous system, causing antidopaminergic, anticholinergic, antihistaminic, and some traces of antiadrenergic effect. The α-adrenergic inhibition of chlorpromazine may cause the lowering of blood pressure, which is later accompanied by dizziness. Chlorpromazine inhibits the α1- and α2-adrenergic receptors, which results in symptoms including reflex tachycardia, sedation, vertigo, hypersalivation, and a possibility of sexual dysfunction.

Which class of antihypertensive drugs inhibits the release of renin from the kidney? β-adrenergic receptor blockers Angiotensin-converting enzyme (ACE) inhibitors Loop diuretic Angiotensin II receptor blockers Calcium channel blockers

β-adrenergic receptor blockers Renin release from the kidney is stimulated by the β1-adrenergic receptors in the juxtaglomerular cells of the kidneys. β-adrenergic blockers (beta blockers) block the release of renin. Angiotensin-converting enzyme (ACE) inhibitors and angiotensin II receptor blockers do not inhibit renin release. ACE inhibitors increase the plasma renin levels.

The average size of the maxillary sinus is 14.75 mL, with a range of 9.5-20 mL. On average, the width is _____; the height is _____; and the depth is _____. 1.0 cm, 2.75 cm, 2 cm 2.5 cm, 3.75 cm, 3 cm 4.0 cm, 4.75 cm, 4 cm 5.0 cm, 5.75 cm, 3.5 cm

• 2.5 cm, 3.75 cm, 3 cm The maxillary sinus opens into the middle meatus of the nose through the hiatus semilunaris. Unfortunately, this opening lies high up on the medial wall of the sinus, so that the sinus readily accumulates fluid. Since the frontal and anterior ethmoidal sinuses drain into the infundibulum, which in turn drains into the hiatus semilunaris, the chance that infection may spread from these sinuses into the maxillary sinus is great. 2 types of sinusitis: acute and chronic: common clinical manifestations include sinus congestion, discharge, pressure, face pain, and headaches. Acute Sinusitis: the most common form of sinusitis, typically caused by a cold that results in inflammation of the sinus membranes, normally resolves in 1 to 2 weeks. Sometimes a secondary bacterial infection may settle in the passageways after a cold; bacterial populations normally located in the area (Streptococcus pneumoniae and Haemophilus influenzae) may begin to increase, producing an acute bacterial sinusitis. Clinical signs of acute sinusitis include: Severe pain, constant and localized Tenderness to percussion of the maxillary posterior teeth A mucopurulent exudate Any unusual motion or jarring accentuates the pain Tenderness over the anterior sinus wall Chronic sinusitis: an infection of the sinuses that is present for longer than 1 month and requires longer duration medical therapy. Typically either chronic bacterial sinusitis or chronic noninfectious sinusitis. Chronic bacterial sinusitis is treated with antibiotics (ampicillin or augmentin). Chronic noninfectious sinusitis often is treated with steroids (topical or oral) and nasal washes. Locations of sinusitis: Maxillary: the most common location for sinusitis; associated with all of the common signs and symptoms but also results in tooth pain, usually in the molar region Sphenoid: rare, but in this location can result in problems with the pituitary gland, cavernous sinus syndrome, and meningitis Frontal: usually associated with pain over the forehead and possibly fever Ethmoid: potential complications include meningitis and orbital cellulitis. Note: The maxillary sinus is innervated by the maxillary division of the trigeminal nerve (CN V-2), specifically, the ASA, PSA, and MSA nerves as well as the infraorbital nerve.

A full E cylinder of oxygen contains approximately: 150 L at a pressure of 2000 psi 300 L at a pressure of 2000 psi 600 L at a pressure of 2000 psi 750 L at a pressure of 2000 psi

• 600 L at a pressure of 2000 psi Nitrous oxide: Is a colorless, nonirritating gas with a pleasant, mild odor and taste Has a blood/gas partition coefficient of 0.47 and is thus poorly soluble in blood Is excreted unchanged by the lungs Is the oldest gaseous anesthetic in use today Is the only inorganic substance used as an anesthetic As a general anesthetic, the only disadvantage is its lack of potency Nitrous oxide should be stored under pressure in steel cylinders painted blue. Oxygen is stored in green tanks. A full E cylinder of oxygen contains approximately 600 L at a pressure of 2000 psi. At 2 L/min, a full E cylinder will deliver oxygen for approximately 300 min, or 5 hrs.

The trigeminal ganglion is located: superior to the deep lobe of the submandibular salivary gland posterior surface of the maxillary tuberosity of the maxilla anterior to the infraorbital foramen of the maxilla at the apex of the petrous part of the temporal bone in the middle cranial fossa

• at the apex of the petrous part of the temporal bone in the middle cranial fossa The trigeminal nerve emerges from the anterior surface of the pons by a large sensory and a small motor root, the motor root lying medial to the sensory root. The nerve passes forward out of the posterior cranial fossa, below the superior petrosal sinus, and carries with it a pouch derived from the meningeal layer of dura mater. On reaching the depression on the apex of the petrous part of the temporal bone in the middle cranial fossa, the large sensory root expands to form the trigeminal ganglion. The motor root of the trigeminal nerve is situated below the sensory ganglion and is completely separate from it. The ophthalmic, maxillary, and mandibular nerves arise from the anterior border of the ganglion. Somatic sensory cell bodies of the ganglion's sensory fibers enter the: Ophthalmic division (CN V1) to supply general sensation to the orbit and skin of face above eyes Maxillary division (CN V2) to supply general sensation to the nasal cavity, maxillary teeth, palate, and skin over maxilla Mandibular division (CN V3) to supply general sensation to the mandible, TMJ, mandibular teeth, floor of mouth, tongue, and skin of mandible The axons of the neurons enter the pons through the sensory root and terminate in one of the three nuclei of the trigeminal sensory nuclear complex:

The retrodiscal tissue of the TMJ is highly vascularized and innervated. Only the extreme periphery of the articular disc is slightly innervated. both statements are true both statements are false the first statement is true, the second is false the first statement is false, the second is true

• both statements are true The articular disc (meniscus) is composed of dense fibrous connective tissue, and it is positioned in between the condyle and the fossa, thereby dividing the joint into superior and inferior joint spaces. The articular disc (meniscus) varies in thickness; the thinner central intermediate zone separates the thicker portions, which are the anterior and posterior bands. The posterior band of the articular disc is the thickest of the two bands, and it is attached with posterior loose connective tissues called retrodiscal tissues (bilaminar zone; posterior attachment). The less thick anterior band of the articular disc is contiguous with the capsular ligament, the condyle, and the superior belly of the lateral pterygoid muscle. Note: The retrodiscal tissue is highly vascularized and innervated, whereas the articular disc for the most part is not. Only the extreme periphery of the articular disc is slightly innervated.

Which of the following are involved in the path for parasympathetic innervation of the parotid gland? Select all that apply. trigeminal nerve glossopharyneal nerve vagus nerve otic ganglion pterygopalatine ganglion

• glossopharyneal nerve• otic ganglion The pterygopalatine ganglion is responsible for innervation of the lacrimal gland and other glands of the nasal cavity. The other parasympathetic ganglia include the ciliary, submandibular, and otic. The nerve fibers pass to the otic ganglion via the tympanic branch of the glossopharyngeal nerve and the lesser petrosal nerve. Postganglionic parasympathetic fibers reach the parotid gland via the auriculotemporal nerve, which lies in contact with the deep surface of the gland. Note: Postganglionic sympathetic fibers reach the gland as a plexus of nerves around the external carotid artery. The parotid gland is the largest of the major salivary glands and is entirely serous in secretion. The parotids are located below and just anterior to the ear. The gland's capsule is from the deep cervical fascia. About 75% or more of the parotid gland overlies the masseter muscle, the rest is retromandibular. The parotid gland is drained by Stenson duct, which forms within the deep lobe and passes from the anterior border of the gland across the masseter muscle superficially, through the buccinator muscle into the oral cavity opposite the maxillary second molar. The external carotid artery and its terminal branches within the gland, namely, the superficial temporal and the maxillary arteries, supply the parotid gland. The lymph vessels drain into the parotid lymph nodes and deep cervical lymph nodes. 1. Mumps is a viral disease of the parotid gland. Parotitis is the inflammation of the parotid gland.2. Ebner glands are the only other adult salivary glands that are purely serous.3. Although it passes through the parotid gland, the facial nerve does not provide any innervation to it.

Shock is characterized by all of the following EXCEPT one. Which one is the EXCEPTION? increased vascular resistance bradycardia myocardial ischemia mental status changes adrenergic response anxiety, vomiting and diarrhea oliguria cool mottled skin

• bradycardia The term "shock" denotes a clinical syndrome in which there is inadequate cellular perfusion and inadequate oxygen delivery for the metabolic demands of the tissues.Important: Reduced cardiac output is the main factor in all types of shock. In general, shock is characterized by: Increased vascular resistance: cool mottled skin, oliguria Tachycardia Adrenergic response: diaphoresis, anxiety, vomiting, diarrhea Myocardial ischemia Mental status changes The stages of shock include: (1) Compensatory (early) stage: compensatory mechanisms (increased heart rate and peripheral resistance) maintain perfusion to vital organs, (2) Progressive stage: metabolic acidosis occurs (compensatory mechanisms are no longer adequate), (3) Irreversible (refractory stage): organ damage, survival is not possible. Major categories of shock: Hypovolemic shock is produced by a reduction in blood volume. Cardiac output will below due to inadequate left ventricular filling. Causes include severe hemorrhage, dehydration, vomiting, diarrhea, and fluid loss from burns. Cardiogenic shock is circulatory collapse resulting from pump failure of the left ventricle, most often caused by massive myocardial infarction. Septic shock is due to severe infection. Causes include the endotoxin from gram-negative bacteria. Neurogenic shock results from severe injury or trauma to the CNS. Anaphylactic shock occurs with severe allergic reaction.

All of the following drugs help to reduce salivary flow during dental treatment EXCEPT one. Which one is the EXCEPTION? scopolamine atropine local anesthesia cevimeline HCL benztropine

• cevimeline HCL Local anesthesia acts by reducing sensitivity which reduces anxiety and stress related to treatment; salivation is also decreased. Scopolamine, atropine, and benztropine are anticholinergic drugs. Not only do they decrease the flow of saliva, but they also decrease the secretion from respiratory glands during general anesthesia. Cevimeline HCL increases the flow of saliva. The duration of action of local anesthetics is directly proportional to protein binding and lipid solubility. Increased protein binding — increased lipid solubility = increased duration of action. The lower the pKa (dissociation constant) of the local anesthetic, the faster the onset of action. Important point: a local anesthetic with a low pKa has a very large number of lipophilic free base molecules that are able to diffuse through the nerve membrane. Increased blood flow — shorter duration of action. Metabisulfite is an antioxidant that protects the vasoconstrictor from oxidation. It has a low incidence of allergenicity. The local anesthetic prilocaine can produce methemoglobinemia when administered in larger doses in patients with subclinical methemoglobinemia. The topical anesthetic benzocaine also can induce methemoglobinemia, but only when administered in very large doses. The administration of levonordefrin should be avoided in patients receiving tricyclic antidepressants. There is an increased sensitivity to vasoconstrictors.*** Epinephrine should be used cautiously. The administration of vasoconstrictors in patients being treated with nonselective beta-blockers (i.e., propranolol) increases the likelihood of a serious elevation of the blood pressure accompanied by a reflex bradycardia. Use vasoconstrictors cautiously.

The TMJ is a _____ joint whose articular disc is composed of _____ tissue. The TMJ is bound anteriorly by the articular eminence anteriorly and posteriorly by the tympanic part of the temporal bone. ginglymoarthrodial joint, fibrous connective ginglymus joint, fibrocartilaginous ginglymus joint, fibrous connective ginglymoarthrodial joint, fibrocartilaginous connective

• ginglymoarthrodial joint, fibrocartilaginous connective Because the TMJ has characteristics of both a hinge joint and a gliding joint, it is classified as a ginglymoarthrodial joint. A unique feature of the TMJ is that it is rigidly connected to both the dentition and the contralateral TMJ. Components of the TMJ: Mandibular condyle (sometimes called the condyloid process of the mandible) - the articulating surface or functioning part of the condyle is located on the superior and anterior surfaces of the head of the condyle. This surface is covered with a dense layer of fibrous connective tissue. Articular fossa - this fossa is the anterior three-fourths of the larger mandibular fossa. It is considered to be a nonfunctioning portion of the joint. Remember: The mandibular fossa (glenoid fossa) is the temporal component of the TMJ; it is bounded in front by the articular eminence, and behind, by the tympanic part of the temporal bone, which separates it from the external auditory meatus. Articular eminence (also called the articular tubercle) - is a ridge that extends mediolaterally just in front of the mandibular fossa. It is considered to be the functional portion of the joint. It is lined with a thick dense layer of fibrous connective tissue. Articular disc (also called the meniscus) - is a biconcave fibrocartilaginous disc interposed between the condyle of the mandible and the mandibular (glenoid) fossa of the temporal bone which provides the gliding surface for the mandibular condyle, resulting in smooth joint movement. The central part is avascular and devoid of nerve tissue. Only the extreme periphery is slightly innervated.

A 52-year-old woman requests removal of a painful mandibular second molar. She tells you that she has not rested for 2 days and nights because of the pain. Her medical history is unremarkable, except that she takes 20 mg of prednisone daily for erythema multiforme. How do you treat this patient? instruct the patient to take 3 grams of amoxicillin 1 hour prior to extraction give steroid supplementation and remove the tooth with local anesthesia and sedation have patient discontinue the prednisone for 2 days prior to the extraction no special treatment is necessary prior to extraction

• give steroid supplementation and remove the tooth with local anesthesia and sedation Important: The fear here is that the patient may not have sufficient adrenal cortex secretion (adrenal insufficiency) to withstand the stress of an extraction without taking additional steroids. (This holds true for any patient who has been treated with steroid therapy.) Patients with adrenal insufficiency, patients on daily steroid therapy, and patients who have recently finished a course of steroids should receive steroid supplementation for dental procedures. The concerns about adrenal insufficiency should be raised on the basis of clinical history. In the majority of cases, the dentist should ask: Is it known that the patient's adrenal glands do not function adequately? Is the patient on chronic steroid therapy at doses of prednisone higher than 15 mg/day? Has the patient been on steroid therapy at doses of prednisone higher than 15 mg/day within the last 2 weeks?***If the answer to any of the above questions is yes, the dentist should assume that the patient will need stress-dose steroids. General guidelines for the management of patients on steroid therapy: Steroid supplementation in patients who can develop adrenal insufficiency Early morning appointments Shorter appointments Minimize stress Use sedation techniques when appropriate Modify dental treatment plan when appropriate The major goal in these patients is to avoid precipitating of adrenal insufficiency Remember: Erythema multiforme is a hypersensitivity syndrome characterized by polymorphous eruption of skin and mucous membranes. Macules, papules, nodules, vesicles, or bullae and target or ("bull's-eye-shaped") lesions are seen. A severe form of this condition is known as Stevens-Johnson syndrome. These patients may be receiving moderate doses of systemic corticosteroids and therefore may be unable to withstand the stress of an extraction. Consultation with their physician is absolutely necessary before treating these patients.

The mandible is formed by _____ ossification and has synovial TMJ with fibrocartilage articular surfaces. The _____ artery provides branches for the most direct blood supply of the TMJ. intramembranous, internal carotid intramembranous, external carotid endochondral, internal carotid endochondral, external carotid

• intramembranous, external carotid The major arterial blood supply to the TMJ is derived from the superficial temporal artery and from the maxillary artery posteriorly, and from smaller masseteric, posterior deep temporal, and lateral pterygoid arteries anteriorly. The venous drainage is through a diffuse plexus around the capsule and rich venous channels that drain the retrodiscal tissue. Note: The two terminal branches of the external carotid artery are the superficial temporal artery and the maxillary artery. The fibrous capsule of the TMJ is innervated from a large branch of the auriculotemporal nerve (branch of CN V3). The anterior region of the joint is innervated from the masseteric nerve (also a branch of CN V3) and from the posterior deep temporal nerve (also a branch of CN V3). The sensory innervation of the TMJ is via the trigeminal nerve as well. The nerve fibers primarily follow the vascular supply and terminate as free nerve endings. Thus, the capsule, synovial tissue, and extreme periphery of the disc are innervated. The articular cartilage and the central part of the disc contain no nerves. Both myelinated and nonmyelinated nerves are seen in the TMJ. The retrodiscal bilaminar zone has a rich neurovascular supply and is the source of proprioception. Remember: Most synovial joints have hyaline cartilage on their articular surface; however, several joints, such as the sternoclavicular, acromioclavicular, and TMJs, are associated with bones that develop from intramembranous ossification. These have fibrocartilage articular surfaces.

The _________ arises from the anterior surface of the external carotid artery and then passes near the greater cornu of the hyoid bone. submental artery inferior alveolar artery lingual artery ascending pharyngeal artery

• lingual artery It loops upward and then passes deep to the posterior border of the hyoglossus muscle to enter the submandibular region. The loop of the artery is crossed superficially by the hypoglossal nerve. The lingual artery supplies structures of the floor of the mouth and the posterior and inferior surface of the tongue. Major branches include the: Suprahyoid artery: supplies the suprahyoid muscles Dorsal lingual artery: supplies the tongue, tonsils, and soft palate Sublingual artery: supplies the floor of the mouth, mylohyoid muscle, and sublingual gland Deep lingual artery: supplies the tongue Important: The lingual artery does not accompany the corresponding nerve throughout its course. Remember: The inferior alveolar nerve, artery, and vein along with the lingual nerve are found in the pterygomandibular space between the medial pterygoid muscle and the ramus of the mandible. The inferior alveolar nerve passes lateral to the sphenomandibular ligament. The submandibular duct is crossed twice by the lingual nerve. If the lingual nerve is cut after the chorda tympani joins, there will be loss of both taste and tactile sensation. Note: The lateral pterygoid muscle forms the roof of the pterygomandibular space.

Which of the following nerves exits the skull through the foramen rotundum? ophthalmic nerve (CN V1) maxillary nerve (CN V2) facial nerve mandibular nerve (CN V3)

• maxillary nerve (CN V2) The ophthalmic nerve (CN V1) enters the middle cranial fossa through the superior orbital fissure and courses within the lateral wall of the cavernous sinus on its way to the trigeminal ganglion. The maxillary nerve (CN V2) enters the middle cranial fossa through foramen rotundum and may or may not pass through the cavernous sinus en route to the trigeminal ganglion. The mandibular nerve (CNV3) enters the middle cranial fossa through foramen ovale, coursing directly into the trigeminal ganglion. The trigeminal ganglion (aka semilunar ganglion) lies in a depression known as the trigeminal cave (or Meckel cave). The trigeminal nerve exits the trigeminal ganglion and courses "backward" to enter the mid-lateral aspect of the pons. The mandibular division is the largest of the 3 divisions of the trigeminal nerve. It has motor and sensory functions. It is created by a large sensory and a small motor root that unites just after passing through the foramen ovale to enter the infratemporal fossa. It immediately gives rise to a meningeal branch and then divides into anterior and posterior divisions. Anterior Division: Smaller, mainly motor, with 1 sensory branch (buccal): Masseteric: innervates the masseter muscle and provides a small branch to the TMJ Anterior and posterior deep temporal: innervates the temporalis muscle Medial pterygoid: innervates the medial pterygoid muscle Lateral pterygoid: innervates the lateral pterygoid muscle Buccal: supplies the skin over the buccinator muscle before passing through it to supply the mucous membrane lining its inner surface and the gingiva along the mandibular molars Posterior Division: Larger, mainly sensory, with 1 motor branch (nerve to mylohyoid): Auriculotemporal: supplies the TMJ, auricle, and external auditory meatus Lingual: supplies the mucous membrane of the anterior 2/3 of the tongue and gingiva on the lingual side of the mandibular teeth Inferior alveolar: largest branch of the mandibular division; innervates all mandibular teeth and the gingiva from the premolars anteriorly to the midline via the mental branch Mylohyoid: supplies the mylohyoid and the anterior belly of the digastric muscle Remember: The trigeminal nerve contains no parasympathetic component at its origin.

The submandibular gland is a _____ gland whose secretomotor innervationcomes from the _____ nerve. The sublingual caruncle marks the opening of its duct into the oral cavity. purely mucous, CN VII mixed, CN IX purely mucous, CN IX mixed, CN VII

• mixed, CN VII The submandibular glands (formerly called the submaxillary glands) are located in the submandibular triangle of the neck and the floor of the oral cavity. The submandibular duct (Wharton duct) is a long duct that travels along the anterior floor of the mouth. The duct opens into the oral cavity at the sublingual caruncle, a small papilla near the midline of the mouth floor on each side of the lingual frenum. Clinically, the gland is effectively palpated inferior and posterior to the body of the mandible, moving inward from the inferior border of the mandible near its angle as the patient lowers the head. Note: The submandibular gland is a mixed gland, secreting both serous and mucous saliva, but predominantly secreting serous mucous. The submandibular glands are innervated by efferent (parasympathetic) secretomotor fibers from the facial nerve, which run in the chorda tympani and in the lingual nerve (branch of CN V3) and synapse in the submandibular ganglion. Note: This is the same as the sublingual glands. The blood supply comes from branches of the facial and lingual arteries. The veins drain into the facial and lingual veins. The lymph vessels drain into the submandibular and deep cervical lymph nodes. Important: During its course, Wharton's duct is closely related to the large lingual nerve that eventually crosses over it. This is important because, if you incise the mucous membranes of the floor of the mouth, depending on where you cut, you may expose the lingual nerve, Wharton duct, and the sublingual gland. 1. To expose the duct intraorally, only mucous membrane needs to be cut through.2. Lymphadenopathy is the most common cause of swelling of the tissues in the submandibular triangle.

To extract a single mandibular central incisor, which of the following nerve block is considered the most appropriate? supraperiosteal nerve block none of the above lingual nerve block posterior superior alveolar nerve block

• supraperiosteal nerve block To achieve adequate local and regional anesthesia, a clinician should have sufficient knowledge regarding the different anesthetic agents, the methods of administration, and the proper procedures. Dental extractions are done under regional nerve block as they cannot be managed only with local infiltration. Supraperiosteal nerve block technique is ideal for anesthesia of a single tooth. In this, a needle with its bevel facing the bone is entered into the mucobuccal fold, and 1 to 2 mL of anesthetic solution is injected near the apex of the tooth. Supraperiosteal nerve block produces anesthesia in 5 to 10 minutes, longer than most blocks, as the anesthetic agent has to perforate the bone cortex in order to reach the nerve. Remember: Dental anesthesia is usually effective and well tolerated; however, it can lead to a number of mild-to-severe side effects. The side effects to local anesthetics can be categorized as follows: Technique: incorrect placement of the needle leading to nerve or vessel damage Anesthetic activity: allergic reaction or irritation after injecting the solution Systemic effects: if injected accidently into a blood vessel Effect of additives: vasoconstrictors, preservatives, etc. Nausea, dizziness, agitation, tremors, tachycardia, syncope, and seizures are potential systemic adverse events seen when the block injection containing the local anesthetic solution accidentally punctures the blood vessel. Pre-existing diseases or risk factors such as allergy, cardiovascular diseases, and pregnancy increase the rate of complications. It is important for the clinician to obtain relevant medical history and calculate the required dose of anesthesia prior to injecting the local anesthetic solution and carrying out a dental extraction.

The sublingual gland is located in the oral cavity between the mucosa of the oral cavity and the: masseter muscle mylohyoid muscle buccinator muscle temporalis muscle

• mylohyoid muscle The sublingual glands are located in the floor of the mouth beneath the tongue, close to the midline. It lies between the sublingual fossa of the mandible and the genioglossus muscle of the tongue. The mylohyoid muscle supports the individual sublingual glands inferiorly. Unlike the submandibular gland, which drains via one large duct, the sublingual gland drains via approximately 12-20 small secretory ducts (ducts of Rivinus). The majority open into the mouth on the summit of the sublingual fold, but a few open into the submandibular duct. The sublingual gland is innervated by parasympathetic secretomotor fibers from the facial nerve, which run in the chorda tympani and in the lingual nerve (branch of CNV3) and synapse in the submandibular ganglion. The blood supply comes from branches of the facial and lingual arteries. The veins drain into the facial and lingual veins. The lymph vessels drain into the submandibular and deep cervical lymph nodes. Important: The lymph vessels from both the sublingual and submandibular glands drain into the submandibular and the deep cervical lymph nodes Bartholin duct, a common duct that drains the anterior part of the sublingual glandin the region of the sublingual papilla, may be present The submandibular duct lies on the sublingual gland The sublingual gland is a mixed salivary gland, secreting both mucous and serous saliva, but it predominantly secretes mucous Note: Ebner glands are located around the circumvallate papilla of the tongue. Their main function is to rinse the food away from the papilla after it has been tasted by the taste buds. They are purely serous.

After a stroke on the left side of the brain that affects the left upper motor neurons, the tongue deviates to the: left on protrusion right on protrusion neither of the above, the tongue would not be affected

• right on protrusion — and the right half of the tongue will atrophy Lesions of the hypoglossal nerve: Hypoglossal nerve lesions paralyze the tongue on one side On protrusion, the tongue deviates to the ipsilateral (same) or contralateral side, depending on the lesion site Lower motor neuron lesion: Lesions to the hypoglossal nerve cause paralysis on the ipsilateral (same) side: Tongue deviates to the paralyzed side on protrusion (the paralyzed muscles will lag, causing the tip to deviate) Musculature atrophies on the paralyzed side Tongue fasciculations occur on the paralyzed sideExample: With a neck wound that cuts the right hypoglossal nerve, the tongue deviates to the right on protrusion, and the right half of the tongue will later demonstrate atrophy and fasciculations. Upper motor neuron lesion: Causes paralysis on the contralateral side: Tongue deviates to the side opposite the lesion Musculature atrophies on side opposite the lesionExample: After a stroke on the right side of the brain that affects the right upper motor neurons, the tongue deviates to the left on protrusion, and the left half of the tongue will atrophy. Important: If the genioglossus muscle is paralyzed, the tongue has a tendency to fall back and obstruct the oropharyngeal airway with risk of suffocation.

A dentist is performing a routine restoration on the left mandibular first molar. He is giving an inferior alveolar nerve block injection, where he deposits anesthetic solution right next to the lingula and mandibular foramen. Which ligament is most likely to get damaged? sphenomandibular ligament temporomandibular ligament stylomandibular ligament none of the above

• sphenomandibular ligament The sphenomandibular and stylomandibular ligaments are considered to be accessory ligaments. The former is attached to the lingula of the mandible and the latter at the angle of the mandible. These ligaments are responsible for limitation of mandibular movements (they limit excessive opening). Note: The sphenomandibular ligament is most often damaged in an inferioralveolar nerve block. The temporomandibular ligament (also called the lateral ligament) runs from the articular eminence to the mandibular condyle. It provides lateral reinforcement for the capsule. This ligament prevents posterior and inferior displacement of the condyle (it is the main stabilizing ligament of the TMJ). Note: This ligament keeps the head of the condyle in the mandibular fossa if the condyle is fractured. Collateral ligaments (medial and lateral) also referred to as "discal ligaments," are ligaments that arise from the periphery of the disc, are attached to the medial and lateral poles of the condyle, respectively, and stabilize the disc on the top of the condyle. These ligaments restrict movement of the disc away from the condyle during function. Note: They are composed of collagenous connective tissue; thus they do not stretch.

According to Guedel's stages of anesthesia, the proper use of nitrous oxide achieves which level of anesthesia? stage I stage II stage III stage IV

• stage I Guedel's Stages of Anesthesia: Stage I (amnesia and analgesia): begins with the administration of anesthesia and continues to the loss of consciousness. Respiration is quiet, though sometimes irregular, and reflexes are still present. Stage II (delirium and excitement): begins with the loss of consciousness and includes the onset of total anesthesia. During this stage, the patient may move his limbs, chatter incoherently, hold his breath, or become violent. Vomiting with the attendant danger of aspiration may occur. The patient is brought to Stage III as quickly and as smoothly as possible. Stage III (surgical anesthesia): begins with the establishment of a regular pattern of breathing, total loss of consciousness, and includes the period during which signs of respiratory or cardiovascular failure first appear. This stage has four planes. Stage IV (premortem): signals danger. This stage is characterized by pupils that are maximally dilated and skin that is cold and ashen. Blood pressure is extremely low, often unmeasurable. Cardiac arrest is imminent. Remember: The eyes appear greatly enlarged in size and nonreactive to bright light when functional circulation to the brain has stopped.

The pterygomandibular raphe, an important landmark for the administration of inferior alveolar block, provides attachment to which of the following muscles? Select all that apply. superior constrictor of the pharynx buccinator masseter levator labii superioris

• superior constrictor of the pharynx• buccinator The pterygomandibular raphe is a thick tendinous intersection attached superiorly to the pterygoid hamulus of the medial pterygoid plate and inferiorly to the posterior end of the mylohyoid line of the mandible. It is the point of attachment for the buccinators anteriorly and the superior constrictors of the pharynx posteriorly. It is palpated medially wherein it is covered by the mucous membrane of the mouth. Laterally, it is separated from the mandibular ramus by fat tissue. The pterygomandibular raphe also marks the anterior boundary of the masticator space and prestyloid compartment of the parapharyngeal space, through which the lingual and inferior alveolar branches of the trigeminal nerves traverse. Note: On opening the mouth, the pterygomandibular raphe raises a mucosal fold. This fold indicates the posterior boundary of the cheek, which is a pivotal landmark for an inferior alveolar nerve block.

The carotid sheath contains all of the following EXCEPT one. Which one is the EXCEPTION? carotid artery sympathetic trunk jugular vein vagus nerve

• sympathetic trunk *** The carotid sheath does not contain the sympathetic trunk, which lies posterior to the carotid sheath and anterior to the prevertebral fascia. The carotid sheath is located at the lateral boundary of the retropharyngeal space at the level of the oropharynx on each side of the neck deep to the sternocleidomastoid muscle. It extends from the base of the skull to the first rib and sternum. It contains the carotid arteries, the jugular vein, and the vagus nerve. Within the carotid sheath, the vagus nerve (CN X) lies posterior to the common carotid artery and internal jugular vein. The facial vein unites with the retromandibular vein below the border of the mandible and empties into the main venous structure of the neck - the internal jugular vein. The internal jugular vein descends through the neck within the carotid sheath and unites behind the sternoclavicular joint with the subclavian vein to form the brachiocephalic vein. The brachiocephalic veins (right and left) unite in the superior mediastinum to form the superior vena cava, which returns blood to the right atrium of the heart.

Which of the following is a clinical diagnosis based on subjective and objective findings indicating that the vital inflamed pulp is incapable of healing and has the following additional descriptors: lingering thermal pain, spontaneous pain, and referred pain?

• symptomatic irreversible pulpitis

After receiving an injection of a local anesthetic containing 2% lidocaine with 1:100,000 epinephrine, the patient loses consciousness. Which of the following is the most probable cause? acute toxicity allergic response syncope hyperventilation syndrome

• syncope *** Caused by transient cerebral hypoxia Anxiety-induced events are by far the most common adverse reaction associated with local anesthetics in dentistry. These may manifest in numerous ways, the most common of which is syncope. In addition, they may present with a wide variety of symptoms, including hyperventilation, nausea, vomiting, and alterations in heart rate or blood pressure. Psychogenic reactions are often misdiagnosed as allergic reactions and may also mimic them, with signs such as urticaria, edema, and bronchospasm. Proper management of syncope: Place patient in supine position with feet slightly elevated (Trendelenburg position) Establish airway (head tilt/chin lift)Administer 100% oxygen via face mask. O2 is indicated for the treatment of all types of syncope except for hyperventilation syndrome. Monitor vital signs and support patientPupils may dilate from brain not getting oxygen. Maintain your composure. Apply cool, wet towel to patient's forehead. Follow-up treatmentDetermine factors causing unconsciousness. Remember: Hyperventilation in an anxious dental patient leads to carpopedal spasm (a spasm of the hand, thumbs, foot, or toes).

A person who has been on suppressive doses of steroids will? Select all that apply. may show signs of hyperpigmentation take as long as a month to regain full adrenal cortical function take as long as a year to regain full adrenal cortical function does not require consultation with a physician prior to surgery

• take as long as a year to regain full adrenal cortical function• May show signs of hyperpigmentation The following guidelines may help determine if a patient's adrenal function is suppressed, however, if any doubt exists, consult the patient's physician before performing surgery. Some Guidelines: People on small doses (5 mg prednisone/day) will have suppression when they have been on the regimen for a month. People taking the equivalence of 100 mg cortisol/day (20-30 mg prednisone/day) will have abnormal cortical function in a week. Short-term therapy (1-3 days) of even high-dose steroids will not alter adrenal cortical function. A person who has been on suppressive doses of steroids will take as long as a year to regain full adrenal cortical function. Patients with adrenal insufficiency are hyperpigmented. This is most noticeable on the buccal and labial mucosa, although other areas such as the gingiva may be involved. The hyperpigmentation is a result of hypersecretion of ACTH, which can stimulate melanocytes to produce pigment. Patients with decreased adrenal gland hormone production experience weakness, weight loss, orthostatic hypotension, nausea, and vomiting. Patients with severe adrenal insufficiency cannot increase steroid production in response to stress and in extreme situations may have cardiovascular collapse. It is important that an adrenally insufficient patient have adequate steroid replacement, since the stress of oral surgery can precipitate adrenal crisis. Adrenal crisis is characterized by acute-onset fatigue, mental status changes, and hypotension. Electrolyte abnormalities are also present in the form of hyponatremia, hyperkalemia, and nonanion gap acidosis. In adrenal crisis, appropriate therapy includes 0.9% normal saline infusion and corticosteroid replacement with hydrocortisone, 100 mg IV every 6-8 hr. If an ACTH stimulation test is needed, dexamethasone (2-4 mg every 8 hr) should be used for replacement instead; this drug, unlike hydrocortisone, is not measured as serum cortisol in the blood. Note: Mineralocorticoid replacement with Florinef is not indicated in the acute management of adrenal crisis.

Epinephrine and levonordefrin are added to local anesthetics because of their: ability to increase the potency of the local anesthetic ability to decrease the pain (burning) caused by the injection of the local anesthetic vasoconstrictive properties ability to decrease the possibility of an allergic reaction to the local anesthetic

• vasoconstrictive properties Vasoconstrictors (i.e., epinephrine and levonordefrin) are added to local anesthetics because of their vasoconstrictive properties. Vasoconstriction at the site of injection is beneficial because it limits the uptake of the anesthetic by the vasculature, thereby increasing the duration of the anesthetic and diminishing systemic effects (reducing systemic toxicity). Note: The use of a vasopressor-containing local anesthetic also may actually be responsible for the sensation of burning on injection. The addition of a vasopressor and an antioxidant (sodium bisulfite) lowers the pH of the solution to between 3.3 and 4, significantly more acidic than solutions not containing a vasopressor (pH about 5.5). Patients are more likely to feel the burning sensation with these solutions. Note: Malamed's book states that "local anesthetics containing the vasoconstrictor levonordefrin (Neo-Cobefrin) have become impossible to obtain (June 2004)."Important: To minimize the likelihood of intravascular injection, aspiration should be performed before the local anesthetic solution is injected. If blood is aspirated, the needle must be repositioned until no return of blood can be elicited by aspiration. Adverse reactions following the administration of a local anesthetic are, in general, dose-related and may result from high plasma levels caused by excessive dosage, rapid absorption, or unintentional intravascular injection. Systemic toxicities of local anesthetics: Initial clinical signs and symptoms of mild to moderate toxicity include: talkativeness, apprehension, excitability, slurred speech, dizziness, and disorientation. The signs and symptoms of severe toxicity include: seizures, respiratory depression, coma, and death. Important: The excitatory manifestations may be very brief or may not occur at all, in which case the first manifestation of toxicity may be drowsiness merging into unconsciousness and respiratory arrest. Remember: Cardiovascular manifestations are usually depressant and are characterized by bradycardia, hypotension, and cardiovascular collapse, which may lead to cardiac arrest. Note: In local anesthesia, the depression of respiration is a manifestation of the toxic effects of the solution. 1. For a normal healthy (ASA I) patient, the maximum dose of epinephrine is 0.2 mg or 200 mg, this equates to roughly 11 cartridges of 1:100,000 epinephrine. (The maximum dose of lidocaine is 7 mg/kg of body weight. Thus, for healthy adult patients, epinephrine is usually the limiting factor.)2. In a cardiac risk patient, the maximum dose of epinephrine is 0.04 mg or 40 mg, which equates roughly to two cartridges of 1:100,000 epinephrine.

The tongue receives its blood supply from all of the following EXCEPT one. Which one is the EXCEPTION? tonsillar branch of the facial artery lingual artery vertebral artery ascending pharyngeal artery

• vertebral artery The lingual artery arises from the anterior surface of the external carotid artery, and travels obliquely toward the greater cornu of the hyoid bone. It loops upward and then passes deep to the posterior border of the hyoglossus muscle to enter the submandibular region. The loop of the artery is crossed superficially by the hypoglossal nerve. Branches include dorsal lingual artery, suprahyoid artery, and sublingual artery (which supplies sublingual gland). It terminates as the deep lingual artery, which ascends between the genioglossus and inferior longitudinal muscles. Note: The floor of the mouth also receives its blood supply from the lingual artery. Things to remember about the tongue: Motor innervation: from the hypoglossal nerve (CN XII). Sensory innervation: lingual (branch of trigeminal CN V3) supplies the anterior two-thirds, glossopharyngeal (CN IX) supplies the posterior one-third (including vallate papillae), vagus (CN X) through the internal laryngeal nerve supplies the area near the epiglottis.Note: Besides the posterior third of the tongue, the glossopharyngeal nerve also supplies sensory innervation to the tonsil, nasopharynx and pharyngeal areas. Taste: facial (CN VII) via chorda tympani supplies the anterior two-thirds; glossopharyngeal (CN IX) supplies the posterior one-third. Note: The vertebral arteries arise from the subclavian arteries and join to form the basilar artery. The basilar artery is the main blood supply to the brain stem and connects to the circle of Willis.


Conjuntos de estudio relacionados

Google Drive Research Questions Study Guide (Midterm version)

View Set

Urticaria & Hypersensitivity Syndromes

View Set

Bio 208 TEST 1 Anatomy Lecture Gardner CSULB

View Set

US History: Chapter 16 Mrs. Vaugh

View Set

AP gov Multiple Choice questions

View Set